Pathology SuperExam

December 27, 2017 | Author: CARES Martin Federic Palco | Category: Coagulation, Melanoma, White Blood Cell, Diseases And Disorders, Medicine
Share Embed Donate


Short Description

Ahsgsg...

Description

TOPNOTCH MEDICAL BOARD PREP PATHOLOGY SUPEREXAM For inquiries visit www.topnotchboardprep.com.ph or email us at [email protected] DEAR TOPNOTCH FRIENDS: PLEASE FOLLOW THESE INSTRUCTIONS: 1. These questions are previous diagnostic, midterm, and finals exams of Topnotch, almost all of them made by Topnotch Board Exam Topnotchers. 2. Answer this Topnotch Superexam seriously 100-items at a time. Cover the “Explanations” Column. Do not immediately look at the answers from the answer key. That’s not the correct way of answering sample exams. You need to treat these MCQs as exercises and not as handouts. 3. Time yourself. 1.5 hours per 100-item block. 4. After answering each 100-item block, refer to the Topnotch Answer Key for the correct answers. Please be careful of “frameshift mutations” when checking your answers – check every 10 items. (the format of the answer key was designed for you to practice against “frameshift mutations”) 5. The Topnotch Superexams are EXERCISES for the actual med boards. They will not appear verbatim in your future exams. More than knowing what’s the correct answer, it’s more important for you to: a. Know why the other choices are wrong b. Know why the other choices were included in the first place c. Know the explanation to the correct answer 6. Sharpen your mind by answering the Topnotch Superexams. Most of these questions based on past feedback are more difficult than the actual questions in the med boards. In these exams made by Board Exam Topnotchers, if you’re getting a score of 60/100 , that’s already a good score. More than 80/100 is outstanding. Item QUESTION EXPLANATION AUTHOR TOPNOTCH # EXAM 1 Which of the following is least likely to facilitate TGF - transforming growth factor. KRISTEL TANHUI DIAGNOSTIC chemotaxis: (TOP 3 - AUG 2015 EXAM - MARCH A. LTB4 Questions in the boards can sometimes be phrased as MED BOARDS; 2016 B. IL8 “least likely or most likely”, so try to accustom TOPNOTCH MD C. C5a yourself to choosing the best answer in such FROM LA SALLE) D. TGF circumstances. E. N-formylmethionine Source: Robbins and Cotran Pathologic Basis of Disease 8th ed p. 50 2 The following are anaphylotoxins The anaphylatoxins are components of the KRISTEL TANHUI DIAGNOSTIC A. C3a complement system which are involved in (TOP 3 - AUG 2015 EXAM - MARCH B. C4a anaphylaxis. Source: Robbins and Cotran Pathologic MED BOARDS; 2016 C. C5a Basis of Disease 8th ed p. 57 TOPNOTCH MD D. A and C only FROM LA SALLE) E. All of the above

3

A 56 year old hypertensive male presented with left sided hemiparesis on waking up. A CT scan was done which revealed an infarct in the distribution of the MCA. Which of the following is the expected gross pathological finding in the brain? A. Coagulative necrosis B. Caseous necrosis C. Liquefactive necrosis D. Fibrinoid necrosis E. Gangrenous necrosis

Infarcts in the brain result to liquefactive necrosis while those in all other organs except the brain exhibit coagulative necrosis. Source: Robbins and Cotran Pathologic Basis of Disease 8th ed p. 15

KRISTEL TANHUI (TOP 3 - AUG 2015 MED BOARDS; TOPNOTCH MD FROM LA SALLE)

DIAGNOSTIC EXAM - MARCH 2016

4

A 45 year old woman presents with recurrent infections and on PE was found to have marked splenomegaly. Her leukocyte count is increased to 300,000. The differential count reveals the presence of myeloblasts and promyelocytes, with predominance of myelocytes, metamyelocytes, bands and segmented neutrophils. Basophils are also increased in number. The patient is not anemic. Leukocyte alkaline phosphatase is decreased. Which of the following describes a major characteristic of this disorder? A. 9:22 translocation B. Expansion of mature B lymphocytes within multiple lymph nodes C. Hypogammaglobulinemia D. Neoplastic cells exhibiting hair-like filamentous projections E. Peak incidence occurs at 65 years A 50 year old man seeks consult due to a pruritic rash which he has had over the past 8 months. On PE, there were erythematous, eczematoid patches and raised plaques distributed asymmetrically over the chest and abdomen. On biopsy of the lesions, atypical CD4+ T cells with cerebriform nuclei were found. What is a possible outcome in the course of this condition? A. Acute leukemia B. Myelofibrosis C. Sezary Syndrome D. A and B only E. All of the above

This is a case of CML. It is associated with 9:22 translocation and may be treated with imatinib mesylate. Basophilia is a rare finding. It is strongly indicative of CML. There are a couple of conditions that cause increased WBC, either a leukemia or an infection. The increase in WBC count due to an infection is called a leukemoid reaction. This can be differentiated from CML via the leukocyte alkaline phosphatase test. In a leukemoid reaction LAP is increased. Source: Topnotch handout on Pathology.

KRISTEL TANHUI (TOP 3 - AUG 2015 MED BOARDS; TOPNOTCH MD FROM LA SALLE)

DIAGNOSTIC EXAM - MARCH 2016

This is a case of Mycosis fungoides, which is a T cell lymphoma of the skin. Atypical CD4+ T cells with cerebriform nuclei are found on biopsy. The disorder may remain confined to the skin for several years. When the neoplastic cells invade the skin and become systemic, this is called Sezary syndrome. Sezary syndrome is the leukemic form of this cutaneous T cell lymphoma and is characterized by the combination of skin lesions and circulating neoplastic cells. Acute leukemia and myelofibrosis are courses in the natural history of the myeloproliferative syndromes. Source: Topnotch handout on Pathology

KRISTEL TANHUI (TOP 3 - AUG 2015 MED BOARDS; TOPNOTCH MD FROM LA SALLE)

DIAGNOSTIC EXAM - MARCH 2016

5

TOPNOTCH MEDICAL BOARD PREP PATHOLOGY SUPEREXAM Page 1 of 99 For inquiries visit www.topnotchboardprep.com.ph or email us at [email protected]

TOPNOTCH MEDICAL BOARD PREP PATHOLOGY SUPEREXAM For inquiries visit www.topnotchboardprep.com.ph or email us at [email protected] Item # 6

QUESTION

EXPLANATION

AUTHOR

TOPNOTCH EXAM DIAGNOSTIC EXAM - MARCH 2016

A 70 year male presented with a 12 hour history of anginal chest pain. The stat troponin I and ECG studies confirm the diagnosis of acute myocardial infarction. Unfortunately, he succumbed to a fatal arrythmia during his 30 minute minute stay in the ER before a definitive intervention could be applied. What are the expected microscopic findings on LM for his heart? A. Dense collagenous scar B. Variable waviness of the fiber C. Coagulation necrosis with neutrophilic infiltrate D. Beginning disintegration of muscle fibers with macrophage infiltrate E. None, it’s too early for histopathologic changes to occur. Which of the following findings is an unlikely finding in malignant hypertension? A. Multiple punctate hemorrhage on the surface of both kidneys B. Trea-bark appearance of the ascending aorta C. Fibrinoid necrosis of arterioles D. Onion skinning of arterioles E. None of the above

0-4hrs: None to Variable waviness of the fiber 1-3 days: neutrophilic infiltrate 3-7 days: macrophage infiltrate 1-2 wks: granulation tissue >2 mos: dense collagenous scar Source: Robbins and Cotran Pathologic Basis of Disease 8th ed p. 550

KRISTEL TANHUI (TOP 3 - AUG 2015 MED BOARDS; TOPNOTCH MD FROM LA SALLE)

Trea-bark appearance of the ascending aorta is a characteristic of tertiary syphilis. Source: Robbins and Cotran Pathologic Basis of Disease 8th ed p. 950

KRISTEL TANHUI (TOP 3 - AUG 2015 MED BOARDS; TOPNOTCH MD FROM LA SALLE)

DIAGNOSTIC EXAM - MARCH 2016

8

A 65 year old female presents with fever, headache and diplopia. On palpation, tenderness is noted along the course of the temporal artery. A biopsy would most likely confirm which of the following diagnosis? A. Takayasu arteritis B. Retinoblastoma C. Giant cell arteritis D. Kaposi sarcoma E. Katayama disease

KRISTEL TANHUI (TOP 3 - AUG 2015 MED BOARDS; TOPNOTCH MD FROM LA SALLE)

DIAGNOSTIC EXAM - MARCH 2016

9

A 50 year old male was recently diagnosed with PTB. On physical examination, he appears emaciated. Which of the following is also called cachectin and is responsible for the weight loss and wasting noted in the patient? A. IL1 B. IL2 C. PAF D. TGF-beta E. TNF

This is a case of Giant cell arteritis. Along with involvement of the branches of the carotid artery, there is a close association with polymyalgia rheumatica. Takayasu arteritis is pulseless disease. It is more common in women of childbearing age. Initial symptoms are usually nonspecific, including fatigue, weight loss, and fever. With progression, vascular symptoms appear and dominate the clinical picture, including reduced blood pressure and weaker pulses in the upper extremities; ocular disturbances, including visual defects, retinal hemorrhages, and total blindness; and neurologic deficits. Involvement of the more distal aorta may lead to claudication of the legs; pulmonary artery involvement may cause pulmonary hypertension. Narrowing of the coronary ostia may lead to myocardial infarction, and involvement of the renal arteries leads to systemic hypertension in roughly half of patients. Source: Robbins and Cotran Pathologic Basis of Disease 8th ed p. 512 Tumor necrosis factor or cachectin is secreted by activated macrophages and is also responsible for the cachexia in cancer patients. Source: Robbins and Cotran Pathologic Basis of Disease 8th ed p. 320

KRISTEL TANHUI (TOP 3 - AUG 2015 MED BOARDS; TOPNOTCH MD FROM LA SALLE)

DIAGNOSTIC EXAM - MARCH 2016

10

Pneumoconiosis refers to nonneoplastic lung reaction to inhalation of mineral dusts encountered in the work place. Which pneumoconiosis is characterized by discrete pale to blackened nodules in the upper zones of the lungs. Radiographically it presents as eggshell calcifications. This disease may be progressive even if patient is no longer exposed and it has also been associated with increased susceptibility to PTB. A. Coal workers’ pneumoconiosis B. Silicosis C. Asbestosis D. Byssinosis E. Sarcoidosis

Silicosis – upper lobes, eggshell calcification, PTB Source: Robbins and Cotran Pathologic Basis of Disease 8th ed p. 699

KRISTEL TANHUI (TOP 3 - AUG 2015 MED BOARDS; TOPNOTCH MD FROM LA SALLE)

DIAGNOSTIC EXAM - MARCH 2016

7

TOPNOTCH MEDICAL BOARD PREP PATHOLOGY SUPEREXAM Page 2 of 99 For inquiries visit www.topnotchboardprep.com.ph or email us at [email protected]

TOPNOTCH MEDICAL BOARD PREP PATHOLOGY SUPEREXAM For inquiries visit www.topnotchboardprep.com.ph or email us at [email protected] Item # 11

QUESTION

EXPLANATION

AUTHOR

A 40 yr old male presents with fever and cough of 5 days duration. PE revealed increased bronchial breath sounds over the lower segment of the right lobe posteriorly. Chest xray showed lobar consolidation on the right lower lobe and culture was positive for pneumonococcus. Which is the prominent inflammatory cells of this exudate? A. Platelets B. Basophils C. Eosinophils D. Neutrophils E. Macrophage

Patient has bacterial pneumonia. Streptococcus is a popular agent for causing lobar pneumonia. Stages of inflammatory response in lobar pneumonia: - Congestion: red, heavy, boggy lung - Red hepatization: massive, confluent exudation with neutrophils, red cells and fibrin - Gray hepatization - Resolution Source: Robbins and Cotran Pathologic Basis of Disease 8th ed p.712

KRISTEL TANHUI (TOP 3 - AUG 2015 MED BOARDS; TOPNOTCH MD FROM LA SALLE)

12

19 year old male presents with acute onset hematuria, oliguria and periorbital edema which is worse in the morning and gradually improves through the day. On PE, blood pressure is elevated and urinalysis shows trace proteinuria, and many RBCs. ASOT is negative and DNAse is positive. Which is the expected finding in light microscopy for this patient? A. Diffuse endocapillary proliferation B. Subepithelial humps C. Normal appearing D. Dense deposits E. Extracapillary proliferation of crescents

KRISTEL TANHUI (TOP 3 - AUG 2015 MED BOARDS; TOPNOTCH MD FROM LA SALLE)

DIAGNOSTIC EXAM - MARCH 2016

13

Patient sought consult for chest pain and hemoptysis associated with hematuria, and signs and symptoms of uremia. His blood pressure is elevated and he has grade III bipedal edema. As the clinician in charge, you suspect that he is suffering from an autoimmune disease in which antibodies against type IV collagen attack the basement membrane of the lungs and kidneys. On immunofluorescence renal biopsy would most probably reveal: A. Granular IgG and C3 in GBM and mesangium B. Linear IgG and C3 C. Negative D. Focal IgM or C3 E. IgA in the mesangium

When you study for patho and all basic sciences, try to make sure you know how the disease will present clinically cause they like to correlate things. The hardcore micro and biochem or patho knowledge will only get you halfway cause sometimes they will not give you the diagnosis. This is a case of PSGN. ASOT may be negative if the nephritic strain comes from a skin infection because skin lipids bind to streptolysin O. DNAse is the most sensitive test for skin infection with Group A streptococcus. The patient in this case presented with the classic nephritic syndrome of hypertension, hematuria and oliguria. A and B are both PSGN but B is an electron microscopy finding. Source: Robbins and Cotran Pathologic Basis of Disease 8th ed p.918 Don’t just focus on the electron and LM findings and sacrifice immunofluorescence. SIMILAR TO PREVIOUS BOARD EXAM CONCEPT/PRINCIPLE. A rule of thumb is if the pathophysiology involves immune complex deposition, it would usually present with a granular pattern. If the pathophysiology involves antibodies against basement membranes, the pattern is usually linear. This is actually a case of Goodpasture syndrome which involves antibodies against the basement membrane of the lungs and the kidneys and the findings on immunofluorescent studies show linear IgG and C3. Source: Robbins and Cotran Pathologic Basis of Disease 8th ed p. 709, 918

KRISTEL TANHUI (TOP 3 - AUG 2015 MED BOARDS; TOPNOTCH MD FROM LA SALLE)

DIAGNOSTIC EXAM - MARCH 2016

14

Patent urachus is a risk factor for which type of carcinoma of the bladder? A. Transitional cell carcinoma B. Squamous cell carcinoma C. Adenocarcinoma D. Clear cell carcinoma E. Sarcoma

Transitional cell carcinoma is associated with smoking. Squamous cell carcinoma is associated with chronic irritation and S. haematobium infection. Source: Robbins and Cotran Pathologic Basis of Disease 8th ed p. 979

KRISTEL TANHUI (TOP 3 - AUG 2015 MED BOARDS; TOPNOTCH MD FROM LA SALLE)

DIAGNOSTIC EXAM - MARCH 2016

15

In which of the following nutritional deficiency is hepatic steatosis an expected finding? A. Vitamin A deficiency B. Vitamin E deficiency C. Copper deficiency D. Kwashiorkor E. Marasmus

Kwashiorkor is a protein deficiency while marasmus a balanced deficiency of all macronutrients. Source: Robbins and Cotran Pathologic Basis of Disease 8th ed p. 429

KRISTEL TANHUI (TOP 3 - AUG 2015 MED BOARDS; TOPNOTCH MD FROM LA SALLE)

DIAGNOSTIC EXAM - MARCH 2016

16

Which of the following is not a characteristic of a benign peptic ulcer? A. Sharply punched out defect B. Heaped-up margins C. Hemorrhage and fibrin deposition in the gastric serosa D. Perforation E. Malignant transformation is very rare

Benign ulcers may have mucosal margins which overhang the base slightly but is usually level with the surrounding mucosa. Heaped up margins are more characteristic of malignant ulcers. Source: Robbins and Cotran Pathologic Basis of Disease 8th ed p. 780

KRISTEL TANHUI (TOP 3 - AUG 2015 MED BOARDS; TOPNOTCH MD FROM LA SALLE)

DIAGNOSTIC EXAM - MARCH 2016

TOPNOTCH MEDICAL BOARD PREP PATHOLOGY SUPEREXAM Page 3 of 99 For inquiries visit www.topnotchboardprep.com.ph or email us at [email protected]

TOPNOTCH EXAM DIAGNOSTIC EXAM - MARCH 2016

TOPNOTCH MEDICAL BOARD PREP PATHOLOGY SUPEREXAM For inquiries visit www.topnotchboardprep.com.ph or email us at [email protected] Item # 17

18

19

20

21

22

QUESTION

EXPLANATION

AUTHOR

TOPNOTCH EXAM DIAGNOSTIC EXAM - MARCH 2016

Which of the following is the least likely characteristic of right-sided colonic malignancy? A. Adenocarcinoma B. Napkin ring constriction and luminal narrowing C. Desmoplastic response D. Liver metastasis E. None of the above Which of the following is the most common cause of chronic pancreatitis? A. Gallstones B. Alcoholism C. Hypercholesterolemia D. Smoking E. Hereditary predisposition

Napkin ring constrictions are characteristic of left sided colonic malignancy. Right sided lesions present as polypoid, exophytic masses. Left sided lesions present clinically as obstruction while right sided ones present as anemia. Source: Robbins and Cotran Pathologic Basis of Disease 8th ed p. 824

KRISTEL TANHUI (TOP 3 - AUG 2015 MED BOARDS; TOPNOTCH MD FROM LA SALLE)

Most common cause Acute pancreatitis – gallstones Chronic pancreatitis – alcoholism Source: Robbins and Cotran Pathologic Basis of Disease 8th ed p. 896

KRISTEL TANHUI (TOP 3 - AUG 2015 MED BOARDS; TOPNOTCH MD FROM LA SALLE)

DIAGNOSTIC EXAM - MARCH 2016

A 22 year old male consults for ptosis and diplopia which usually begins to occur in the late afternoon and improves upon waking up after a long good sleep. Recently he also noted dysphagia and muscle weakness which also improves with rest. What is the pathophysiology of his condition? A. Antibodies to presynaptic Ca channel preventing release of Acetylcholine B. Antibodies to Acetylcholine receptors C. Autoimmune peripheral demyelination D. Inflammation of the spinal cord E. Patient appears to be faking it. He should be referred to psych. HSV 1 is the most common cause of viral encephalitis. Which is the least likely expected pathological finding? A. The encephalitis most severely affects the frontal lobe. B. The infection is necrotizing and often hemorrhagic in the most severely affected regions C. Cowdry type A intranuclear viral inclusion bodies may be found D. All of the above E. None of the above A 4-year-old boy presents with recurrent joint pain involving the knees and hips. He had always bruised easily, and recently the parents had seen blood in his urine. A presumptive diagnosis of classic hemophilia (hemophilia A) is made, and coagulation blood tests are performed. Which of the following is the most likely set of findings of coagulation screening tests? A. Normal bleeding time, platelet count, and thrombin time; prolonged PT and APTT B. Normal bleeding time, platelet count, thrombin time, and APTT; prolonged PT C. Normal bleeding time, platelet count, thrombin time, and PT; prolonged APTT D. Normal platelet count and thrombin time; prolonged bleeding time, PT, and APTT E. Prolonged bleeding time, PT, APTT, and thrombin time; decreased platelet count A 9-year-old girl is diagnosed with acute rheumatic fever. Instead of recovering as expected, her condition worsens, and she dies. Which of the following is the most likely cause of death? A. Central nervous system involvement B. Endocarditis C. Myocarditis D. Pericarditis E. Streptococcal sepsis

This is a case of myasthenia gravis Antibodies to presynaptic Ca channel preventing release of Acetylcholine – Lambert Eaton myasthenic syndrome (paraneoplastic syndrome) Source: Robbins and Cotran Pathologic Basis of Disease 8th ed p. 1275

KRISTEL TANHUI (TOP 3 - AUG 2015 MED BOARDS; TOPNOTCH MD FROM LA SALLE)

DIAGNOSTIC EXAM - MARCH 2016

Accustom yourself to answering question phrased as most likely or least likely. Examiners may want to phrase it that way. HSV1 encephalitis classically most severely affects the temporal lobe. It most commonly presents as alterations in mood, behavior and memory.

KRISTEL TANHUI (TOP 3 - AUG 2015 MED BOARDS; TOPNOTCH MD FROM LA SALLE)

DIAGNOSTIC EXAM - MARCH 2016

SIMILAR TO PREVIOUS BOARD EXAM CONCEPT/PRINCIPLE. Classic hemophilia (factor VIII deficiency) is an abnormality of the intrinsic pathway of coagulation proximal to the final common pathway, which begins at factor X → Xa activation. This defect leads to a prolonged APTT. The other laboratory tests listed remain normal, because the bleeding time is a measure of platelet plug formation, the PT is a measure of the extrinsic pathway of coagulation, and the thrombin time is an assay of the conversion of fibrinogen to fibrin. The presumptive diagnosis is confirmed by specific factor VIII assay.

LESTER BRYAN CO (TOP 10 - AUG 2015 MED BOARDS; TOPNOTCH MD FROM UST)

MIDTERM 1 EXAM - MARCH 2016

The most common cause of death that occurs during acute rheumatic fever is cardiac failure secondary to myocarditis.

LESTER BRYAN CO (TOP 10 - AUG 2015 MED BOARDS; TOPNOTCH MD FROM UST)

MIDTERM 1 EXAM - MARCH 2016

TOPNOTCH MEDICAL BOARD PREP PATHOLOGY SUPEREXAM Page 4 of 99 For inquiries visit www.topnotchboardprep.com.ph or email us at [email protected]

TOPNOTCH MEDICAL BOARD PREP PATHOLOGY SUPEREXAM For inquiries visit www.topnotchboardprep.com.ph or email us at [email protected] Item # 23

24

25

26

27

QUESTION

EXPLANATION

A 25-year-old man presents with hematuria, SIMILAR TO PREVIOUS BOARD EXAM periorbital edema, hypertension, and CONCEPT/PRINCIPLE. The clinical description is that hemoptysis. He has also experienced nausea, of Goodpasture syndrome (antiglomerular basement vomiting, fever, and chills. Serologic testing is membrane disease), caused by antibodies directed positive for antiglomerular basement against antigens in the glomerular and pulmonary membrane antibodies. Which of the following is alveolar basement membranes. Because antigens are the classic histologic finding in this renal an intrinsic component of the basement membrane, disease? labeled antibodies “paint” the surface of the basement A. Linear immunofluorescence membrane, resulting in the characteristic linear B. “Lumpy-bumpy” immunofluorescence immunofluorescent pattern characteristic of this C. “Spike and dome” appearance of the disorder. “Lumpy-bumpy” immunofluorescence is glomerular basement membrane very coarse, granular immunofluorescence found in D. Subendothelial immune complex deposition poststreptococcal immune complex deposit disease. E. Tram-track appearance of the glomerular The tram-track appearance is seen in basement membrane on electron microscopy membranoproliferative glomerulonephritis. The “spike and dome” appearance is seen in membranous glomerulonephritis. Subendothelial immune complex deposition is seen in lupus nephropathy. A 19-year-old young woman who emigrated Inflammation and stenosis of branches of the aortic from Taiwan 8 years ago presents with fever, arch is known as Takayasu arteritis, or “pulseless malaise, myalgias, and arthritis and “coldness” disease.” It most commonly occurs in young Asian in her upper extremities. She has a weak radial females. Buerger disease usually affects young Jewish pulse bilaterally, and a magnetic resonance males and involves the arteries of the extremities. The angiogram demonstrates nearly 75% stenosis disease is exacerbated by smoking and can lead to of the main arteries originating from the aorta. gangrene of the extremities. Kawasaki disease affects She likely has which of the following the branches of the coronary arteries. Raynaud rheumatologic conditions? disease is due to vasospasm of small vessels of the A. Buerger disease fingers and toes, leading to cyanosis and pallor of the B. Kawasaki disease affected tissues. Temporal arteritis is usually C. Raynaud disease encountered in older patients and affects the branches D. Takayasu arteritis of the carotid artery, most commonly the temporal E. Temporal arteritis artery. A 45-year-old man presents with involuntary This is a case of Huntington disease, which is an facial grimaces and movements of the fingers. autosomal dominant, fatal, progressive degeneration His mother had had similar symptoms and atrophy of the striatum (caudate nucleus and beginning at about the same age. Her disorder putamen). The disorder is characterized by an had progressed to dancing movements, increased number of trinucleotide (CAG) repeats in writhing of the arms and legs, and eventually the HD (huntingtin) gene on the short arm of coma and death. His maternal grandfather had chromosome 4. Degeneration of the upper and lower had a similar disorder but at an age older than motor neurons is characteristic of ALS. Dopamine the mother. Which of the following is most depletion and depigmentation of the substantia nigra characteristic of this disease? is characteristic of Parkinson disease. Neurofibrillary A. Degeneration of upper and lower motor tangles and amyloid plaques are found in Alzheimer neurons disease. Pick bodies can be found in Pick disease, B. Dopamine depletion and depigmentation of which clinically resembles Alzheimer disease. the substantia nigra C. Increased number of trinucleotide repeats in a gene on chromosome 4 D. Neurofibrillary tangles and amyloid plaques in the cerebral cortex E. Pick bodies, characterized by round intracytoplasmic inclusions consisting of neurofilaments A 14-year-old girl presents with prolonged von Willebrand disease, a disorder transmitted by bleeding from wounds and minor trauma and autosomal modes of inheritance (both dominant and severe menorrhagia. Family history reveals that recessive) is the most common hereditary bleeding her father also has prolonged bleeding from disorder. There are many variants, all marked by wounds and minor trauma, as does her brother. either qualitative or quantitative deficiencies of vWF. Which of the following is the most likely mechanism of this patient’s disorder? A. Absence of platelet glycoprotein IIb-IIIa B. Antiplatelet antibodies reacting with platelet surface glycoproteins C. Deficiency of factor VIII D. Deficiency of factor IX E. Deficiency of vWF A 42-year-old man is seen because of a long Cardiomyopathies are noninflammatory myocardial history of slowly developing congestive heart disorders that are not associated with coronary artery failure. His blood pressure is normal. Coronary obstruction, hypertension, valvular disease, congenital artery angiography reveals no vascular disease. heart disease, or infectious disease. They are most No heart murmurs are heard. The white blood often characterized by otherwise unexplained cell count, differential, and erythrocyte ventricular dysfunction, such as cardiac failure, sedimentation rate are normal. The most likely ventricular enlargement, or ventricular arrhythmias. diagnosis is A. carcinoid heart disease B. cardiomyopathy C. coarctation of the aorta D. constrictive pericarditis E. myocardial infarction

AUTHOR LESTER BRYAN CO (TOP 10 - AUG 2015 MED BOARDS; TOPNOTCH MD FROM UST)

TOPNOTCH EXAM MIDTERM 1 EXAM - MARCH 2016

LESTER BRYAN CO (TOP 10 - AUG 2015 MED BOARDS; TOPNOTCH MD FROM UST)

MIDTERM 1 EXAM - MARCH 2016

LESTER BRYAN CO (TOP 10 - AUG 2015 MED BOARDS; TOPNOTCH MD FROM UST)

MIDTERM 1 EXAM - MARCH 2016

LESTER BRYAN CO (TOP 10 - AUG 2015 MED BOARDS; TOPNOTCH MD FROM UST)

MIDTERM 1 EXAM - MARCH 2016

LESTER BRYAN CO (TOP 10 - AUG 2015 MED BOARDS; TOPNOTCH MD FROM UST)

MIDTERM 1 EXAM - MARCH 2016

TOPNOTCH MEDICAL BOARD PREP PATHOLOGY SUPEREXAM Page 5 of 99 For inquiries visit www.topnotchboardprep.com.ph or email us at [email protected]

TOPNOTCH MEDICAL BOARD PREP PATHOLOGY SUPEREXAM For inquiries visit www.topnotchboardprep.com.ph or email us at [email protected] Item # 28

29

30

31

32

33

QUESTION

EXPLANATION

AUTHOR

A 28-year-old woman complains of fatigue, SIMILAR TO PREVIOUS BOARD EXAM LESTER BRYAN CO dyspnea, and malaise. She also notes that her CONCEPT/PRINCIPLE. Paroxysmal nocturnal (TOP 10 - AUG 2015 urine has been reddishbrown, particularly with hemoglobinuria results in an acquired MED BOARDS; the first void of the morning. Subsequent intracorpuscular defect in the ability to synthesize GPI TOPNOTCH MD studies confirm that she has paroxysmal anchors required for appropriate placement of FROM UST) nocturnal hemoglobinuria. Which of the complement regulatory proteins on the surface of red following best describes the defect leading to blood cells. Functional deficiency of such proteins as this condition? CD55 and CD59 renders the cells sensitive to A. Anti-intrinsic factor antibodies complement-mediated lysis. Anti-intrinsic factor B. Deficiency of the intracellular structural antibodies are seen in pernicious anemia. Spectrin is protein spectrin deficient in hereditary spherocytosis. Ineffective C. Impaired synthesis of the cell-surface GPI erythropoiesis is seen in megaloblastic anemia due to anchor folate or vitamin B12 deficiencies. Substitution of D. Ineffective erythropoiesis valine for glutamic acid in the β-globin gene underlies E. Substitution of a valine for a glutamate the defect in sickle cell anemia. residue in the β-globin gene A 45-year-old man presents with abdominal Berry aneurysms, which occur in 10% to 15% of LESTER BRYAN CO pain and hypertension. On physical patients with adult polycystic kidney disease, are (TOP 10 - AUG 2015 examination, he is found to have an abdominal small saccular lesions that develop at the site of MED BOARDS; mass. Further workup confirms the diagnosis of congenital weakness of cerebral arteries, especially TOPNOTCH MD adult polycystic kidney disease. Which of the those of the circle of Willis. Rupture of these FROM UST) following vascular complications is associated aneurysms is the most common cause of subarachnoid with this condition? hemorrhage. Arteriovenous fistulas are often A. Arteriovenous fistula secondary to trauma. Dissecting aneurysm is B. Atherosclerotic aneurysm associated with hypertension or with diseases C. Berry aneurysm affecting the vascular media, most notably Marfan D. Dissecting aneurysm syndrome. Syphilitic (luetic) aneurysm is associated E. Luetic aneurysm with tertiary syphilis. A 55-year-old man presents with a large, blackMalignant melanoma arises from melanocytes or LESTER BRYAN CO colored, asymmetric skin lesion with ill-defined nevus cells, is most often associated with excessive (TOP 10 - AUG 2015 borders on his back. He reports a family history sun exposure, and is most common in fair-skinned MED BOARDS; of malignant melanoma. Which of the following persons. Of the clinical variants of malignant TOPNOTCH MD clinical variants of malignant melanoma has the melanoma, nodular melanoma has the worst FROM UST) poorest prognosis? prognosis. Malignant melanomas have a better A. Lentigo maligna melanoma prognosis when characterized by a long period of B. Superficial spreading melanoma radial (superficial) growth, as opposed to early C. Nodular melanoma vertical growth (as in nodular melanoma). D. Acral-lentiginous melanoma E. uveal melanoma When ordering academic attire for a recent A mosaic pattern of bone caused by increases in both LESTER BRYAN CO graduation, a 65-year-old university professor osteoblastic and osteoclastic activity is characteristic (TOP 10 - AUG 2015 is surprised to find that his hat size has of Paget disease of bone (osteitis deformans). Serum MED BOARDS; increased. Shortly thereafter, in a routine alkaline phosphatase is markedly increased. Hearing TOPNOTCH MD checkup, serum alkaline phosphatase activity is loss is common (from narrowing of the auditory FROM UST) found to be markedly elevated. Serum calcium foramen and compression of the eighth cranial nerve), and phosphorus are normal. Examination and an increase in hat size due to frontal bossing is reveals enlargement of the skull with frontal often noted. bossing and enlarged maxilla, and hearing loss is evident. Which of the following abnormalities is associated with the bone disorder suggested by these findings? A. Brown tumor of bone B. Defective calcification of osteoid matrix C. Mosaic pattern of bone D. Polyostotic fibrous dysplasia with severe deformity E. Subperiosteal hemorrhage and osteoporosis A 3-year-old African-American man with a Sickle cell anemia is the most common hereditary LESTER BRYAN CO history since early childhood of severe anemia anemia in persons of African lineage. Leg ulcers and (TOP 10 - AUG 2015 requiring many transfusions has nonhealing leg recurring painful crises are characteristic. In sickle cell MED BOARDS; ulcers and recurrent periods of abdominal and anemia, in contrast to sickle cell trait, sickle cells are TOPNOTCH MD chest pain. These signs and symptoms are most often seen on the peripheral blood smear. FROM UST) likely to be associated with which of the following laboratory abnormalities? A. Decreased erythropoietin B. Increased erythrocyte osmotic fragility C. Schistocytes D. Sickle cells on peripheral blood smear E. Teardrop-shaped cells For the past week, a 65-year-old woman has Pseudomembranous colitis is caused by overgrowth of LESTER BRYAN CO been treated for a severe infection with broadC. difficile. This organism produces exotoxin that (TOP 10 - AUG 2015 spectrum antibiotics, and she had recovered induces necrosis of the superficial mucosa, leading to MED BOARDS; well. Over the past day, however, she has pseudomembrane formation. The bacteria itself does TOPNOTCH MD developed foul-smelling, voluminous, greenish, not invade the mucosa. This condition most often FROM UST) watery diarrhea, as well as abdominal pain and occurs in patients with a history of broad-spectrum fever. Which of the following is the mechanism antibiotic use, because elimination of normal associated with this condition? intestinal flora promotes overgrowth of C. difficile. A. Aggregation of bacterial colonies on the lumen, forming pseudomembranes B. Bacterial release of exotoxin, inducing necrosis of the mucosa C. Physical invasion of bacteria into the superficial mucosa, leading to pseudomembrane formation D. Selective killing of C. difficile bacteria by antibiotics

TOPNOTCH MEDICAL BOARD PREP PATHOLOGY SUPEREXAM Page 6 of 99 For inquiries visit www.topnotchboardprep.com.ph or email us at [email protected]

TOPNOTCH EXAM MIDTERM 1 EXAM - MARCH 2016

MIDTERM 1 EXAM - MARCH 2016

MIDTERM 1 EXAM - MARCH 2016

MIDTERM 1 EXAM - MARCH 2016

MIDTERM 1 EXAM - MARCH 2016

MIDTERM 1 EXAM - MARCH 2016

TOPNOTCH MEDICAL BOARD PREP PATHOLOGY SUPEREXAM For inquiries visit www.topnotchboardprep.com.ph or email us at [email protected] Item #

QUESTION

EXPLANATION

AUTHOR

TOPNOTCH EXAM

E. Spread of the previous infection to the colon

34

35

36

37

38

A 50-year-old woman with a 20-year history of SIMILAR TO PREVIOUS BOARD EXAM type 2 diabetes mellitus presents with CONCEPT/PRINCIPLE. Diabetic nephropathy proteinuria, hypoalbuminemia, edema, and manifests clinically as the nephrotic syndrome; hyperlipidemia. She has not monitored her however, this syndrome is compounded by renal serum glucose levels over the past several failure and hypertension. Ultrastructural changes years. What is the classic morphologic finding include a marked increase in the thickness of the in this condition? glomerular basement membrane and mesangial A. Crescentic formation in glomeruli on light accumulation of glycosylated basement membranelike microscopy material. Light microscopy findings include diffuse B. Intramembranous and epimembranous glomerulosclerosis (a diffuse increase in mesangial immune complex deposits in the glomerular matrix) and nodular glomerulosclerosis (nodular basement membrane on electron microscopy accumulations of mesangial matrix). C. Nodular accumulations of mesangial matrix on light microscopy D. Sclerosis within capillary tufts that involves only some glomeruli and only parts of affected glomeruli on light microscopy E. Wire-loop abnormalities from immune complex deposits and thickening of the glomerular basement membrane on light microscopy A 23-year-old woman presents with cervical SIMILAR TO PREVIOUS BOARD EXAM and mediastinal lymphadenopathy. Biopsy of a CONCEPT/PRINCIPLE. The diagnosis is Hodgkin cervical lymph node reveals a nodular lymphoma, nodular sclerosing subtype. This form of appearance with fibrous bands, effacement of Hodgkin lymphoma differs from other forms of the lymph node architecture, and numerous classical Hodgkin lymphoma in being the most lacunar cells. Which of the following is common in young women, having a relatively characteristic of this disorder? favorable clinical course, and having little association A. Benign neoplasm with EBV infection. Lacunar cells are considered a B. Frequent association with EBV infection Reed-Sternberg cell variant, and the diagnosis of NS C. Most often a complication of human can be based on the finding of fibrous bands and immunodeficiency virus infection lacunar cells. D. Peak incidence in early childhood E. Relatively favorable clinical course A 15-year-old boy presents with a pathologic In young patients, bone malignancies showing fracture following a minor injury on the soccer prominent cartilaginous differentiation are almost field. The area of fracture is surrounded by a assuredly chondroblastic osteosarcomas, rather than large tumor which shows marked chondrosarcomas. Conventional chondrosarcomas pleomorphism, high mitotic activity, and occur almost exclusively in older patients. extensive cartilaginous differentiation on microscopy. The most likely diagnosis is A. giant cell tumor B. osteochondroma C. chondrosarcoma D. osteosarcoma E. Ewing sarcoma A 45-year-old woman presents to her primary Primary biliary cirrhosis is an autoimmune condition care physician with jaundice, pruritus, and that typically presents in middle-aged women. The periocular and intradigital xanthomas. Her itching and hypercholesterolemia are secondary to laboratory results indicate a significantly severe obstructive jaundice. Leptospirosis is a increased alkaline phosphatase as well as a condition caused by a treponemal bacterium that positive test for antimitochondrial antibodies. results in jaundice, renal failure, and hemorrhagic The most likely cause of her symptoms is phenomena. Macronodular cirrhosis is usually a result A. leptospirosis of hepatitis B or hepatitis C infection. Primary B. macronodular cirrhosis sclerosing cholangitis is associated with ulcerative C. primary biliary cirrhosis colitis and with an increased incidence of D. primary sclerosing cholangitis cholangiocarcinoma. Secondary biliary cirrhosis is E. secondary biliary cirrhosis caused by extrahepatic biliary obstruction. The chest radiograph of a 23-year-old medical Cavitation occurs only in secondary tuberculosis. Both student reveals a calcified cavitary pulmonary primary and secondary tuberculosis are characterized lesion. The tuberculin test is positive, but by caseating granulomas, often with Langhans giant sputum smears and cultures are negative for cells, which heal by scarring and calcification. The skin Mycobacterium tuberculosis. A presumptive test result for tuberculin sensitivity is positive in both diagnosis of secondary tuberculosis is made. If forms. further studies, including a biopsy, were performed, which of the following findings would justify the diagnosis of secondary tuberculosis, as contrasted to primary tuberculosis? A. Calcification B. Caseating granulomas C. Cavitation D. Langhans giant cells

LESTER BRYAN CO (TOP 10 - AUG 2015 MED BOARDS; TOPNOTCH MD FROM UST)

MIDTERM 1 EXAM - MARCH 2016

LESTER BRYAN CO (TOP 10 - AUG 2015 MED BOARDS; TOPNOTCH MD FROM UST)

MIDTERM 1 EXAM - MARCH 2016

LESTER BRYAN CO (TOP 10 - AUG 2015 MED BOARDS; TOPNOTCH MD FROM UST)

MIDTERM 1 EXAM - MARCH 2016

LESTER BRYAN CO (TOP 10 - AUG 2015 MED BOARDS; TOPNOTCH MD FROM UST)

MIDTERM 1 EXAM - MARCH 2016

LESTER BRYAN CO (TOP 10 - AUG 2015 MED BOARDS; TOPNOTCH MD FROM UST)

MIDTERM 1 EXAM - MARCH 2016

TOPNOTCH MEDICAL BOARD PREP PATHOLOGY SUPEREXAM Page 7 of 99 For inquiries visit www.topnotchboardprep.com.ph or email us at [email protected]

TOPNOTCH MEDICAL BOARD PREP PATHOLOGY SUPEREXAM For inquiries visit www.topnotchboardprep.com.ph or email us at [email protected] Item #

QUESTION

EXPLANATION

AUTHOR

TOPNOTCH EXAM

E. Positive tuberculin test result

39

A 40-year-old woman presents with a painless mass anterior to her left ear. The mass had been slowly enlarging over the past year. The mass is firm and nontender. Computed tomography and magnetic resonance imaging reveal a well-circumscribed, homogeneous mass within the left parotid gland. Biopsy reveals anastomosing strands of stellate and fusiform epithelial cells embedded in a myxoid stroma. Which of the following is a characteristic of the lesion? A. It is also called papillary cystadenoma lymphomatosum. B. It is most often localized to the submandibular gland. C. It is the most common malignant salivary gland tumor. D. Recurrence often takes place after surgical resection. E. Surgical resection should not be performed, because this condition is usually already metastatic on diagnosis. A 70-year-old man presents with severe bone pain and frequent respiratory infections. Serum protein electrophoresis demonstrates an M protein spike in the gamma region. Radiographs of the skull, long bones, and spine demonstrate multiple “punched-out” lesions, and bone marrow aspiration demonstrates large numbers of neoplastic plasma cells. Which of the following statements is true of this disorder? A. Although this patient presents at 70 years of age, the average age of presentation is 50 years of age. B. Renal insufficiency is a common cause of death. C. The M spike is most often an IgM. D. The M spike is most often polyclonal in nature. E. This disorder is the most common T-cell neoplasm. Spongiotic dermatitis is the histologic pattern for which of the following dermatoses? A. Discoid lupus erythematosus B. Contact dermatitis C. Psoriasis D. Erythema nodosum E. Erythema multiforme

Approximately 80% to 90% of salivary gland tumors originate in the parotid gland and, of these, approximately 70% are pleomorphic adenomas. The term “mixed tumor” properly applies to this benign tumor, which often demonstrates myxoid and cartilage-like elements in addition to stellate or fusiform epithelial cells. Complete surgical resection is difficult because of the tumor’s proximity to the facial nerve, and, thus, recurrence is frequent.

LESTER BRYAN CO (TOP 10 - AUG 2015 MED BOARDS; TOPNOTCH MD FROM UST)

MIDTERM 1 EXAM - MARCH 2016

The diagnosis is multiple (plasma cell) myeloma, a neoplastic proliferation of malignant plasma cells (mature B cells, not T cells). Death is often caused by renal insufficiency caused by myeloma kidney. The average age of presentation is approximately 70 years of age. IgM myeloma is very uncommon. Both the neoplastic cells and the serum protein spike are monoclonal rather than polyclonal, and the monoclonal spike protein is most frequently an IgG or an IgA.

LESTER BRYAN CO (TOP 10 - AUG 2015 MED BOARDS; TOPNOTCH MD FROM UST)

MIDTERM 1 EXAM - MARCH 2016

Contact dermatitis is a type of spongiotic dermatitis with edema and perivascular lymphocytic infiltrate.

GEORGE MICHAEL SOSUAN (TOP 5 - AUG 2015 MED BOARDS; TOPNOTCH MD FROM UST)

MIDTERM 2 EXAM - MARCH 2016

42

Oral lesions may be seen secondary to fungal pulmonary infections which spread by hematogenously or by direct inoculation of infected sputum. Which of the following maybe used to stain these fungal elements A. Gomori methenamine stain B. PAS C. Gram's stain D. Both A and B E. AOTA

Special histochemical stains (Grocott-Gomori methenamine silver (GMS), or periodic acid-Schiff (PAS) highlight fungi in sections.

GEORGE MICHAEL SOSUAN (TOP 5 - AUG 2015 MED BOARDS; TOPNOTCH MD FROM UST)

MIDTERM 2 EXAM - MARCH 2016

43

A 72 y/o patient with diabetes complains of severe pain and bloody discharge of the right ear. Which of the following would a Gram stain of material from surgical debridement be expected to show? A. Gram-positive cocci B. Gram-negative cocci C. Gram-positive bacilli D. Gram-negative bacilli E. Septate, branching hyphae

Malignant otitis externa is most commonly associated with Pseudomonas aeruginosa, which is a gramnegative bacillus.

GEORGE MICHAEL SOSUAN (TOP 5 - AUG 2015 MED BOARDS; TOPNOTCH MD FROM UST)

MIDTERM 2 EXAM - MARCH 2016

40

41

TOPNOTCH MEDICAL BOARD PREP PATHOLOGY SUPEREXAM Page 8 of 99 For inquiries visit www.topnotchboardprep.com.ph or email us at [email protected]

TOPNOTCH MEDICAL BOARD PREP PATHOLOGY SUPEREXAM For inquiries visit www.topnotchboardprep.com.ph or email us at [email protected] Item # 44

45

QUESTION Which of the following is incorrectly matched? A. Bronchopneumonia: Patchy multifocal consolidation B. Lobar pneumonia: Lobar consolidation C. Atypical pneumonia: Inflammation within alveolar interstitium D. AOTA E. NOTA Leading cause of blood transfusion related mortality A. Transfusion related acute lung injury B. Graft versus host reaction C. Hemolytic transfusion reaction D. Sepsis E. Anaphylaxis

EXPLANATION

AUTHOR

TOPNOTCH EXAM MIDTERM 2 EXAM - MARCH 2016

All of them are correct

GEORGE MICHAEL SOSUAN (TOP 5 - AUG 2015 MED BOARDS; TOPNOTCH MD FROM UST)

TRALI is currently the leading cause of transfusion mortality. Donor antibodies to recipient leukocyte antigens activate complement and encourage granulocytes to aggregate within the pulmonary microvasculature. The diagnosis is based on clinical findings that are similar to ARDS but arise within 6 hours after transfusion. TRALI has a 5–10% mortality rate, but with interim ventilatory support, most symptoms resolve within 96 hours. In patients with cardiac septal defects, an embolus may detour into the left heart as a paradoxical embolus to organs and the brain.

GEORGE MICHAEL SOSUAN (TOP 5 - AUG 2015 MED BOARDS; TOPNOTCH MD FROM UST)

MIDTERM 2 EXAM - MARCH 2016

GEORGE MICHAEL SOSUAN (TOP 5 - AUG 2015 MED BOARDS; TOPNOTCH MD FROM UST)

MIDTERM 2 EXAM - MARCH 2016

46

Paradoxical embolus is usually seen among patients with: A. Tetralogy of Fallot B. Transposition of great vessels C. Cardiac septal defect D. Tricuspid atresia E. NOTA

47

A 13 y/o F girl with asthma seeks medical attention at her pediatrician’s office complaining of shortness of breath. What would sputum cytology tests reveal? A. Fungal hyphae B. Acid-fast bacilli C. Keratin pearls D. Charcot-Leyden crystals E. Atypical squamous epithelial cells This type of gastritis is characterized by autoantibodies to gastric parietal cells and intrinsic factor. A. Type A B. Type B C. Type AB D. Type C E. Type O

Charcot-Leyden crystals are a sign of eosinophilic degranulation, and Curschmann spirals are a sign of excess mucus in sputum of asthma patients.

GEORGE MICHAEL SOSUAN (TOP 5 - AUG 2015 MED BOARDS; TOPNOTCH MD FROM UST)

MIDTERM 2 EXAM - MARCH 2016

Type A - Autoimmune gastritis (10%), autoantibodies are made against parietal cells and intrinsic factor, gland atrophy leads to pernicious anemia; Type B Helicobacter pylori infection (90%), the most common cause of gastritis, Increased risk of peptic ulcers and carcinoma

GEORGE MICHAEL SOSUAN (TOP 5 - AUG 2015 MED BOARDS; TOPNOTCH MD FROM UST)

MIDTERM 2 EXAM - MARCH 2016

A 32 y/o F has progressive dyspnea, cough, and fever. A chest radiograph shows bilateral infiltrates. She also notes dark brown urine. Which of the following statements is FALSE? A. Low levels of serum complement may be seen B. Blood cultures should be performed C. Serum ANCA should be performed D. The differential diagnosis includes systemic lupus erythematosus E. Polyarteritis nodosa is the most likely diagnosis The leather-bottle appearance of the gastric adenocarcinoma is characteristic of this histologic type. A. Intestinal B. Diffuse C. Nodular D. Esophageal E. Squamous

Polyarteritis nodosa typically does not involve the pulmonary vessels.

GEORGE MICHAEL SOSUAN (TOP 5 - AUG 2015 MED BOARDS; TOPNOTCH MD FROM UST)

MIDTERM 2 EXAM - MARCH 2016

Linitis plastica refers to a thickened “leather-bottle” appearance of the stomach caused by diffuse infiltration of the gastric wall by a poorly differentiated signet-ring type of adenocarcinoma.

GEORGE MICHAEL SOSUAN (TOP 5 - AUG 2015 MED BOARDS; TOPNOTCH MD FROM UST)

MIDTERM 2 EXAM - MARCH 2016

51

An elderly woman complains of fatigue, anemia, and bright red blood in stool. Which of the following is the MOST LIKELY diagnosis? A. Rectal adenocarcinoma B. Anal squamous cell carcinoma C. Endometriosis D. AOTA E. NOTA

10% of colon cancers occur in the rectum. Anal carcinoma is usually human papillomavirus– related and occurs in younger patients; endometriosis causing rectal bleeding would not be expected in a postmenopausal patient.

GEORGE MICHAEL SOSUAN (TOP 5 - AUG 2015 MED BOARDS; TOPNOTCH MD FROM UST)

MIDTERM 2 EXAM - MARCH 2016

52

True of HBsAg: A. First evidence of infection B. Appears in the serum after symptoms C. Co-incide with the elevation of ALT D. AOTA E. Both A and C

HBsAg (surface antigen) provides the first evidence of infection and appears in the serum before symptoms.

GEORGE MICHAEL SOSUAN (TOP 5 - AUG 2015 MED BOARDS; TOPNOTCH MD FROM UST)

MIDTERM 2 EXAM - MARCH 2016

48

49

50

TOPNOTCH MEDICAL BOARD PREP PATHOLOGY SUPEREXAM Page 9 of 99 For inquiries visit www.topnotchboardprep.com.ph or email us at [email protected]

TOPNOTCH MEDICAL BOARD PREP PATHOLOGY SUPEREXAM For inquiries visit www.topnotchboardprep.com.ph or email us at [email protected] Item # 53

QUESTION

EXPLANATION

AUTHOR

TOPNOTCH EXAM MIDTERM 2 EXAM - MARCH 2016

Hallmark of malignant transformation A. Keratin pearls B. Dedifferentiation C. Lack of differentiation D. Metastasis E. Skip lesions

Anaplasia or lack of differentiation is the hallmark of malignant transformation.

GEORGE MICHAEL SOSUAN (TOP 5 - AUG 2015 MED BOARDS; TOPNOTCH MD FROM UST)

54

The leading causative agent of injection drug use hepatitis: A. Hepatitis A B. Hepatitis B C. Hepatitis C D. Hepatitis D E. Both B and C

In hepatitis C, chronic disease occurs in 85% of patients. The leading cause is injection drug use. Cirrhosis will develop in 20% of patients within 20 years.

GEORGE MICHAEL SOSUAN (TOP 5 - AUG 2015 MED BOARDS; TOPNOTCH MD FROM UST)

MIDTERM 2 EXAM - MARCH 2016

55

Hepatocellular carcinoma with the best prognosis A. Hepatitis B variant B. Fibrolamellar variant C. Scirrhous type D. Mucinous type E. NOTA

Fibrolamellar variant of HCC may have a better prognosis than conventional HCC. It usually occurs in young adults without a history of prior liver disease. It is characterized by oncocytic-like hepatocytes (abundant intracellular mitochondria) infiltrating fibrous stroma.

GEORGE MICHAEL SOSUAN (TOP 5 - AUG 2015 MED BOARDS; TOPNOTCH MD FROM UST)

MIDTERM 2 EXAM - MARCH 2016

56

This pathology is characterized by distinctive combination of mesangial and endothelial proliferation, along with thickening and duplication of the capillary basement membrane A. Amyloidosis B. Diabetic nephropathy C. Membranoproliferative glomerulonephritis D. IgA nephropathy E. Post-streptococcal glomerulonephritis

The decription stated is of a "tram track," characteristic of MPGN.

GEORGE MICHAEL SOSUAN (TOP 5 - AUG 2015 MED BOARDS; TOPNOTCH MD FROM UST)

MIDTERM 2 EXAM - MARCH 2016

57

This pathological process is characterized by acute vascular lesions include fibrinoid necrosis of renal arteries and arterioles, with onion skin lesions A. Renal artery stenosis B. Malignant hypertension C. Interstitial nephritis D. Hypertensive nephrosclerosis E. Vasculitis

Malignant hypertension is a rapid disease results in acute vascular lesions and parenchymal injury referred to as “malignant nephrosclerosis.”

GEORGE MICHAEL SOSUAN (TOP 5 - AUG 2015 MED BOARDS; TOPNOTCH MD FROM UST)

MIDTERM 2 EXAM - MARCH 2016

58

Crescentic glomerulonephritis is most associated with: A. p-ANCA B. HIV C. Cytomegalovirus D. Cryoglobulin E. Nephritic factor

GEORGE MICHAEL SOSUAN (TOP 5 - AUG 2015 MED BOARDS; TOPNOTCH MD FROM UST)

MIDTERM 2 EXAM - MARCH 2016

59

Pretibial myxedema is characterized by: A. Hyaluronic acid deposition in the subcutaneous tissue B. Lymphocytic inflitrates in the dermis C. Chondroitin sulfate deposition within the dermis D. Both A and B E. AOTA This is a disorder characterized by peripheral resistance to circulating thyroid hormone A. Refetoff syndrome B. Zellweger syndrome C. Reiter syndrome D. Thyroid dysplasmocytic syndrome E. NOTA

ANCAs (anti-neutrophil cytoplasmic antibodies) are associated with pauci-immune crescentic glomerulonephritis. HIV is associated with focal segmental glomerulosclerosis; cryoglobulin and C3 nephritic factor may be associated with different types of membranoproliferative glomerulonephritis. Cytomegalovirus infection causes tubulointerstitial nephritis and is seen in immunocompromised patients. Pretibial myxedema is secondary to hyaluronic acid deposition and lymphocytic infiltrates within the dermis.

GEORGE MICHAEL SOSUAN (TOP 5 - AUG 2015 MED BOARDS; TOPNOTCH MD FROM UST)

MIDTERM 2 EXAM - MARCH 2016

Refetoff syndrome is a disorder of peripheral resistance to circulating thyroid hormone.

GEORGE MICHAEL SOSUAN (TOP 5 - AUG 2015 MED BOARDS; TOPNOTCH MD FROM UST)

MIDTERM 2 EXAM - MARCH 2016

60

61

What is the interluekin that is vital for neutrophil chemotaxis? A. IL-1 B. IL-2 C. IL-6 D. IL-8 E. IL-10

IL-8 is the major chemoatactic factor for neutrophils. IL-1 is mostly resposnsible for fever. IL-2 stimutales grwoth of T cells. IL-6 stimulates acute phase protein production and IL-10 modulates the immune response

JAN CHRISTIAN FELICIANO (TOP 2 - AUG 2015 MED BOARDS; TOPNOTCH MD FROM UST)

MIDTERM 3 EXAM - MARCH 2016

62

What cytokine mediates septic shock and plays a role in cancer cachexia? A. TNF alpha B. IL-1 C. Inteferon alpha D. Inferterferon gamma E. Lipopolysaccharide

SIMILAR TO PREVIOUS BOARD EXAM CONCEPT/PRINCIPLE/ TNF alpha is believed to be the cytokine that mediates septic shock and is implicated in cachexia of malignancy.

JAN CHRISTIAN FELICIANO (TOP 2 - AUG 2015 MED BOARDS; TOPNOTCH MD FROM UST)

MIDTERM 3 EXAM - MARCH 2016

TOPNOTCH MEDICAL BOARD PREP PATHOLOGY SUPEREXAM Page 10 of 99 For inquiries visit www.topnotchboardprep.com.ph or email us at [email protected]

TOPNOTCH MEDICAL BOARD PREP PATHOLOGY SUPEREXAM For inquiries visit www.topnotchboardprep.com.ph or email us at [email protected] Item # 63

EXPLANATION

AUTHOR

This is considered an irreversible histologic manifestation of cellular injury A. Cellular swelling B. Nuclear chromatin clumping C. Nuclear pyknosis D. Ribosomal detachment E. Membrane blebbing

Nuclear pyknosis, karyorrhesis and karyolysis are considered irreversible changes of cell injury, The rest of the choices are reversible.

JAN CHRISTIAN FELICIANO (TOP 2 - AUG 2015 MED BOARDS; TOPNOTCH MD FROM UST)

Which statement refers to dystrophic calcification rather than metastatic calcification? A. Diffuse and widespread B. Ca deposition in normal tissues C. Patients are usually not normocalcemic D. Occurs in patients on long term hemodialysis E. Associated with thrombus and infarcts Which of the ff mechanisms regarding leukocyte extravasatation is incorrectly paired? A. Margination and rolling- Selectins B. Margination and rolling- GlyCAM-1 C. Tight-binding- ICAM-1 D. Diapededis- VCAM-1 E. Migration- IL-8

Dystrophic calcification is ca deposition in abnormal tissues usually due to necrosis. It is seen in TB, infarcts, thrombus, schistosomiasis, congenital CMV, toxoplasmosis and psamomma bodies. The rest of the choices refers to metastatic calcification.

JAN CHRISTIAN FELICIANO (TOP 2 - AUG 2015 MED BOARDS; TOPNOTCH MD FROM UST)

MIDTERM 3 EXAM - MARCH 2016

Diapedesis uses PECAM-1 not VCAM (utilized in tight binding together with ICAM). All the other choices are correct

JAN CHRISTIAN FELICIANO (TOP 2 - AUG 2015 MED BOARDS; TOPNOTCH MD FROM UST)

MIDTERM 3 EXAM - MARCH 2016

66

Cigarette smoking is carcinogenic to the ff organs EXCEPT? A. Colon B. Bladder C. Pancreas D. Kidney E. Cervix

SIMILAR TO PREVIOUS BOARD EXAM CONCEPT/PRINCIPLE. Cigarrete smoking is assocated with maligancies in the ff organs: bladder, cervix, esophagus, kidne, larynx, lung, pancreas. Colon and breast malignancies usually does not have smoking as a risk factor/

JAN CHRISTIAN FELICIANO (TOP 2 - AUG 2015 MED BOARDS; TOPNOTCH MD FROM UST)

MIDTERM 3 EXAM - MARCH 2016

67

Which of the ff statements regarding neoplasia is INCORRECT? A. Tumor is said to be benign when its gross and microscopic appearances are considered relatively innocent B. Environmental influences not genetic factors appear to be the dominant risk factors for most cancers. C. Hamartomas are disorganized but benign masses composed of cells indigenous to the involved site. D. Benign neoplasms and poorly differentiated carcinomas of endocrine glands frequently secrete hormones characteristic of their origin E. Once the tumor cells breach the basement membrane, the tumor is said to be invasive. Overall, what is the most commonly mutated proto-oncogene seen in approximately 15% to 20% of all human tumors? A. RET B. BCL-2 C. RAS D. p53 E. Rb

Benign neoplasms and well-differentiated carcinomas of endocrine glands frequently secrete hormones character tic of their origin. Well-differentiated squamous cell carcinomas of the epidermis synthesize keratin, and well-differentiated hepatocellular carcinomas elaborate bile. All other statements are correct

JAN CHRISTIAN FELICIANO (TOP 2 - AUG 2015 MED BOARDS; TOPNOTCH MD FROM UST)

MIDTERM 3 EXAM - MARCH 2016

Point mutations of RAS family genes constitute the most common type of abnormality involving protooncogenes in human tumors. Approximately 15% to 20% of all human tumors express mutated RAS proteins, but in some types of cancers the frequency of RAS mutations is much higher. p53 and Rb are tumor suprresor genes not proto-oncogene

JAN CHRISTIAN FELICIANO (TOP 2 - AUG 2015 MED BOARDS; TOPNOTCH MD FROM UST)

MIDTERM 3 EXAM - MARCH 2016

Which of these statements regarding atherosclerosis is correct? A. It literally means “hardening of the arteries" B. Fatt streaks are present in virtually all adolescents even without risk factors C. Age is the most important independent risk factor for atherosclerosis. D. Because the modified lipoproteins cannot be completely degraded, chronic ingestion leads to the formation of lipid-filled macro- phages called fatty streaks E. Fibrinogen is the major structural component of the fibrous cap, and accounts for its mechanical strength and stability Pertaining to the previous question, where is the most common site of atherosclerosis? A. Abdominal aorta B. Coronorary artery C. Internal carotid artery D. Thoracic aorta E. Popliteal artery

True enough, aortas of infants can exhibit fatty streaks, and such lesions are present in virtually all adolescents, even those without known risk factors. Arteriosclerosis literally means “hardening of the arteries”. Family history and not age is the most important independent risk factor for atherosclerosis. lipid-filled macrophages are called foam cells. Collagen is the major structural component of the fibrous cap, and accounts for its mechanical strength and stability.

JAN CHRISTIAN FELICIANO (TOP 2 - AUG 2015 MED BOARDS; TOPNOTCH MD FROM UST)

MIDTERM 3 EXAM - MARCH 2016

In descending order, the most extensively involved vessels are the lower abdominal aorta, the coronary arteries, the popliteal arteries, the internal carotid arteries, and the vessels of the circle of Willis.

JAN CHRISTIAN FELICIANO (TOP 2 - AUG 2015 MED BOARDS; TOPNOTCH MD FROM UST)

MIDTERM 3 EXAM - MARCH 2016

64

65

68

69

70

QUESTION

TOPNOTCH MEDICAL BOARD PREP PATHOLOGY SUPEREXAM Page 11 of 99 For inquiries visit www.topnotchboardprep.com.ph or email us at [email protected]

TOPNOTCH EXAM MIDTERM 3 EXAM - MARCH 2016

TOPNOTCH MEDICAL BOARD PREP PATHOLOGY SUPEREXAM For inquiries visit www.topnotchboardprep.com.ph or email us at [email protected] Item # 71

QUESTION

EXPLANATION

AUTHOR

TOPNOTCH EXAM MIDTERM 3 EXAM - MARCH 2016

A 70 yr old male had severe crushing chest pain and died 3 hours later before he was brought to the ER. Cause of death is myocardial infarction. What do you expect to see on light microscopy on time of death? A. Early coagulation necrosis B. Marginal contraction band necrosis C. Myocyte hypereosinophilia D. Waviness of fibers E. Pyknosis of nuclei

SIMILAR TO PREVIOUS BOARD EXAM CONCEPT/PRINCIPLE. The gross and microscopic appearance of an infarct depends on the duration of survival of the patient following the MI. Light microcopy findings: Less than 30 mins- None; 30 mins-4 hours- Usually none but variable waviness of fibers at border; 4-12 hours- Early coag necrosis; 1224 hrs- pyknosis; myocyte hypereosniphilia and marginal contraction band necrosis

JAN CHRISTIAN FELICIANO (TOP 2 - AUG 2015 MED BOARDS; TOPNOTCH MD FROM UST)

72

A 50 yr old patient had routine checkup and the only finding is a midsystolic click on auscultation. You are suspecting a valvular defect. Which of the following pathologic changes is most likely present in the valve? A. Destructive vegetations B Dystrophic calcification C Fibrinoid necrosis D Myxomatous degeneration E Rheumatic fibrosis

The case points to possible mitral valve prolapse wherein the underlying pathology is myxomatous degeneration. The chordae tendineae become elongated and can rupture to produce acute valvular incompetence.

JAN CHRISTIAN FELICIANO (TOP 2 - AUG 2015 MED BOARDS; TOPNOTCH MD FROM UST)

MIDTERM 3 EXAM - MARCH 2016

73

A patient came to you complaining of chronic back pain. Radiagraphic exams shows multple punched out lytic bone lesions. Which lab abnomrality is most likely? A. Serum IgM M protein B. Hypocalemia C. Increased serum alkaline phpsphatase D. Marked splenomegaly E. Polyclonal urinary light chains This pneumoconioses is seen as radiographically as an eggshell calcification of hilar lymph nodes described as stained glass in appearance? A. Asbestosis B. Berylloisis C. Coal worker's pneumoconioses D. Silicosis E. Hypersenstivity pneumonitis This statement is correct regarding esophageal cancer. A. Adenocarcinoma is more common worldwide B. Half of squamous cell carcinomas occur in the lower third of the esophagus C. Reduced rates of Helicobacter pylori infection may be a factor in the increasing incidence of esophageal adenocarcinoma D. Squamous cell CA most commonly produce mucin and form glands often with intestinal type morphology E. Esophageal adenocarcinoma occurs most frequently in Caucasians and is equally distributed among genders On endoscopy of a 14 year old boy complaining of bleeding per rectum, a colonic mass was seen. Histologic examination demonstrates a characteristic arborizing network of connective tissue, smooth muscle, lamina propria, and glands lined by normal-appearing intestinal epithelium. What is the diagnosis? A. Familial adenomatous polyposis B. Peutz-Jeghers syndrome C. Juvenile polyposis D. Lynch syndrome E. Colon Adenocarcinoma What is the most important characteristic of adenomatous polyps that best correlates with risk of malignancy? A. Morphology (tubular or villous) B. Degree of differentiation C. Size D. Presence of stalk (pedunculated or sessile) E. APC proto-oncogene mutation

Answer is A. Typo error. Choice should have been Serum IgA. This is highly suggestive of multitple myeloma. IgG or IgA M proteins are almaot always found in multiple myeloma. Additionaly lab criteria is is hypercalcemia, monocloonal urinary light chains and rolouex formation.

JAN CHRISTIAN FELICIANO (TOP 2 - AUG 2015 MED BOARDS; TOPNOTCH MD FROM UST)

MIDTERM 3 EXAM - MARCH 2016

Eggshell calcification of hilar lymph nodes is suggestive of silicosis.

JAN CHRISTIAN FELICIANO (TOP 2 - AUG 2015 MED BOARDS; TOPNOTCH MD FROM UST)

MIDTERM 3 EXAM - MARCH 2016

Some serotypes of Helicobacter pylori are associated with decreased risk of esophageal adenocarcinoma, because they cause gastric atrophy, which in turn leads to reduced acid secretion and reflux, and reduced incidence of Barrett esophagus. Squamous CA is most common and occurs at middle third of esophagus. AdenoCA produces mucin and is often the intestinal type morphology. It is 7 times more common in males than females.

JAN CHRISTIAN FELICIANO (TOP 2 - AUG 2015 MED BOARDS; TOPNOTCH MD FROM UST)

MIDTERM 3 EXAM - MARCH 2016

Grossly, the polyps are large and pedunculated with a lobulated contour. Histologic examination demonstrates a characteristic arborizing network of connective tissue, smooth muscle, lamina propria, and glands lined by normal-appearing intestinal epithelium. The arborization and presence of smooth muscle intermixed with lamina propria are helpful in distinguishing polyps of Peutz-Jeghers syndrome from juvenile polyps.

JAN CHRISTIAN FELICIANO (TOP 2 - AUG 2015 MED BOARDS; TOPNOTCH MD FROM UST)

MIDTERM 3 EXAM - MARCH 2016

Size is the most important characteristic that correlates with risk of malignancy. Although villous adenomas contain foci of invasion more frequently than tubular adenomas, villous architecture alone does not increase cancer risk when polyp size is considered.

JAN CHRISTIAN FELICIANO (TOP 2 - AUG 2015 MED BOARDS; TOPNOTCH MD FROM UST)

MIDTERM 3 EXAM - MARCH 2016

A patient is suspected of having alcholic liver disease possible alcohol hepatitis. You expect to see what finding in liver biopsy? A. Clumped, amorphous, eosinophilic material in ballooned hepatocytes made up of tangled skeins of intermediate filaments B. Plasma cell predominance in the mononuclear inflammatory infiltrates C. Ground-glass hepatocytes D. Extensive feathery degeneration of periportal hepatocytes E. Circumferential “onion skin” fibrosis around

SIMILAR TO PREVIOUS BOARD EXAM CONCEPT/PRINCIPLE. Do not rely on buzzwords. Understand the morhpology. Choice a refers to Mallory bodies and is characertistic of Alcoholic hepatitis. Choice B refrs to autoimmune hepatitis. Choice C refers to chronic HepB infection. Choice D refers to cholestatis. Choice E refers to Primary sclerosing cholangitis

JAN CHRISTIAN FELICIANO (TOP 2 - AUG 2015 MED BOARDS; TOPNOTCH MD FROM UST)

MIDTERM 3 EXAM - MARCH 2016

74

75

76

77

78

TOPNOTCH MEDICAL BOARD PREP PATHOLOGY SUPEREXAM Page 12 of 99 For inquiries visit www.topnotchboardprep.com.ph or email us at [email protected]

TOPNOTCH MEDICAL BOARD PREP PATHOLOGY SUPEREXAM For inquiries visit www.topnotchboardprep.com.ph or email us at [email protected] Item #

QUESTION

EXPLANATION

AUTHOR

TOPNOTCH EXAM

an increasingly atrophic duct lumen

79

A child was brought to your clinic complaining of hematuria. He had sore throat 5 days prior to consult. What do you expect to find on electron microscopy? A. discrete, amorphous, electron-dense deposits on the epithelial side of the membrane, often having the appearance of “humps B. uniform and diffuse effacement of foot processes C. double contour” or “tram-track” appearance D. presence of electron-dense deposits predominantly in the mesangium E. proliferation of parietal cells forming crescents What is the microscopic finding in diabetic glomerulonephropathy? A. Mesangial expansion B. GBM thickening C. Eosinophilic nodular glomerulosclerosiss D. Renal atherosclerosis E. All of the above

SIMILAR TO PREVIOUS BOARD EXAM CONCEPT/PRINCIPLE.The case most likely points to IgA nephropathy. PSGN usually appears 1 to 4 weeks after a streptococcal infection of the pharynx or skin (impetigo). Choice A refers to PSGN, Choice B is miniman change disease. Choice C refers to MPGN Type 1. Choice E is RPGN

JAN CHRISTIAN FELICIANO (TOP 2 - AUG 2015 MED BOARDS; TOPNOTCH MD FROM UST)

MIDTERM 3 EXAM - MARCH 2016

SIMILAR TO PREVIOUS BOARD EXAM CONCEPT/PRINCIPLE. The most important glomerular lesions are capillary basement membrane thickening, diffuse mesangial sclerosis, and nodular glomerulosclerosis aka Kimmelstiel-Wilson disease. Renal atherosclerosis and arteriolosclerosis constitute part of the macrovascular disease in diabetics.

JAN CHRISTIAN FELICIANO (TOP 2 - AUG 2015 MED BOARDS; TOPNOTCH MD FROM UST)

MIDTERM 3 EXAM - MARCH 2016

81

1. A 58 year old male presented with left sided weakness associated with headache and vomiting 4 hours prior to admission. Patient is a known hypertensive and currently takes Losartan. What is the most probable pattern of tissue necrosis seen in this patient? a. gangrenous necrosis b. coagulative necrosis c. liquefactive necrosis d. fat necrosis e. none of the above

ANDREW TIU (TOP 1 - AUG 2015 MED BOARDS; TOPNOTCH MD FROM CIM)

FINAL EXAM - MARCH 2016

82

2. A 20 year old male was bitten by a red ant on the dorsum of his foot. After 30 minutes, he noticed his foot was swollen, erythematous, and itchy. What is the most likely mechanism for the symptoms? a. increased transcytosis b. endothelial injury c. retraction of endothelial cells d. leukocyte mediated vascular injury e. none of the above 3. After 1 day, patient noted increased pruritus, swelling, and erythema over the dorsum of his foot after repeatedly scratching it. What is now the most likely mechanism for the symptoms? a. increased transcytosis b. endothelial injury c. retraction of endothelial cells d. leukocyte mediated vascular injury e. none of the above 4. A 28 year old female, CRB, presents 1 month history of easy fatigability, knee pain, mouth sores, and tea colored urine. Past medical history and family history was unremarkable. On workup, the physician noted a positive ANA and anti - dsDNA. Which of the following patterns of nuclear fluorescence most likely suggests the type of antibody present in the patient? a. homogeneous b. rim c. speckled d. nucleolar e. none of the above

liquefactive necrosis is characterized by digestion of dead cells resulting in transmoration of tissue into a liquid viscous mass. for unknown reasons, hypoxic death of cells within CNS often manifests as such. It is also seen in bacterial and fungal infections. Coagulative necrosis - architecture of dead tissues is preserved for a span of at least some days. Gangrenous necrosis - usually applied to a limb that has lost its blood supply and has undergone typically coagulative necrosis involving multiple tissue planes. Fat necrosis refers to focal areas of fat destruction from release of activated pancreatic lipases into pancreas and peritoneal cavity. (robbins 8th edition p.16) contraction of endothelial cells resulting in increased endothelial spaces is the most common mechanism of vascular leakage and is elicited by histamine, bradykinin, leukotrienes, substance P, etc. It is called the immediate transient response because it occurs rapidly after exposure to the mediator. (Robbins 8th edition p. 47)

ANDREW TIU (TOP 1 - AUG 2015 MED BOARDS; TOPNOTCH MD FROM CIM)

FINAL EXAM - MARCH 2016

contraction of endothelial cells resulting in increased endothelial spaces is the most common mechanism of vascular leakage and is elicited by histamine, bradykinin, leukotrienes, substance P, etc. It is called the immediate transient response because it occurs rapidly after exposure to the mediator. (Robbins 8th edition p. 47)

ANDREW TIU (TOP 1 - AUG 2015 MED BOARDS; TOPNOTCH MD FROM CIM)

FINAL EXAM - MARCH 2016

rim or peripheral staining pattern are most indicative of antibodies to dsDNA. Homogeneous or diffuse usually reflects antibodies to chromatin, histones, and occasionally dsDNA. Speckled pattern is least specific and is the most commonly observed. This includes Sm antigen, RNP, SS-A, SS-B. (RObbins 8th edition p. 214)

ANDREW TIU (TOP 1 - AUG 2015 MED BOARDS; TOPNOTCH MD FROM CIM)

FINAL EXAM - MARCH 2016

80

83

84

TOPNOTCH MEDICAL BOARD PREP PATHOLOGY SUPEREXAM Page 13 of 99 For inquiries visit www.topnotchboardprep.com.ph or email us at [email protected]

TOPNOTCH MEDICAL BOARD PREP PATHOLOGY SUPEREXAM For inquiries visit www.topnotchboardprep.com.ph or email us at [email protected] Item # 85

86

87

88

89

90

91

92

QUESTION 5. Patient CRB was then started on steroids. Creatinine was noted to be slightly elevated on follow up. Which of the following is the hallmark of her disease? a. malar rash b. glomerulonephritis c. production of autoantibodies d. normocytic normochromic anemia e. none of the above 6. A 2 year old female presented with 12 week history of recurrent, fever, cough, 15% weight loss, rash, and generalized lymphadenopathy. Patient was started with antibiotics without relief of symptoms. Biopsy of the lymph node was done which showed follicles depleted of cells and the organized network of follicular dendritic cells is disrupted. What is the most likely diagnosis? a. hodgkin's lymphoma b. AIDS c. ALL d. Miliary TB e. none of the above 7. A 56 year old farmer comes to you with complaints of a painful chronic skin ulcer. Biopsy was done which revealed squamous cell carcinoma. On further workup, patient also has pulmonary nodules on the right middle lobe and left upper lobe. Which of the following agents is most likely responsible? a. benzene b. beryllium c. chromium d. arsenic e. none of the above 8. JCT, 5 year old child presents with fever, neck mass, and decreased level of consciousness. Past medical history includes pneumonia at 8 months old and German measles at 2 years old. Biopsy of neck mass showed a edematous interstitium diffusely infiltrated with macrophages, lymphocytes, and plasma cells. Which of the following is the most likely etiologic agent? a. SSPE b. mumps c. polio d. EBV e. none of the above 9. A 72 year old male chronic smoker presented with abdominal pain and bone pains at night. Serum calcium was noted to be elevated. Which of the following is the organ specific carcinogen in tobacco smoke? a. tar b. formaldehyde c. polycyclic aromatic hydrocarbons d. nicotine e. none of the above 10. A previously healthy neonate presented with tachypnea, retractions, and cyanosis. CRP was elevated. What is the most likely diagnosis? a. RDS type I b. RDS type II c. ARDS d. BPD e. none of the above 11. Which of the following refers to a plaque witha superficial fibrous cap composed of smooth muscle cells and relaively dense collagen with a lipid core? a. atherosclerotic plaque b. fatty streak c. all of the above d. none of the above e. none of the above 12. A 58 year old retired Chinese teacher came in for multiple injuries secondary to fall in the bathroom. X ray was done and showed a femoral fracture. ORIF was done and 48 hours post -op, patient was noted to be dyspneic, tachycardic with O2 sats 68%. Despite resuscitative efforts, patient died. What is the most likely autopsy findings of the heart? a. thickened right ventricular wall b. dilated right ventricular wall

EXPLANATION

AUTHOR

TOPNOTCH EXAM FINAL EXAM - MARCH 2016

robbins 8th edition p. 213

ANDREW TIU (TOP 1 - AUG 2015 MED BOARDS; TOPNOTCH MD FROM CIM)

robbins 8th edition p. 249

ANDREW TIU (TOP 1 - AUG 2015 MED BOARDS; TOPNOTCH MD FROM CIM)

FINAL EXAM - MARCH 2016

benzene - leukemia, hodgkins lymphoma (principal component of light oil); beryllium - lung cancer (missile fuel and space vehicles); chromium - lung cancer (metal alloys, paints, pigments, preservatives) Robbins 8th edition p. 274

ANDREW TIU (TOP 1 - AUG 2015 MED BOARDS; TOPNOTCH MD FROM CIM)

FINAL EXAM - MARCH 2016

Mumps encephalitis causes perivenous demyelination and perivascular mononuclear cuffing. Aseptic meningitis is the most common extrasalivary gland complication of mumps infection occuring in 10% of cases. Robbins 8th edition p. 250

ANDREW TIU (TOP 1 - AUG 2015 MED BOARDS; TOPNOTCH MD FROM CIM)

FINAL EXAM - MARCH 2016

Components of cigarette smoke particularly polycyclic hydrocarbons and nitrosamines are potent carcinogens in animals and likely to be directly involved in the development of lung cancer in humans. Robbins 8th edition p. 411-412

ANDREW TIU (TOP 1 - AUG 2015 MED BOARDS; TOPNOTCH MD FROM CIM)

FINAL EXAM - MARCH 2016

Robbins 8th edition p.456

ANDREW TIU (TOP 1 - AUG 2015 MED BOARDS; TOPNOTCH MD FROM CIM)

FINAL EXAM - MARCH 2016

Atherosclerotic plaques have 3 principal components: 1) cells 2) ECM 3)intracellular and extracellular lipid. Robbins 8th edition p. 502

ANDREW TIU (TOP 1 - AUG 2015 MED BOARDS; TOPNOTCH MD FROM CIM)

FINAL EXAM - MARCH 2016

In acute cor pulmonale, there is marked dilation of right ventricle without hypertrophy. The rest of the choices refer to chronic cor pulmonale. Robbins 8th edition p. 560

ANDREW TIU (TOP 1 - AUG 2015 MED BOARDS; TOPNOTCH MD FROM CIM)

FINAL EXAM - MARCH 2016

TOPNOTCH MEDICAL BOARD PREP PATHOLOGY SUPEREXAM Page 14 of 99 For inquiries visit www.topnotchboardprep.com.ph or email us at [email protected]

TOPNOTCH MEDICAL BOARD PREP PATHOLOGY SUPEREXAM For inquiries visit www.topnotchboardprep.com.ph or email us at [email protected] Item #

QUESTION

EXPLANATION

AUTHOR

TOPNOTCH EXAM

B - RHD, C - NBTE, D - Libmann sacks endocarditis robbins 8th edition p. 567

ANDREW TIU (TOP 1 - AUG 2015 MED BOARDS; TOPNOTCH MD FROM CIM)

FINAL EXAM - MARCH 2016

Nodes involved in acute lymphadenitis are enlarged and painful. When pyogenic organisms are the cause, the centers of the follicles may undergo necrosis. Choices A, B, and D refer to chronic nonspecific lymphadenitis. (robbins 8th edition p.595)

ANDREW TIU (TOP 1 - AUG 2015 MED BOARDS; TOPNOTCH MD FROM CIM)

FINAL EXAM - MARCH 2016

It is characterized by vascular engorgement, intraalveolar fluid with few neutrophils and often the presence of numerous bacteria. In red hepatization, the lobe now appears red, firm, and airless with a liver - like consistency. (robbins 8th edition p. 713)

ANDREW TIU (TOP 1 - AUG 2015 MED BOARDS; TOPNOTCH MD FROM CIM)

FINAL EXAM - MARCH 2016

A - goodpasture's syndrome; B - MPGN type 1; D - MPGN type 2; E - IgA nephropathy Robbins 8th edition p.921

ANDREW TIU (TOP 1 - AUG 2015 MED BOARDS; TOPNOTCH MD FROM CIM)

FINAL EXAM - MARCH 2016

B - uniform diffuse thickening of the glomerular capillary wall C - glomeruli show mesangial widening and endocapillary proliferation D - crescents which are proliferation of parietal cells and migration of monocytes and macrophages into the urinary space E - collapse of capillary loops, increase in matrix, and segmental deposition of plasma proteins aong capillary wall (hyalinosis) Robbins 8th edition p. 929 Robbins 8th edition p. 945

ANDREW TIU (TOP 1 - AUG 2015 MED BOARDS; TOPNOTCH MD FROM CIM)

FINAL EXAM - MARCH 2016

ANDREW TIU (TOP 1 - AUG 2015 MED BOARDS; TOPNOTCH MD FROM CIM)

FINAL EXAM - MARCH 2016

c. disappareance of fat in the ventricular wall d. regurgitation and fibrous thickening of tricuspid valve e. none of the above

93

94

95

96

97

98

13. An 8 year old male came for complaints of exertional dyspnea, recurrent high grade fever, subcutaneous nodules in the pulps of the digits, and tea colored urine. A year ago, patient was noted to have recurrent sore throat however no consult was done and no medications were taken. Which of the following is the hallmark seen in this case? a. friable bulky destructive vegetations b. small warty vegetations along the lines of closure c. small bland vegetations along the lines of closure d. medium sized vegetations on either sides of the valve leaflets e. none of the above 14. One week prior to consult, an 18 year old nursing student came in for complaints of impacted wisdom tooth on the right side of the mandible. After 24 hours, a painful neck mass was noted. Which of the following morphology of the neck mass may undergo necrosis and pus formation? a. follicular hyperplasia b. paracortical hyperplasia c. acute lymphadenitis d. reticular hyperplasia e. none of the above 15. What stage of inflammatory response of the lung is described when the lung is heavy, boggy, and red? a. congestion b. red hepatization c. gray hepatization d. resolution e. none of the above 16. A 13 year old female presented with headache and decreased urine output after a 4 week history of skin infection. Which of the following is the most likely morphology of the kidney? a. linear GBM fluorescence for Ig and complement b. discrete subendothelial electron dense deposits c. granular deposits of IgG, IgM, and C3 in the mesangium and along the GBM d. GBM transformed into an irregular ribbon like, electron dense structure e. mesangial deposition of IgA 17. An 17 year old female presented with elevated blood pressure, tea colored urine, and pallor. Kidney biopsy was done which revealed glomeruli being large, hypercellular, and having an accentuated "lobular appearance". Which of the following is the most likely diagnosis? a. membranoproliferative glomerulonephritis b. membranous nephropathy c. IgA nephropathy d. rapid crescentic glomerulonephritis e. focal segmental glomerulosclerosis 18. A 63 year old female, ECT, came in for complaints of bilateral knee pain. ECT has been self medicating Ibuprofen for 5 years. For the past week, ECT has been having recurrent headaches, vague abdominal pain, and easy fatigability. Which of the following occurs first in renal damage in analgesic nephropathy? a. cortical tubulointerstitial nephritis b. papillary necrosis c. ischemic kidney injury d. toxic kidney injury e. none of the above

TOPNOTCH MEDICAL BOARD PREP PATHOLOGY SUPEREXAM Page 15 of 99 For inquiries visit www.topnotchboardprep.com.ph or email us at [email protected]

TOPNOTCH MEDICAL BOARD PREP PATHOLOGY SUPEREXAM For inquiries visit www.topnotchboardprep.com.ph or email us at [email protected] Item # 99

QUESTION

EXPLANATION

AUTHOR

TOPNOTCH EXAM FINAL EXAM - MARCH 2016

19. Which of the following nipple discharges would most likely signify malignancy? a. bloody b. serous c. milky d. both a and b e. none of the above

robbins 8th edition p. 1068

ANDREW TIU (TOP 1 - AUG 2015 MED BOARDS; TOPNOTCH MD FROM CIM)

20. A 14 year old female presented with a breast mass. Biopsy was done which showed dense collagenous connective tissue and marked micropapillary epithelial hyperplasia of the duct lining. Which of the following is the most likely diagnosis? a. fibroadenoma b. fibrocystic disease c. gynecomastia d. phylloides tumor e. Papilloma Characteristic cytologic feature of medullary thyroid cancer: A. Orphan annie B. Vacuolated nucleus C. Amyloid stroma D. Ground glass E. Pleomorphic giant cells

Yes during our exam, it was female and not male but the description was really referring to gynecomastia. A - delicate cellular, and often myxoid stroma which resembles normal intralobular stroma. B - nonproliferative changes which includes cysts, fibrosis, and adenosis. D - bulbous protrusions due to presence of nodules of proliferating stroma covered by epithelium. E - multiple branching fibrovascular cores each having a connective tissue axis lined by luminal and myoepithelial cells Robbins 8th edition p. 1093 Page 73 of Topnotch Handout. Medullary thyroid carcinoma has polygonal to spindle shaped cells, acellular amyloid deposits, and C-cell hyperplasia.

ANDREW TIU (TOP 1 - AUG 2015 MED BOARDS; TOPNOTCH MD FROM CIM)

FINAL EXAM - MARCH 2016

ANGELA PAULINE P. CALIMAG-LOYOLA (TOP 8 - FEB 2015 MED BOARDS; TOPNOTCH MD FROM UST)

DIAGNOSTIC EXAM - AUG 2015

102

Which of the following is a calcitonin secreting tumor of the thyroid gland? A. Papillary CA B. Follicular CA C. Medullary CA D. Hurthle cell CA E. Anaplastic CA

Page 73 of Topnotch Handout. Medully thyroid carcinoma is a neuroendocrine neoplasma derived from the parafollicular cells or c-cells which secrete calcitonin and is important in diagnosis.

ANGELA PAULINE P. CALIMAG-LOYOLA (TOP 8 - FEB 2015 MED BOARDS; TOPNOTCH MD FROM UST)

DIAGNOSTIC EXAM - AUG 2015

103

23y/o female consults because of bipedal Page 125 of Topnotch Handout. The most common ANGELA PAULINE P. edema & passing out of tea colored urine. On and severe type of Lupus nephritis is Type IV: Diffuse CALIMAG-LOYOLA PE, her BP is 160/100. she has +2 bipedal proliferative GN which has a wire-loop capillary (TOP 8 - FEB 2015 edema. The urinalysis showed +4 protein, 50appearance. MED BOARDS; 60RBC/hpf, 0-1 WBC/hpf and occasional red TOPNOTCH MD cell casts. Serum creatinine is elevated 2mg/dL FROM UST) from a previously normal level of 0.9 mg/dL 1 week ago. If this is a case of SLE, which of the following is the most common and severe histopathologic type? A. Mesangial GN B. Focal proliferative GN C. Membranous GN D. Difffuse proliferative GN E. Cresenteric GN A 40 y/o female was found to have a diastolic Page 112 of hand out. Mitral stenosis causes an ANGELA PAULINE P. rumble at the apex. The chest x-ray showed increase in LA size due to restriction of blood flow CALIMAG-LOYOLA uplifting of the left main stem bronchus, from the left atrium to the left ventricle as a result of a (TOP 8 - FEB 2015 retrosternal fullness, and dilated main narrowed mitral passage. Murmurs associated with MED BOARDS; pulmonary artery. What is the most likely MS are opening snap, accentuated S1, and diastolic TOPNOTCH MD diagnosis? A. rumble. Plain film chest xray findings are FROM UST) Mitral Stenosis cardiomegaly, double right heart border (enlarged left B. Mitral Regurgitation atrium and normal right atrium), prominent left atrial C. Aortic Stenosis appendage and splaying of the subcarinal angle (>120 D. Pulmonic Stenosis degrees). E. Aortic Regurgitation What is the expected thyroid function test in Page 71 of Topnotch Handout. Primary ANGELA PAULINE P. primary hyperthyroidism? hyperthyroidism is the term used when the pathology CALIMAG-LOYOLA A. decreased TSH, increased T4 is within the thyroid gland. Secondary (TOP 8 - FEB 2015 B. decreased TSH, decreased T4 hyperthyroidism is the term used when the thyroid MED BOARDS; C. increased TSH, increased T4 gland is stimulated by excessive thyroid-stimulating TOPNOTCH MD D. increased TSH, decreased T4 hormone (TSH) in the circulation.The diagnosis of FROM UST) E. None of the above hyperthyroidism is confirmed by blood tests that show a decreased thyroid-stimulating hormone (TSH) level and elevated T4 and T3 levels. A low TSH level typically indicates that the pituitary gland is being inhibited or "instructed" by the brain to cut back on stimulating the thyroid gland, having sensed increased levels of T4 and/or T3 in the blood. What can generally differentiate between Page 115of Topnotch Handout. Asthma and COPD are ANGELA PAULINE P. asthma and COPD in spirometry? both obstructive lung diseases. On spirometry FEV1, CALIMAG-LOYOLA a. FEV1 FVC and FEV1/FVC are decreased in both how ever (TOP 8 - FEB 2015 b. reversibility asthma is a reversible condition. MED BOARDS; c. PEF TOPNOTCH MD d. TLC FROM UST) E. FVC

DIAGNOSTIC EXAM - AUG 2015

100

101

104

105

106

TOPNOTCH MEDICAL BOARD PREP PATHOLOGY SUPEREXAM Page 16 of 99 For inquiries visit www.topnotchboardprep.com.ph or email us at [email protected]

DIAGNOSTIC EXAM - AUG 2015

DIAGNOSTIC EXAM - AUG 2015

DIAGNOSTIC EXAM - AUG 2015

TOPNOTCH MEDICAL BOARD PREP PATHOLOGY SUPEREXAM For inquiries visit www.topnotchboardprep.com.ph or email us at [email protected] Item # 107

108

109

110

111

112

113

114

QUESTION

EXPLANATION

AUTHOR

Ranson’s criteria is used in acute panceatitis Page 101 of Topnotch Handout. Ranson's criteria is a ANGELA PAULINE P. for: criteria for prognostication on admission and for the CALIMAG-LOYOLA A. Diagnosis first 48 hours. (TOP 8 - FEB 2015 B. Monitor abscess formation MED BOARDS; C. Predict possible etiology TOPNOTCH MD D. Decide surgical intervention FROM UST) E. Prognostication

TOPNOTCH EXAM DIAGNOSTIC EXAM - AUG 2015

Which collagen type is commonly found in the Page 8 of Topnotch Handout. Epidermolysis bullosa is ANGELA PAULINE P. DIAGNOSTIC dermoepidermal junction and is usually caused by genetic defects (or mutations) within the CALIMAG-LOYOLA EXAM - AUG defective in Epidermolysis bullosa: A. human COL7A1 gene encoding the protein type VII (TOP 8 - FEB 2015 2015 Type I collagen (collagen VII). MED BOARDS; B. Type 3 TOPNOTCH MD C. Type 5 FROM UST) D. Type 7 E. Type 9 In a patient with pulmonary embolism the Page 11 of Topnotch Handout. The most common ECG ANGELA PAULINE P. DIAGNOSTIC most common ECG manifestation that should manifestation is a sinus tachycardia. Right ventricular CALIMAG-LOYOLA EXAM - AUG be expected is? A. strain pattern or S1Q3T3 is found in only 6% of (TOP 8 - FEB 2015 2015 Incomplete right bundle branch block patients. MED BOARDS; B. Sinus tachycardia TOPNOTCH MD C. Right ventricular strain pattern FROM UST) D. S1Q3T3 E. Junctional rhythm A patient diagnosed with drug-induced lupus Page 20 of Topnotch Handout. A patient with drug ANGELA PAULINE P. DIAGNOSTIC would most likely be positive for which induced lupus will be positive to anti-histone. ACALIMAG-LOYOLA EXAM - AUG autoantibody? A. Sjorgren syndrome, B-CREST syndrome, D-SLE, E(TOP 8 - FEB 2015 2015 Anti-SS-A Primary biliary cirrhosis MED BOARDS; B. Anti-centromere TOPNOTCH MD C. Anti-histone FROM UST) D. Anti-dsDNA E. Anti-mitochondrial A 59 y/o male, received a blood group identical Page 23 of Topnotch Handout. Hyperacute rejection ANGELA PAULINE P. DIAGNOSTIC living unrelated kidney graft. During surgery no occurs within minutes to hours due to preformed CALIMAG-LOYOLA EXAM - AUG abnormalities occurred. Four hours after the antibodies. It is a Type II hypersensitibity reaction. (TOP 8 - FEB 2015 2015 transplantation, it was noted that diuresis Morphological features: thrombotic occlusion of MED BOARDS; suddenly decreased. Upon repeat laparotomy capillaries and fibrinoid necrosis occurs in arterial TOPNOTCH MD the transplanted kidney showed signs of walls. FROM UST) hyperacute rejection and had to be removed. Which pathological examination findings are consistent with hyperacute rejection? A. Thrombosis of capillaries and fibrinoid necrosis in arterial walls B. Presence of interstitial mononuclear cell infiltration and edema C. Necrotizing vasculitis withendothelial cell necrosis D. Interstitial fibrosis and tubular atrophy with loss of renal parenchyma E. Neutrophilic infiltration, deposition of Ig, complement and fibrin A 32-day-old female infant was admitted due to Page 25 of Topnotch Handout. This is a case of ANGELA PAULINE P. DIAGNOSTIC delayed umbilical cord detachment and Leukocyte adhesion deficiency syndrome it is an CALIMAG-LOYOLA EXAM - AUG omphalitis. After admission, CBC revealed autosomal recessive disease involving a mutation in (TOP 8 - FEB 2015 2015 severe leukocytosis, and there was poor clinical integrins hence defective adhesion LFA-1 proteins on MED BOARDS; response to several kinds of antibiotics. She is the surface of phagocytes. It is commonly manifested TOPNOTCH MD diagnosed to have aan immunodefiency disease bu severe pyogenic infections in infancy and delayed FROM UST) specifically a phagocyte disorder. What is the cord separation. most probable pathophysiologic mechanism of this disease? A. Mutation in WASP gene for actin filament assembly B. Mutation in DNA repair enzymes C. Lack of NADPH oxidase activity D. Failure of phagolysosomal fusion E. Defective LFA-1 proteins Which electrolyte abnormality is not usually Page 29 of Topnotch Handout. Tumor lysis syndrome ANGELA PAULINE P. DIAGNOSTIC found in tumor lysis syndrome? is an oncologic emergency that is caused by massive CALIMAG-LOYOLA EXAM - AUG A. Hypocalcemia tumor cell lysis with the release of large amounts of (TOP 8 - FEB 2015 2015 B. Hypercalcemia potassium, phosphate, and nucleic acids into the MED BOARDS; C. Hyperphosphatemia systemic circulation. TOPNOTCH MD D. Hyperuricemia FROM UST) E. Hyperkalemia Early chronic myeloid leukemia (CML) and Page 30 of Topnotch Handout. CML must be leukemoid reaction (LR) sometimes show differentiated from leukemoid reactions in which similar histological pictures. To differentiate there is a marked increase in myeloid elements between a leukemoid reaction and CML the secondary to infection, chronic inflammation and following should be requested: other causes. Both present with extreme leukocytosis A. Neutrophil alkaline phosphatase however CML has low NAP and CRP. Cytogenetic B. C Reactive Protein testing will determine the presence of the BCR-ABL C. Cytogenetic testing gene. D. A and B only E. All of the above

ANGELA PAULINE P. CALIMAG-LOYOLA (TOP 8 - FEB 2015 MED BOARDS; TOPNOTCH MD FROM UST)

TOPNOTCH MEDICAL BOARD PREP PATHOLOGY SUPEREXAM Page 17 of 99 For inquiries visit www.topnotchboardprep.com.ph or email us at [email protected]

DIAGNOSTIC EXAM - AUG 2015

TOPNOTCH MEDICAL BOARD PREP PATHOLOGY SUPEREXAM For inquiries visit www.topnotchboardprep.com.ph or email us at [email protected] Item # 115

QUESTION

EXPLANATION

AUTHOR

TOPNOTCH EXAM DIAGNOSTIC EXAM - AUG 2015

This type of Hodgkin's lymphoma which has an Page 32 of Topnotch Handout. Refer to the table on intermediate prognosis, has a highly significant types of hodgkin's lymphoma. Mixed cellularity association with EBV infection and numerous involved lymph nodes are diffusely effaced by a R-S cells in a mixed inflammatory background heterogenous cellular infiltrate. Plentiful RS cells that obliterates the normal architecture: admixed with lymphocytes. It has an intermediate A. Lymphocyte depleted prognosis and EBV-infected in 70% of cases. B. Lymphocyte predominant C. Lymphocyte rich D. Mixed cellularity E. Nodular sclerosis

ANGELA PAULINE P. CALIMAG-LOYOLA (TOP 8 - FEB 2015 MED BOARDS; TOPNOTCH MD FROM UST)

Cold agglutinin disease is a form of Page 38 of Topnotch Handout. IgM antibodies autoimmune hemolytic anemia caused by coldgenerally cause cold agglutinin disease. M-Malamig! reacting autoantibodies. Autoantibodies bind to the erythrocyte membrane leading to premature erythrocyte destruction. Which antibody is commonly involved? A. IgM B. IgG C. IgA D. IgD E. IgE Incomplete excision of a dentigerous cyst may Page 42 of Topnotch Handout. Dentigerous cyst result in a neoplastic transformation, originates around the crown of an unerupted tooth, specifically into a/an: A. often associated with an impacted third molar. Odontoma Complete excision is curative, however incomplete B. Ameloblastoma excision may result to recurrence or neoplastic C. Basal cell carcinoma transformation into an ameloblastoma or a squamous D. Cholesteatoma cell carcinoma. E. Paraganglioma A primary melanoma located in which part of SIMILAR TO PREVIOUS BOARD EXAM the body has the worst prognosis? CONCEPT/PRINCIPLE. Page 44 of Topnotch Handout. A. Sole Anatomic location of the primary melanoma is an B. Palm important independent predictor of SLN status and C. Scalp prognosis. Patients with primary melanomas of the D. Chest head/neck and trunk have a worse prognosis than E. Back primary melanomas of other anatomic locations.

ANGELA PAULINE P. CALIMAG-LOYOLA (TOP 8 - FEB 2015 MED BOARDS; TOPNOTCH MD FROM UST)

DIAGNOSTIC EXAM - AUG 2015

ANGELA PAULINE P. CALIMAG-LOYOLA (TOP 8 - FEB 2015 MED BOARDS; TOPNOTCH MD FROM UST)

DIAGNOSTIC EXAM - AUG 2015

ANGELA PAULINE P. CALIMAG-LOYOLA (TOP 8 - FEB 2015 MED BOARDS; TOPNOTCH MD FROM UST)

DIAGNOSTIC EXAM - AUG 2015

119

Penile carcinoma in situ has a strong SIMILAR TO PREVIOUS BOARD EXAM association with: CONCEPT/PRINCIPLE. Page 52 of Topnotch Handout. A. HPV 6 Penile Carcinoma in situ is strongly associated with B. HPV 11 HPV 16 infection. C. HPV 16 D. HPV 18 E. HPV 31

ANGELA PAULINE P. CALIMAG-LOYOLA (TOP 8 - FEB 2015 MED BOARDS; TOPNOTCH MD FROM UST)

DIAGNOSTIC EXAM - AUG 2015

120

Psammoma bodies are frequently encountered Page 59 of Topnotch Handout. PSaMMoma bodies are ANGELA PAULINE P. in the following conditions except? encountered in A-D. CALIMAG-LOYOLA A. Papillary thyroid CA (TOP 8 - FEB 2015 B. Serous cystadeno CA MED BOARDS; C. Meningioma TOPNOTCH MD D. Mesothelioma FROM UST) E. Medullary thyroid CA

DIAGNOSTIC EXAM - AUG 2015

121

Which of the following is an effect of PAF? A. Vasodilation B. Bronchoconstriction C. Decreased vascular permeability D. Decreased platelet aggregation E. Decreased leukocyte adhesion

122

Cell-derived mediators of inflammation, in contrast with plasma-derived mediators A. Are normally sequestered in granule and can be rapidly secreted by granule exocytosis in response to stimulus. B. Are produced mainly in the liver. C. Are inactive precursors that must be activated, usually by a series of proteolytic cleavages to acquire their biologic properties. D. Act only in one or few target cell types. E. None of the above. The following presents with granulomatous inflammation EXCEPT: A. Histoplasmosis B. Tuberculosis C. Sarcoidosis D. Schistomiasis E. Molluscum contagiosum

116

117

118

123

Platelet activating factor is a phospholipid-dervided mediator that has multiple inflammatory effects. It cause platelet aggregation, vasoconstriction, bronchoconstriction, increased venular permeability, increased leukocyte adhesion to endothelium and chemotaxis. (Robbin's) PAF causes bronchoconstriction and also vasodilation in low doses. SIMILAR TO PREVIOUS BOARD EXAM CONCEPT. Options B and C are properties of plasma-derived mediators of inflammation. Option D is incorrect. Both types of mediators can act on one or few target cells. SIMILAR TO PREVIOUS BOARD EXAM CONCEPT.

LYNN DARYL FELICIANO VILLAMATER, MD (TOP 5 - FEB 2015 MED BOARDS; TOPNOTCH MD FROM EAC)

MIDTERM 1 EXAM - AUG 2015

LYNN DARYL FELICIANO VILLAMATER, MD (TOP 5 - FEB 2015 MED BOARDS; TOPNOTCH MD FROM EAC)

MIDTERM 1 EXAM - AUG 2015

Histoplasma mimics tuberculosis. All except E would show granulomatous type of inflammation in the affected organs. SIMILAR TO PREVIOUS BOARD EXAM CONCEPT.

LYNN DARYL FELICIANO VILLAMATER, MD (TOP 5 - FEB 2015 MED BOARDS; TOPNOTCH MD FROM EAC)

MIDTERM 1 EXAM - AUG 2015

TOPNOTCH MEDICAL BOARD PREP PATHOLOGY SUPEREXAM Page 18 of 99 For inquiries visit www.topnotchboardprep.com.ph or email us at [email protected]

TOPNOTCH MEDICAL BOARD PREP PATHOLOGY SUPEREXAM For inquiries visit www.topnotchboardprep.com.ph or email us at [email protected] Item # 124

QUESTION What is the mechanism of edema in nephrotic syndrome? A. Glomerular injury B. Increased capillary permeability to proteins C. Decreased protein absorption D. Tubulointerstitial disorder E. Increased hydrostatic pressure

EXPLANATION

AUTHOR

TOPNOTCH EXAM MIDTERM 1 EXAM - AUG 2015

Increased permeability to plasma proteins resulting from either structural or physicochemical alteration allows protein to escape from the plasma into the urinary space. Massive proteinuria depletes serum albumin resulting in hypoalbuminemia, and thus decreased colloid osmotic pressure of the blood with subsequent accumulation of fluid in the interstitial tissues. Sodium and water retention due to compensatory secretion of aldosterone and stimulation of sympathetic system also contributes and aggravates the edema. SIMILAR TO PREVIOUS BOARD EXAM CONCEPT. This is a morphologic description of Tay-Sachs Disease, a deficiency of hexosaminidase. SIMILAR TO PREVIOUS BOARD EXAM CONCEPT.

LYNN DARYL FELICIANO VILLAMATER, MD (TOP 5 - FEB 2015 MED BOARDS; TOPNOTCH MD FROM EAC)

LYNN DARYL FELICIANO VILLAMATER, MD (TOP 5 - FEB 2015 MED BOARDS; TOPNOTCH MD FROM EAC)

MIDTERM 1 EXAM - AUG 2015

125

A 24-year old male patient presented with cherry-red spots in the macula. Morphology of the brain shows neurons ballooned with cytoplasmic vacuoles. What enzyme is deficient in this condition? A. Alpha 1,4-glucosidase B. Sphingomyelinase C. Arylsulfatase D. Iduronidase E. Hexosaminidase

126

A 48-year old male patient who underwent kidney transplant suddenly developed bloody urine few hours after the procedure. Morphologic changes in this pattern of rejection will reveal: A. Extensive interstitial mononuclear cell infiltration and edema B. Thrombotic occlusion of capillaries and fibrinoid necrosis C. Necrotizing vasculitis with endothelial cell necrosis D. Neutrophilic infiltration and deposition of immunoglobulin, complement, and fibrin E. Interstitial fibrosis and tubular atrophy with loss of renal parenchyma. What is the hallmark of tissue repair? A. Vasoconstriction B. Blood clot formation C. Granulation tissue D. Tissue remodeling E. Wound contraction

Hyperacute rejection occurs few minutes to few hours after transplant. Option A describes Acute cellular rejection. Options C and D are mophologic findings found in acute humoral rejection. Option E describes the morphology of kidney in chronic rejection.

LYNN DARYL FELICIANO VILLAMATER, MD (TOP 5 - FEB 2015 MED BOARDS; TOPNOTCH MD FROM EAC)

MIDTERM 1 EXAM - AUG 2015

SIMILAR TO PREVIOUS BOARD EXAM CONCEPT.

LYNN DARYL FELICIANO VILLAMATER, MD (TOP 5 - FEB 2015 MED BOARDS; TOPNOTCH MD FROM EAC)

MIDTERM 1 EXAM - AUG 2015

128

The first step in phagocytosis is: A. Formation of phagocytic vacuole B. Degradation of ingested material C. Recognition of particle D. Attachment E. Engulfment

Steps in phagocytosis: 1. Recognition and attachment; 2. Engulfment and formation of phagocytic vacuole, 3. Killing and degradation of ingested material. SIMILAR TO PREVIOUS BOARD EXAM CONCEPT.

LYNN DARYL FELICIANO VILLAMATER, MD (TOP 5 - FEB 2015 MED BOARDS; TOPNOTCH MD FROM EAC)

MIDTERM 1 EXAM - AUG 2015

129

An 18-year old male patient presents with easy fatigability, fever, and cutaneous bleeding. Bone marrow biopsy showed 40% myeloblast. What is the most likely diagnosis? A. ALL B. AML C. CML D. Burkitt's lymphoma E. Adult T cell lymphoma Morphologic finding/s in alcoholic hepatitis include: A. Councilman bodies B. Hepatocyte swelling and necrosis C. Lymphoid aggregates within portal tracts D. Hepatocyte apoptosis E. All of the above

The diagnosis of AML is based on the presence of at least 20% myeloid blasts in the bone marrow. Robbins 9th ed., p. 613 . The most common manifestation of AML include fever, easy fatigability and bleeding. SIMILAR TO PREVIOUS BOARD EXAM CONCEPT.

LYNN DARYL FELICIANO VILLAMATER, MD (TOP 5 - FEB 2015 MED BOARDS; TOPNOTCH MD FROM EAC)

MIDTERM 1 EXAM - AUG 2015

The rest are features of viral hepatitis. SIMILAR TO PREVIOUS BOARD EXAM CONCEPT.

LYNN DARYL FELICIANO VILLAMATER, MD (TOP 5 - FEB 2015 MED BOARDS; TOPNOTCH MD FROM EAC)

MIDTERM 1 EXAM - AUG 2015

131

The most common cause of sudden cardiac death in Myocardial infaction is A. Congestive heart failure B. Ventricular fibrillation C. Pulmonary edema D. Acute pericarditis E. Ventricular rupture

SIMILAR TO PREVIOUS BOARD EXAM CONCEPT.

LYNN DARYL FELICIANO VILLAMATER, MD (TOP 5 - FEB 2015 MED BOARDS; TOPNOTCH MD FROM EAC)

MIDTERM 1 EXAM - AUG 2015

132

Juxta-articular osteopenia is characteristic of A. Systemic lupus erythematosus B. Rheumatoid arthritis C. Osteoarthritis D. Ankylosing spondylitis E. Gouty arthritis

SIMILAR TO PREVIOUS BOARD EXAM CONCEPT.

LYNN DARYL FELICIANO VILLAMATER, MD (TOP 5 - FEB 2015 MED BOARDS; TOPNOTCH MD FROM EAC)

MIDTERM 1 EXAM - AUG 2015

127

130

TOPNOTCH MEDICAL BOARD PREP PATHOLOGY SUPEREXAM Page 19 of 99 For inquiries visit www.topnotchboardprep.com.ph or email us at [email protected]

TOPNOTCH MEDICAL BOARD PREP PATHOLOGY SUPEREXAM For inquiries visit www.topnotchboardprep.com.ph or email us at [email protected] Item # 133

QUESTION True of female hemophilia carrier A. Both X chromosomes are defective B. Decrease Factor VIII C. 25% of her offspring are affected D. One of the X chromosome shows abnormality E. Transmits disease to half her sons and half her daughters

EXPLANATION

AUTHOR

TOPNOTCH EXAM MIDTERM 1 EXAM - AUG 2015

Hemophilia is an X-linked recessive disorders wherein heterozygous female or female carriers does not express full phenotypic change because of paired normal allele, with random inactivation of one of the X chromosome leading to variability. Only 1 of the X chromosome is abnormal. Decrease in Factor VIII is a manifestation of affected male offspring. Option C is characteristic of AR while Option E is a property of XLinked Dominant disorders. This is a case of Staphylococcal scalded-skin syndrome or Ritter disease caused by S. aureus. It is distinguished from toxic epidermal necrolysis or Lyell’s disease which is secondary to drug hypersensitivity and causes desquamation at the level of epidermal-dermal junction.

LYNN DARYL FELICIANO VILLAMATER, MD (TOP 5 - FEB 2015 MED BOARDS; TOPNOTCH MD FROM EAC) LYNN DARYL FELICIANO VILLAMATER, MD (TOP 5 - FEB 2015 MED BOARDS; TOPNOTCH MD FROM EAC)

MIDTERM 1 EXAM - AUG 2015

134

An 8-year old male presented with sunburnlike rash that spread over the entire body and evolves into fragile bullae. Desquamation of epidermis follows at the level of granulosa layer. This is most likely caused by: A. Staphylococcus aureus B. Streptococcus pyogenes C. Viridans streptococcus D. Drug hypersensitivity E. Pseudomonas aeruginosa

135

A 62-year old female presented with fever and cough for the past few days. She also had necrotizing oval lesion on her extremities. The most likely etiology is: A. Staphylococcus aureus B. Streptococcus pyogenes C. Clostridium perfringens D. Bacillus anthracis E. Pseudomonas aeruginosa The most likely renal pathology in multiple myeloma is: A. Tubulo-interstitial nephritis B. Membranoproliferative glomerulonephritis C. Acute glomerulonephritis D. Focal segmental glomerulosclerosis E. Tubular necrosis

Pseudomonas causes necrotizing pneumonia, vasculitis accompanied by thrombosis and hemorrhage. It proliferates widely, penetrating deeply into the veins and spreads hematogenously. Ecthyma gangrenosum, well-demarcated necrotic and hemorrhagic oval skin lesion, often appear.

LYNN DARYL FELICIANO VILLAMATER, MD (TOP 5 - FEB 2015 MED BOARDS; TOPNOTCH MD FROM EAC)

MIDTERM 1 EXAM - AUG 2015

Renal insufficiency in multiple myeloma can be due to Bence Jones proteins which are directly toxic to epithelial cell, and accumulation of light chains which results to tubulo-interstitial nephritis. Other causes include hypercalcemia nad hyperuricemia.

LYNN DARYL FELICIANO VILLAMATER, MD (TOP 5 - FEB 2015 MED BOARDS; TOPNOTCH MD FROM EAC)

MIDTERM 1 EXAM - AUG 2015

A 45-year old male, smoker, presented with cough, copious sputum, and progressive dyspnea on exertion for the last two years. On physical examination, he has wheezes all over his lung fields, and cyanotic lips. Morphologic findings of the lungs in this condition will most likely show a/an: A. Abnormally large alveoli separated by thin septa and deformed respiratory bronchioles. B. Enlargement of the mucus-secreting glands of the bronchi and hyperemia and edema of the mucus membranes. C. Dilatation of bronchi and bronchioles caused by destruction of the muscle and elastic tissue. D. Thickening of the basement membrane, edema and inflammatory infiltrate in the bronchial walls. E. Cobblestone pleural surfaces and patchy interstitial fibrosis varying in intensity. The most common malignancy of the stomach: A. Carcinoid B. Lymphoma C. Adenocarcinoma D. Squamous cell carcinoma E. Gastrointestinal stromal tumor

Chronic cough, copious sputum in a smoker points to chronic bronchitis as the diagnosis. Option A - emphysema. Option C - Bronchiectasis. Option D - Asthma; Option E - Idiopathic interstitial fibrosis. SIMILAR TO PREVIOUS BOARD EXAM CONCEPT.

LYNN DARYL FELICIANO VILLAMATER, MD (TOP 5 - FEB 2015 MED BOARDS; TOPNOTCH MD FROM EAC)

MIDTERM 1 EXAM - AUG 2015

SIMILAR TO PREVIOUS BOARD EXAM CONCEPT.

LYNN DARYL FELICIANO VILLAMATER, MD (TOP 5 - FEB 2015 MED BOARDS; TOPNOTCH MD FROM EAC)

MIDTERM 1 EXAM - AUG 2015

A 57-year old, obese male was brought to the hospital because of sudden onset of chest pain, characterized as squeezing, accompanied by diaphoresis and nausea. Neutrophils were noted to be elevated. What is the pathophysiologic mechanism of his condition? A. Tissue necrosis B. Alveolar edema C. Infection D. Inflammation of airways E. B and C Thrombosis of what vessel will most most likely involve the posterior portion of the ventricular septum? A. Left anterior descending coronary artery B. Left circumflex coronary artery C. Marginal branch of left circumflex artery D. Right coronary artery E. Left main coronary artery

This is a case of acute myocardial infarction. Myocardial necrosis begins at approximately 30 minutes after coronary occlusion. SIMILAR TO PREVIOUS BOARD EXAM CONCEPT.

LYNN DARYL FELICIANO VILLAMATER, MD (TOP 5 - FEB 2015 MED BOARDS; TOPNOTCH MD FROM EAC)

MIDTERM 1 EXAM - AUG 2015

Right coronary artery (30-40%) infarct involves the inferior/posterior wall of LV, posterior portion of ventricular septum, inferior/posterior RV free wall in some cases; LADA involves the anterior wall of LV near apex, anterior portion of ventricular septum and apex circumferentially; Left circumflex artery involves the lateral wall of left ventricle except the apex. (Robbin's)

LYNN DARYL FELICIANO VILLAMATER, MD (TOP 5 - FEB 2015 MED BOARDS; TOPNOTCH MD FROM EAC)

MIDTERM 1 EXAM - AUG 2015

136

137

138

139

140

TOPNOTCH MEDICAL BOARD PREP PATHOLOGY SUPEREXAM Page 20 of 99 For inquiries visit www.topnotchboardprep.com.ph or email us at [email protected]

TOPNOTCH MEDICAL BOARD PREP PATHOLOGY SUPEREXAM For inquiries visit www.topnotchboardprep.com.ph or email us at [email protected] Item # 141

QUESTION

EXPLANATION

AUTHOR

TOPNOTCH EXAM MIDTERM 2 EXAM - AUG 2015

A 30 year old male admitted for dengue fever on the 7th day of illness develops pleural effusion, what is the explanation for the pleural effusion? A. Decreased plasma oncotic pressure B. Increased capillary permeability C. Increased capillary hydrostatic pressure D. Decreased platelets E. None of the above JS, a 70 year old male who was a smoker of 40 pack years was diagnosed with squamous cell carcinoma of the lung, which among the following is the most common site of origin of this lesion? A. Trachea B. Secondary bronchus C. Primary bronchus D. Terminal bronchioles E. None of the above

Increased capillary permeability is the pathophysiologic mechanism of pleural effusion in dengue fever more than decreased platelets

EDWARD HARRY VALLAJERA, MD (TOP 8 - FEB 2015 MED BOARDS; TOPNOTCH MD FROM PERPETUAL BINAN)

The major (primary bronchus) is the most common site of development of squamous cell carcinoma of the lung

EDWARD HARRY VALLAJERA, MD (TOP 8 - FEB 2015 MED BOARDS; TOPNOTCH MD FROM PERPETUAL BINAN)

MIDTERM 2 EXAM - AUG 2015

143

RLDL, a 40 year old female was a diagnosed case of SLE, she had a renal biopsy done and results were brought to you, you know that the most common type of renal lesion of lupus has: A. Mesangial lupus glomerulonephritis B. Diffuse proliferative glomerulonephritis C. Membranous glomerulopathy D. Focal proliferative glomerulopathy E. None of the above

Diffuse proliferative GN is the most common as well as the most severe form of glomerulonephritis

EDWARD HARRY VALLAJERA, MD (TOP 8 - FEB 2015 MED BOARDS; TOPNOTCH MD FROM PERPETUAL BINAN)

MIDTERM 2 EXAM - AUG 2015

144

What is the cytokine that stimulates collagen synthesis A. TNF B. IL-1 C. VEGF D. TGF-B E. PDGF

TGF-B stimulates collagen synthesis, IL-1 stimulates fever as well as TNF, VEGF causes angiogenesis, PDGF causes collagenase secretion.

EDWARD HARRY VALLAJERA, MD (TOP 8 - FEB 2015 MED BOARDS; TOPNOTCH MD FROM PERPETUAL BINAN)

MIDTERM 2 EXAM - AUG 2015

145

MIV, a 48 year old female underwent Pap smear, which of the following cytologic findings suggest the presence of HPV infection A. Atypical ductal cells B. Diffuse thickening of the basement membrane C. Koilocytic atypia D. A and B E. None of the above RF, a 30 year old male underwent lymph node biopsy, the result showed a starry sky pattern, you know that his malignancy is associated with? A. Hepatitis B infection B. Cytomegalovirus C. Herpes zoster D. Epstein Barr virus E. Infection with viruses belonging to filoviridae family Which among the following is important in the pathology of HIV infection? A. CD8 B. CD4 C. Macrophages D. Dendritic cells E. Neutrophils

Koilocytic atypia is a characteristic finding in the pap smear which suggests HPV infection

EDWARD HARRY VALLAJERA, MD (TOP 8 - FEB 2015 MED BOARDS; TOPNOTCH MD FROM PERPETUAL BINAN)

MIDTERM 2 EXAM - AUG 2015

Starry sky pattern is seen in Burkitt's lymphoma which is associated with EBV infection

EDWARD HARRY VALLAJERA, MD (TOP 8 - FEB 2015 MED BOARDS; TOPNOTCH MD FROM PERPETUAL BINAN)

MIDTERM 2 EXAM - AUG 2015

HIV invades the CD4 cells which are also responsible for the cell mediated immunity

EDWARD HARRY VALLAJERA, MD (TOP 8 - FEB 2015 MED BOARDS; TOPNOTCH MD FROM PERPETUAL BINAN)

MIDTERM 2 EXAM - AUG 2015

148

A patient was diagnosed with melanoma, one of the following is a prognostic factor for melanoma except A. tumor depth B. presence of tumor infiltrating lymphocytes C. number of dendritic cells D. location of the tumor E. gender

The number of skin dendritic cells is not a prognostic factor for melanoma

EDWARD HARRY VALLAJERA, MD (TOP 8 - FEB 2015 MED BOARDS; TOPNOTCH MD FROM PERPETUAL BINAN)

MIDTERM 2 EXAM - AUG 2015

149

BB, a 70 year old chronic alcoholic presents to you with changes in sensorium, you requested a liver biopsy, which of the following would you expect to see: A. Councilman bodies B. Mallory bodies C. Negri bodies D. Psamomma bodies E. None of the above

Mallory bodies are eosinophilic cytoplasmic clumps in hepatocytes signifying liver injury, councilman bodies are eosinophilic globules seen in acute hepatitis, Negri bodies are associated with rabies while psamomma bodies are associated with meningioma, prolactinoma, ovarian serous cystadenoma and papillary thyroid CA.

EDWARD HARRY VALLAJERA, MD (TOP 8 - FEB 2015 MED BOARDS; TOPNOTCH MD FROM PERPETUAL BINAN)

MIDTERM 2 EXAM - AUG 2015

142

146

147

TOPNOTCH MEDICAL BOARD PREP PATHOLOGY SUPEREXAM Page 21 of 99 For inquiries visit www.topnotchboardprep.com.ph or email us at [email protected]

TOPNOTCH MEDICAL BOARD PREP PATHOLOGY SUPEREXAM For inquiries visit www.topnotchboardprep.com.ph or email us at [email protected] Item # 150

QUESTION

EXPLANATION

AUTHOR

TOPNOTCH EXAM MIDTERM 2 EXAM - AUG 2015

MBDM, a 29 year old male underwent a tissue section that showed 40% myeloid cells, what is the diagnosis? A. CLL B. AML C. ALL D. Mantle cell lymphoma E. Non-Hodgkins lymphoma

AML presents with >20% of myeloblasts in the bone marrow.

EDWARD HARRY VALLAJERA, MD (TOP 8 - FEB 2015 MED BOARDS; TOPNOTCH MD FROM PERPETUAL BINAN)

151

Most common bone involved in osteoporosis A. Pelvis B. Tibia C. Femur D. Vertebra E. Ribcage

The vertebra are the most commonly affected as it is a weight bearing bone.

EDWARD HARRY VALLAJERA, MD (TOP 8 - FEB 2015 MED BOARDS; TOPNOTCH MD FROM PERPETUAL BINAN)

MIDTERM 2 EXAM - AUG 2015

152

What is the most common cause of nephritic syndrome in children? A. Hepatitis B B. Intake of toxic substances C. Antecedent GABHS infection D. Viral infections E. None of the above



EDWARD HARRY VALLAJERA, MD (TOP 8 - FEB 2015 MED BOARDS; TOPNOTCH MD FROM PERPETUAL BINAN)

MIDTERM 2 EXAM - AUG 2015

153

Which of the following lesions is more prone to develop breast CA A. Proliferative atypia B. Proliferation without atypia C. Carcinoma in situ D. Fibroadenoma E. None of the above

Lobular carcinoma in situ is a fertile ground for breast cancer, proliferation without atypia and proliferation with atypia is hyperplasia, fibroadenoma is a benign condition.

EDWARD HARRY VALLAJERA, MD (TOP 8 - FEB 2015 MED BOARDS; TOPNOTCH MD FROM PERPETUAL BINAN)

MIDTERM 2 EXAM - AUG 2015

154

Which among the following is involved in staging a particular malignancy except A. Presence of metastasis B. Lymph node involvement C. Degree of differentiation D. Size of the mass E. None of the above

Degree of differentiation or tissue grading is not needed in the staging in most malignancies

EDWARD HARRY VALLAJERA, MD (TOP 8 - FEB 2015 MED BOARDS; TOPNOTCH MD FROM PERPETUAL BINAN)

MIDTERM 2 EXAM - AUG 2015

155

KB, a 32 year old female fond of using beauty products tried a new product on her face, 5 days later, she noticed redness on the areas of application of the new product, what is the most likely explanation: A. IgE mediated immune reaction B. Cytotoxin mediated C. Immunoglobulin-antigen complex D. Cell mediated hypersensitivity E. None of the above

Type IV or delayed hypersensitivity develops usually after 48 hours on application of offending agent on the skin of the patient.

EDWARD HARRY VALLAJERA, MD (TOP 8 - FEB 2015 MED BOARDS; TOPNOTCH MD FROM PERPETUAL BINAN)

MIDTERM 2 EXAM - AUG 2015

156

What is the mechanism of DIC in meningococcemia? A. Massive intravascular coagulation occuring everywhere in the circulation B. Endothelial dysfunction C. Depletion of clotting factors D. All of the above E. None of the above A 2 week old infant had been undergoing treatment for sepsis when the patient went into hypotension and died, what is the most likely explanation for the infant's death? A. DIC B. Sepsis C. Thrombocytopenia D. B and C E. None of the above The classic anaphylactic reaction is due to: A. Th1 B. Th2 C. NK cell D. Macrophage E. None of the above

All of the above are the mechanism of DIC

EDWARD HARRY VALLAJERA, MD (TOP 8 - FEB 2015 MED BOARDS; TOPNOTCH MD FROM PERPETUAL BINAN)

MIDTERM 2 EXAM - AUG 2015

The most likely cause is septic shock due to systemic vasodilation.

EDWARD HARRY VALLAJERA, MD (TOP 8 - FEB 2015 MED BOARDS; TOPNOTCH MD FROM PERPETUAL BINAN)

MIDTERM 2 EXAM - AUG 2015

SIMILAR TO PREVIOUS BOARD EXAM CONCEPT/PRINCIPLE. Th2 subset of helper T cells are overactive in patients who develop the classic anaphylactic reaction

EDWARD HARRY VALLAJERA, MD (TOP 8 - FEB 2015 MED BOARDS; TOPNOTCH MD FROM PERPETUAL BINAN)

MIDTERM 2 EXAM - AUG 2015

Among the following adaptations of the body to injury, which of the following is a fertile ground for neoplasia? A. Hyperplasia B. Atrophy C. Hypertrophy D. Metaplasia E. None of the above

Metaplasia is defined as the change from one adult cell type into another and is a fertile ground for malignancies, examples of malignancies originating from metaplastic change include Barett's esophagus leading to esophageal adenoCA, cervical CA from squamous metaplasia of cervical columnar cells, SQCA of the lung due to squamous metaplasia of respiratory epithelium.

EDWARD HARRY VALLAJERA, MD (TOP 8 - FEB 2015 MED BOARDS; TOPNOTCH MD FROM PERPETUAL BINAN)

MIDTERM 2 EXAM - AUG 2015

157

158

159

TOPNOTCH MEDICAL BOARD PREP PATHOLOGY SUPEREXAM Page 22 of 99 For inquiries visit www.topnotchboardprep.com.ph or email us at [email protected]

TOPNOTCH MEDICAL BOARD PREP PATHOLOGY SUPEREXAM For inquiries visit www.topnotchboardprep.com.ph or email us at [email protected] Item # 160

QUESTION

EXPLANATION

AUTHOR

TOPNOTCH EXAM MIDTERM 2 EXAM - AUG 2015

What is the most common type of gastric malignancy? A. Lymphoma B. Gastric adenocarcinoma C. Carcinoid tumor D. Gastric adenoma E. Pleomorphic adenoma

Gastric adenoCA is the most common type of gastric malignancy

EDWARD HARRY VALLAJERA, MD (TOP 8 - FEB 2015 MED BOARDS; TOPNOTCH MD FROM PERPETUAL BINAN)

161

This can be a pathologic or physiologic process which is described as the increase in the number of cells in an organ. A. Hypertrophy B. Hyperplasia C. Atrophy D. Metaplasia E. Anaplasia

hypertrophy is increase in size of cells. Atrophy is decrease in number and size of the cell. Metaplasia is replacement of one differentiated type to another. Anaplasia happens when cells lose their morphologic characteristic of a mature cells.

HAROLD JAY S. BAYTEC, MD (TOP 10 - FEB 2015 MED BOARDS; TOPNOTCH MD FROM FEU)

MIDTERM 3 EXAM - AUG 2015

162

In Rheumatic Heart Disease, at which layer/layers of the heart can you find Aschoff bodies? A. pericardium B. myocardium C. endocardium D. B and C E. All layers Valvular vegetations characrerized as small, warty vegetations along the lines of closure of the valve leaflets are usually seen in what disease? A. RHD B. Infective endocarditis C. Non bacterial thrombotic endocarditis D. Libman sacks endocarditis E. marantic endocarditis In Libman-Sacks Disease, which of the following valve is usually affected? A. Tricuspid B. Pulmonary C. Mitral D. Aortic E. All valves are equally involve

Aschoff bodies can be found in all layers of the heart in RF and RHD. SIMILAR TO PREVIOUS BOARD EXAM CONCEPT/PRINCIPLE

HAROLD JAY S. BAYTEC, MD (TOP 10 - FEB 2015 MED BOARDS; TOPNOTCH MD FROM FEU)

MIDTERM 3 EXAM - AUG 2015

IE are large irregular masses on the valve cusps that can extend onto the chordae. NBTE/marantic endocarditis are small bland vegetations usually attached at the line of closure. LSE are small or medium sized vegetation on either or both sides of the valve leaflets

HAROLD JAY S. BAYTEC, MD (TOP 10 - FEB 2015 MED BOARDS; TOPNOTCH MD FROM FEU)

MIDTERM 3 EXAM - AUG 2015

In Robbins, Mitral and Tricuspid. In Medscape, Mitral and Aortic.

HAROLD JAY S. BAYTEC, MD (TOP 10 - FEB 2015 MED BOARDS; TOPNOTCH MD FROM FEU)

MIDTERM 3 EXAM - AUG 2015

165

Which of the following caspase is an example of executioner in apoptosis? A. 8 B. 9 C. 10 D. 6 E. 5

caspases 8, 9, 10 are initiators while caspases 6 and 3 are executioners

HAROLD JAY S. BAYTEC, MD (TOP 10 - FEB 2015 MED BOARDS; TOPNOTCH MD FROM FEU)

MIDTERM 3 EXAM - AUG 2015

166

Wire-loop capillaries are seen in which type of kidney disease? A. Lupus nephritis B. RPGN C. Diabetic nephropathy D. Polycystic kidney disease E. Acute tubular necrosis

SIMILAR TO PREVIOUS BOARD EXAM CONCEPT/PRINCIPLE

HAROLD JAY S. BAYTEC, MD (TOP 10 - FEB 2015 MED BOARDS; TOPNOTCH MD FROM FEU)

MIDTERM 3 EXAM - AUG 2015

167

What is the most common and the most severe type of Lupus nephropathy? A. Type I B. Type II C. Type III D. Type IV E. Type V

type IV or the diffuse proliferative glomerulonephritis is the most common and most severe type.

HAROLD JAY S. BAYTEC, MD (TOP 10 - FEB 2015 MED BOARDS; TOPNOTCH MD FROM FEU)

MIDTERM 3 EXAM - AUG 2015

168

Minamata disease is an environmental concern because it causes cerebral palsy, deafness, blindness, mental retardation and other major CNS defects in children in utero. Which of the following elements is the cause of the disease? A. arsenic B. lead C. mercury D. cadnium E. chromium

Mercury poisoning is the one associated with Minamata disease.

HAROLD JAY S. BAYTEC, MD (TOP 10 - FEB 2015 MED BOARDS; TOPNOTCH MD FROM FEU)

MIDTERM 3 EXAM - AUG 2015

169

All of the following cancers are strongly associated with smoking except: A. Oral cavity B. pancreas C. esophagus D. bladder E. Breast

cancers of the lung, larynx, esophagus, pancreas, bladder, oral cavity are strongly associated with smoking.

HAROLD JAY S. BAYTEC, MD (TOP 10 - FEB 2015 MED BOARDS; TOPNOTCH MD FROM FEU)

MIDTERM 3 EXAM - AUG 2015

163

164

TOPNOTCH MEDICAL BOARD PREP PATHOLOGY SUPEREXAM Page 23 of 99 For inquiries visit www.topnotchboardprep.com.ph or email us at [email protected]

TOPNOTCH MEDICAL BOARD PREP PATHOLOGY SUPEREXAM For inquiries visit www.topnotchboardprep.com.ph or email us at [email protected] Item # 170

EXPLANATION

AUTHOR

Among the types of Hodgkin's lymphoma, this has the highest association with EBV and also has the poorest prognosis. A. Lymphocyte-depleted B. Lymphocyte rich C. Lymphocyte predominant D. Nodular sclerosis E. Mixed cellularity

lymphocyte depleted has the highest association with EBV and also has the poorest prognosis.Nodular sclerosis is the most common and has excellent prognosis

HAROLD JAY S. BAYTEC, MD (TOP 10 - FEB 2015 MED BOARDS; TOPNOTCH MD FROM FEU)

Chromosome translocations are important in diagnosing and prognosticating certain types of cancers. Which of the following may be seen in Burkitt's lymphoma? A. t9:22 B. T8:14 C. T14:18 D. T11:14 E. T15:17 Which of the following types of vasculitis is highly associated with c-ANCA or the antibodies against proteinase-3? A. PAN B. Takayasu arteritis C. Wegener Granulomatosis D. Microscopic polyangitis E. Churg-Strauss syndrome

9:22 is associated with CML. 14:18 is associated with Follicular lymphoma. 11:14 is associated with mantle cell lymphoma. 15:17 is AML M3

HAROLD JAY S. BAYTEC, MD (TOP 10 - FEB 2015 MED BOARDS; TOPNOTCH MD FROM FEU)

MIDTERM 3 EXAM - AUG 2015

p-ANCA is associated with microscopic polyangitis and Churgstraus while c-ANCA is for Wegener Granulomatosis

HAROLD JAY S. BAYTEC, MD (TOP 10 - FEB 2015 MED BOARDS; TOPNOTCH MD FROM FEU)

MIDTERM 3 EXAM - AUG 2015

A patient came in for second opinion about his newly diagnosed polyarteritis nodosa. All of the following statements are true about this disease EXCEPT A. Does NOT affect lungs B. Affects mainly young adults C. Usually involves medium sized arteries D. NOT associated with hepatitis B E. very responsive to steroid therapy and cyclophospahmide Which among the following arteries is commonly affected in Kawasaki disease? A. Abdominal Aorta B. Thoracic aorta C. Arch of the aorta D. Pulmonary artery E. Coronary arteries

30% of patients with PAN are HBsAg positive. pathology topnotch handout page 106. PAN does not affect the lungs, affects mainly young adults, usually involves medium sized arteries, very responsive to steroid therapy and cyclophosphamide, and 30% of patients are HbsAg positive.

HAROLD JAY S. BAYTEC, MD (TOP 10 - FEB 2015 MED BOARDS; TOPNOTCH MD FROM FEU)

MIDTERM 3 EXAM - AUG 2015



HAROLD JAY S. BAYTEC, MD (TOP 10 - FEB 2015 MED BOARDS; TOPNOTCH MD FROM FEU)

MIDTERM 3 EXAM - AUG 2015

175

Which among the following medications is/are essential in treating a patient with Kawasaki disease? A. ASA B. IVIg C. Steroids D. A and B E. All of the above

Steroids is contraindicated because it promote rupture of coronary vessels and aggration of the disease.

HAROLD JAY S. BAYTEC, MD (TOP 10 - FEB 2015 MED BOARDS; TOPNOTCH MD FROM FEU)

MIDTERM 3 EXAM - AUG 2015

176

This is a primary malignant small round-cell tumor of the bone and soft tissue which usually involve the mutation of a gene in chromosome 22. Homer-Wright rosettes can be seen histologically and onion-skin appearance can be seen radiographically on patients with this disease. A. Ewing sarcoma B. Osteosarcoma C. Chondrosarcoma D. Giant Cell Tumor E. Fibrous Dysplasia Reiter syndrome is an autoimmune disease which is usually triggered by infections caused by Shigella, Salmonella, Yersinia, and Chlamydia. Which among the following is/are component of the disease? A. urethritis B. arthritis C. uveitis D. A and B E. All of the above



HAROLD JAY S. BAYTEC, MD (TOP 10 - FEB 2015 MED BOARDS; TOPNOTCH MD FROM FEU)

MIDTERM 3 EXAM - AUG 2015

these three are the triad of Reiter syndrome

HAROLD JAY S. BAYTEC, MD (TOP 10 - FEB 2015 MED BOARDS; TOPNOTCH MD FROM FEU)

MIDTERM 3 EXAM - AUG 2015

In infective endocarditis, what is the most organism involved in patients with native but previously damaged heart valves? A. Strep viridans B. Staph epidermidis C. Strep bovis D. Staph aureus E. Enterococcus



HAROLD JAY S. BAYTEC, MD (TOP 10 - FEB 2015 MED BOARDS; TOPNOTCH MD FROM FEU)

MIDTERM 3 EXAM - AUG 2015

171

172

173

174

177

178

QUESTION

TOPNOTCH MEDICAL BOARD PREP PATHOLOGY SUPEREXAM Page 24 of 99 For inquiries visit www.topnotchboardprep.com.ph or email us at [email protected]

TOPNOTCH EXAM MIDTERM 3 EXAM - AUG 2015

TOPNOTCH MEDICAL BOARD PREP PATHOLOGY SUPEREXAM For inquiries visit www.topnotchboardprep.com.ph or email us at [email protected] Item # 179

QUESTION

EXPLANATION

AUTHOR

TOPNOTCH EXAM MIDTERM 3 EXAM - AUG 2015

Which of the following conditions will cause edema? A. Increase oncotic pressure in the vessels B. Decrease plasma volume in blood vessels C. Decrease oncotic pressure outside the vessels D. Decrease in plasma proteins E. None of the above Which of the following diseases is an X-linked dominant disorder? A. G6PD deficiency B. Hemophilia C. Ehlers Danlos syndrome D. Marfan syndrome E. Alport syndrome

decrease in plasma proteins will decrease oncotic pressure within the vessel which can cause edema

HAROLD JAY S. BAYTEC, MD (TOP 10 - FEB 2015 MED BOARDS; TOPNOTCH MD FROM FEU)

Alport syndrome and vitamin D resistant rickets are the only most commonly associated with X linked dominant genetic transmission.

HAROLD JAY S. BAYTEC, MD (TOP 10 - FEB 2015 MED BOARDS; TOPNOTCH MD FROM FEU)

MIDTERM 3 EXAM - AUG 2015

181

What is the most common tumor in the stomach? A. adenocarcinoma B. GIST C. carcinoid D. adenoma E. lymphoma

SIMILAR TO PREVIOUS BOARD EXAM CONCEPT/PRINCIPLE.

JEAN PAOLO M. DELFINO, MD (TOP 10 - FEB 2015 MED BOARDS; TOPNOTCH MD FROM FATIMA)

FINAL EXAM - AUG 2015

182

True of Luetic aneurysm? A. Inflammation begins in the tunica adventitia B. With characteristic tree-barking appearance C. Involvement of aorta favors development of superimosed atheromatosisof the aortic root D. All of the above E. A and C Microscopically, the earliest change of systemic Hypertensive Heart Disease is? A. Increase in transverse diameter of myocytes B. Irregular cellular enlargement C. Irregular nuclear enlargement D. Interstitial fibrosis E. All of the above In acute Rheumatic Fever, inflammation and Aschoff bodies are commonly found in which layer of the heart? A. pericardium B. myocardium C. endocardium D. All of the above E. A and B

All are true regarding Syphilitic/Luetic aneurysm

JEAN PAOLO M. DELFINO, MD (TOP 10 - FEB 2015 MED BOARDS; TOPNOTCH MD FROM FATIMA)

FINAL EXAM - AUG 2015



JEAN PAOLO M. DELFINO, MD (TOP 10 - FEB 2015 MED BOARDS; TOPNOTCH MD FROM FATIMA)

FINAL EXAM - AUG 2015

SIMILAR TO PREVIOUS BOARD EXAM CONCEPT/PRINCIPLE.. During acute RF, diffuse inflammation and Aschoff bodies may be found in any of the 3 layers of the heart- hence the lesion is called a PANCARDITIS.

JEAN PAOLO M. DELFINO, MD (TOP 10 - FEB 2015 MED BOARDS; TOPNOTCH MD FROM FATIMA)

FINAL EXAM - AUG 2015

185

What is the valve most commonly affected in Libman-Sacks endocarditis? A. Mitral B. Tricuspid C. Aortic D. Pulmonic E. A and B

JEAN PAOLO M. DELFINO, MD (TOP 10 - FEB 2015 MED BOARDS; TOPNOTCH MD FROM FATIMA)

FINAL EXAM - AUG 2015

186

What is the most severe form of α-thalassemia which is caused by deletion of all four α-globin genes? A. αThalassemia trait B. Silent carrier state C. Hydrops fetalis D. Hemoglobin H disease E. A and D

SIMILAR TO PREVIOUS BOARD EXAM CONCEPT/PRINCIPLE.. In SLE, mitral and tricuspid valvulitis with small, sterile vegetations, called Libman-Sacks endocarditis is ocassionally encountered. according to harrison, "The characteristic endocardial lesions of SLE are verrucous valvular abnormalities known as LibmanSacks endocarditis . They most often are located on the left-sided cardiac valves, particularly on the ventricular surface of the posterior mitral leaflet, and are made up almost entirely of fibrin. " So yes, the correct answer is mitral valve. Hydrops fetalis is the most severe form of αthalassemia. it is caused by deletion of all four αglobin genes. In the fetus, excess γ-globin chains form tetramers (hemoglobin Barts) that have such a high affinity for oxygen that they deliver little to tissues.

JEAN PAOLO M. DELFINO, MD (TOP 10 - FEB 2015 MED BOARDS; TOPNOTCH MD FROM FATIMA)

FINAL EXAM - AUG 2015

187

A 58 year old male presented with easy fatigability, anorexia and weight loss. On PE, there were noted lymphadenopathies and hepatosplenomegaly. CBC revealed leukocytosis, with absolute lymphocyte count 3000 per mm3. There were numerous small, round lymphocytes with scant cytoplasm which are frequently disrupted in the process of making smears. What is the diagnosis? A. ALL B. SLL C. CLL D. CML E. Multiple Myeloma

CLL and SLL differ only in the degree of peripheral blood lymphocytosis. Most affected patients have sufficient lymphocytosis to fulfill the diagnostic requirement for CLL (absolute lymphocyte count >4000 per mm3). In this condition, lymph nodes are diffusely effaced by an infiltrate of predominantly small lymphocytes 6 to 12 μm in diameter with round to slightly irregular nuclei, condensed chromatin, and scant cytoplasm. Some of these cells are usually disrupted in the process of making smears, producing so-called smudge cells.

JEAN PAOLO M. DELFINO, MD (TOP 10 - FEB 2015 MED BOARDS; TOPNOTCH MD FROM FATIMA)

FINAL EXAM - AUG 2015

180

183

184

TOPNOTCH MEDICAL BOARD PREP PATHOLOGY SUPEREXAM Page 25 of 99 For inquiries visit www.topnotchboardprep.com.ph or email us at [email protected]

TOPNOTCH MEDICAL BOARD PREP PATHOLOGY SUPEREXAM For inquiries visit www.topnotchboardprep.com.ph or email us at [email protected] Item # 188

QUESTION

EXPLANATION

AUTHOR

TOPNOTCH EXAM FINAL EXAM - AUG 2015

What is the most common lesion of the salivary gland? A. Sialolithiasis B. Pelomorphic adenoma C. Warthin tumor D. Mucocele E. Sialadenitis

Mucocele is the most common lesion of the salivary glands and it results from either blockage or rupture of a salivary gland duct, with consequent leakage of saliva into the surrounding connective tissue stroma.

JEAN PAOLO M. DELFINO, MD (TOP 10 - FEB 2015 MED BOARDS; TOPNOTCH MD FROM FATIMA)

Acute hepatitis is characterized by what morphologic feature? A. Swollen hepatocytes with irregulaly clumped cytoplasmic organelles and large clear spaces B. Presence of feathery degeneration C. Deposition of fibrous tissue in the portal tracts and periportal septa D. Bridging inflammation and necrosis E. All of the above True statement about Crigler-Najjar Syndrome type I A. Liver morphology is normal B. There is decreased UGT1A1 enzyme activity C. It is generally mild with occasional kernicterus D. There is mutation in MRP2 E. Hyperbilirubinemia is of direct type Hepatitis B carrier state is most commonly acquired via what mode of transmission? A. Heterosexual transmission B. Needle-stick injuries C. Vertical transmission D. Blood transfusion E. Homosexual transmission

With acute hepatitis, hepatocyte injury takes the form of diffuse swelling (“ballooning degeneration”;), so the cytoplasm looks empty and contains only scattered eosinophilic remnants of cytoplasmic organelles. Feathery degeneration is retention of biliary material causing foamy appearance of hepatocytes seen in cholestatic liver injury. C and D are charcteristics of chronic hepatitis.

JEAN PAOLO M. DELFINO, MD (TOP 10 - FEB 2015 MED BOARDS; TOPNOTCH MD FROM FATIMA)

FINAL EXAM - AUG 2015

Crigler-Najjar Syndrome type I is an autosomal recessive condition wherein there is ABSENT UGT1A1 activity causing indirect hyperbilirubinemia. Liver pathology is normal and it is fatal in the neonatal period. MRP2 mutation is seen in Dubin-Johnson Syndrome

JEAN PAOLO M. DELFINO, MD (TOP 10 - FEB 2015 MED BOARDS; TOPNOTCH MD FROM FATIMA)

FINAL EXAM - AUG 2015

In endemic regions such as Africa and Southeast Asia, spread of Hepatitis B from an infected mother to a neonate during birth (vertical transmission) is common. These neonatal infections often lead to a carrier state for life.

JEAN PAOLO M. DELFINO, MD (TOP 10 - FEB 2015 MED BOARDS; TOPNOTCH MD FROM FATIMA)

FINAL EXAM - AUG 2015

192

Variant of Renal Cell Carcinoma which is made up of pale eosinophilic cells, often with a perinuclear halo, arranged in solid sheets with a concentration of the largest cells around blood vessels? A. Clear cell CA B. Papillary CA C. Chromophobe CA D. Collecting Duct CA E. Urothelial CA

SIMILAR TO PREVIOUS BOARD EXAM CONCEPT/PRINCIPLE.. Chromophobe renal carcinoma is made up of pale eosinophilic cells, often with a perinuclear halo, arranged in solid sheets with a concentration of the largest cells around blood vessels.

JEAN PAOLO M. DELFINO, MD (TOP 10 - FEB 2015 MED BOARDS; TOPNOTCH MD FROM FATIMA)

FINAL EXAM - AUG 2015

193

This agent predisposes susceptible populations to develop prostate cancer? A. Nickel B. Chromium C. Cadmium D. Vinyl chloride E. Benzene

The uses of cadmium include yellow pigments and phosphors; found in solders; used in batteries and as alloy and in metal platings and coatings. It is associated with prostate cancer. Nickel is associated with nose, lung cancer; Benzene- Leukemia, Hodgkin lymphoma; Chromium- lungs; Vinyl chlorideAngiosarcoma, liver.

JEAN PAOLO M. DELFINO, MD (TOP 10 - FEB 2015 MED BOARDS; TOPNOTCH MD FROM FATIMA)

FINAL EXAM - AUG 2015

194

Metastasis unequivocally marks a tumor as malignant. What cancer does not metastasize? A. hepatoma B. seminoma C. lipoma D. glioma E. meningioma

All cancers metastasize except glioma and basal cell carcinoma. Lipoma does not metastasize. But the question asks what "cancer" does not metastasize. Lipoma is benign so it should be ruled out from the choices. Glioma is the correct answer

JEAN PAOLO M. DELFINO, MD (TOP 10 - FEB 2015 MED BOARDS; TOPNOTCH MD FROM FATIMA)

FINAL EXAM - AUG 2015

195

True of chronic bronchitis except A. Grossly, there may be hyperemia, swelling and edema of the mucus membranes B. The ratio of the number of mucus glands to the thickness of the wall is increased C. There is excessive mucinous to mucopurulent secretions layering the epithelial surfaces D. The major change is in the size of the mucus glands E. All are true 20 year old male presented with history of allergic rhinitis, asthma and recurrent sinusitis. An autoimmune etiology is suspected. What is the most likely autoantibody involved in this condition? A. Anti-myeloperoxidase Ab B. Anti-proteinase 3 Ab C. Anti-endomysial Ab D. Anti-mitochondrial Ab E. Anti-saccharomyces Ab

Although the numbers of goblet cells increase slightly, the major change is in the size of the mucous gland (hyperplasia). This increase can be assessed by the ratio of the thickness of the mucous gland layer to the thickness of the wall between the epithelium and the cartilage (Reid index).

JEAN PAOLO M. DELFINO, MD (TOP 10 - FEB 2015 MED BOARDS; TOPNOTCH MD FROM FATIMA)

FINAL EXAM - AUG 2015

Diagnosis for this case is Churg-Strauss Syndrome. It is a small-vessel necrotizing vasculitis classically associated with asthma, allergic rhinitis, lung infiltrates, peripheral hypereosinophilia, and extravascular necrotizing granulomas. The autoantibody implicated in this condition is the antimyeloperoxidase antibody and p-ANCA. B-Wegener's; C- Celiac Disease; D- primary biliary cirrhosis; E- Crohn's disease.

JEAN PAOLO M. DELFINO, MD (TOP 10 - FEB 2015 MED BOARDS; TOPNOTCH MD FROM FATIMA)

FINAL EXAM - AUG 2015

189

190

191

196

TOPNOTCH MEDICAL BOARD PREP PATHOLOGY SUPEREXAM Page 26 of 99 For inquiries visit www.topnotchboardprep.com.ph or email us at [email protected]

TOPNOTCH MEDICAL BOARD PREP PATHOLOGY SUPEREXAM For inquiries visit www.topnotchboardprep.com.ph or email us at [email protected] Item # 197

QUESTION

EXPLANATION

AUTHOR

TOPNOTCH EXAM FINAL EXAM - AUG 2015

What is the most common site of ectopic pancreas? A. duodenum B. jejunum C. ileum D. Meckel's diverticulum E. spleen

Aberrantly situated, or ectopic, pancreatic tissue is found in about 2% of careful routine postmortem examinations. The favored sites for ectopia are the stomach and duodenum, followed by the jejunum, Meckel diverticula, and ileum.

JEAN PAOLO M. DELFINO, MD (TOP 10 - FEB 2015 MED BOARDS; TOPNOTCH MD FROM FATIMA)

8 year old male patient abruptly develops malaise, fever, nausea, oliguria, and hematuria 2 weeks after recovery from a sore throat. The patient has red cell casts in the urine, mild proteinuria, periorbital edema, and hypertension. What is the expected electron microscopy finding in this case? A. Loss of foot processes B. Subendothelial deposits C. Mesangial and paramesangial dense deposits D. Subepithelial deposits E. Subepithelial humps In the heart, grossly apparent bands of yellowed myocardium alternating with bands of darker, red-brown, uninvolved myocardium is known as tigered effect. This is due to accumulation of what material in the cardiac cells? A. calcium B. glycogen C. triglyceride D. cholesterol E. lipofuscin What is the immunologically mediated pathologic lesion seen in Acute Rheumatic Fever? A. Vascular dilation, edema, smooth muscle contraction B. Necrotizing vasculitis C. Phagocytosis and cell lysis D. Perivascular cellular infiltrates E. B and C The gene involved in Williams syndrome is: A. Fibrillin B. Collagen C. Elastin D. Reticulin E. Chondroitin

Diagnosis is PSGN. The electron microscopy finding in this condition is subepithelial humps. A- minimal change disease; B- MPGN; C- IgA nephropathy; D- Membranous GN

JEAN PAOLO M. DELFINO, MD (TOP 10 - FEB 2015 MED BOARDS; TOPNOTCH MD FROM FATIMA)

FINAL EXAM - AUG 2015

The terms steatosis and fatty change describe abnormal accumulations of triglycerides within parenchymal cells. This is most often seen in the liver and heart. In the heart, fat deposits create grossly apparent bands of yellowed myocardium alternating with bands of darker, red-brown, uninvolved myocardium (tigered effect)

JEAN PAOLO M. DELFINO, MD (TOP 10 - FEB 2015 MED BOARDS; TOPNOTCH MD FROM FATIMA)

FINAL EXAM - AUG 2015

Acute Rheumatic Fever is a Type 2 hypersensitivity reaction. It is an antibody mediated reaction causing phagocytosis and lysis of cells; inflammation; in some diseases, functional derangements without cell or tissue injury. A- Type 1; B- type 3; D- type 4

JEAN PAOLO M. DELFINO, MD (TOP 10 - FEB 2015 MED BOARDS; TOPNOTCH MD FROM FATIMA)

FINAL EXAM - AUG 2015

Williams syndrome: supravalvular aortic stenosis, hypercalcemia, cognitive abnormalities, and hallmark facial anomalies.

GRACE ARVIOLA, MD (TOP 3 - AUG 2014 MED BOARDS; TOPNOTCH MD)

DIAGNOSTIC EXAM - FEB 2015

202

Anti-centromere antibodies are present in: A. Sjogren syndrome B. SLE C. Wegener's granulomatosis D. CREST syndrome E. Churg-Strauss syndrome

CREST syndrome: calcinosis, Raynaud's phenomenon, esophageal dysmotility, sclerodactyly, and telangiectasia.

GRACE ARVIOLA, MD (TOP 3 - AUG 2014 MED BOARDS; TOPNOTCH MD)

DIAGNOSTIC EXAM - FEB 2015

203

The presence of Heinz bodies and bite cells in a patient having hemolytic anemia strongly suggests: A. Sickle cell anemia B. G6PD deficiency C. Alpha thalassemia D. Multiple myeloma E. Hereditary spherocytosis

Heinz bodies: RBCs with denatured hemoglobin. Bite cells results when splenic macrophages pluck out these inclusions.

GRACE ARVIOLA, MD (TOP 3 - AUG 2014 MED BOARDS; TOPNOTCH MD)

DIAGNOSTIC EXAM - FEB 2015

204

Nocturnal pain relieved by aspirin intake is characteristic of: A. Osteomyelitis B. Osteoid osteoma C. Chondroma D. Osteosarcoma E. Ossifying fibroma

The pain is probably caused by excessive prostaglandin E2 (PGE2) production by the proliferating osteoblasts.

GRACE ARVIOLA, MD (TOP 3 - AUG 2014 MED BOARDS; TOPNOTCH MD)

DIAGNOSTIC EXAM - FEB 2015

205

What is the most important factor in the prognosis of GI carcinoid? A. Size B. Nuclear grade C. Location D. Metastasis E. Presence of other tumors

Location in the jejunum or ileum carries the worst prognosis.

GRACE ARVIOLA, MD (TOP 3 - AUG 2014 MED BOARDS; TOPNOTCH MD)

DIAGNOSTIC EXAM - FEB 2015

198

199

200

201

TOPNOTCH MEDICAL BOARD PREP PATHOLOGY SUPEREXAM Page 27 of 99 For inquiries visit www.topnotchboardprep.com.ph or email us at [email protected]

TOPNOTCH MEDICAL BOARD PREP PATHOLOGY SUPEREXAM For inquiries visit www.topnotchboardprep.com.ph or email us at [email protected] Item # 206

QUESTION

EXPLANATION

AUTHOR

TOPNOTCH EXAM DIAGNOSTIC EXAM - FEB 2015

Which condition is most likely to give rise to hepatocellular carcinoma? A. Hereditary tyrosinemia B. Chronic Hepatitis B C. Chronic Hepatitis C D. Chronic Alcoholism E. Autoimmune hepatitis

In hereditary tyrosinemia, almost 40% of patients develop the tumor despite adequate dietary control. However, this condition is extremely rare.

GRACE ARVIOLA, MD (TOP 3 - AUG 2014 MED BOARDS; TOPNOTCH MD)

207

Mucocutaneous lymph node syndrome preferentially affects the: A. Temporal artery B. Aortic arch C. Renal arteries D. Coronary arteries E. Tibial and radial arteries

AKA Kawasaki disease because it presents with conjunctival and oral erythema and erosion, edema of the hands and feet, erythema of the palms and soles, a desquamative rash, and cervical lymph node involvement. Approximately 20% of untreated patients develop cardiovascular sequela involving the coronary arteries.

GRACE ARVIOLA, MD (TOP 3 - AUG 2014 MED BOARDS; TOPNOTCH MD)

DIAGNOSTIC EXAM - FEB 2015

208

A person with hypothyroidism and sensorineural deafness most likely has an abnormality of the: A. H-P-O axis B. Thyroglobulin C. Anion transporter D. Iodide channel E. Peripheral deiodinases

Pendred syndrome is caused by a mutation in the SLC26A4 gene whose product, pendrin, is an anion transporter expressed on the apical surface of thyrocytes and in the inner ear.

GRACE ARVIOLA, MD (TOP 3 - AUG 2014 MED BOARDS; TOPNOTCH MD)

DIAGNOSTIC EXAM - FEB 2015

209

What is the most common clinical manifestation among patients with Sipple Syndrome? A. Pheochromocytoma B. Medullary carcinoma of the thyroid C. Parathyroid adenoma D. Neuroganglioma E. Pituitary adenoma Coffin lid appearance is seen in calculi composed of: A. Calcium oxalate B. Cystine C. Magnesium ammonium phosphate D. Uric acid E. Calcium carbonate

Primary hyperparathyroidism is the most common manifestation in MEN 1 (Wermer syndrome).

GRACE ARVIOLA, MD (TOP 3 - AUG 2014 MED BOARDS; TOPNOTCH MD)

DIAGNOSTIC EXAM - FEB 2015

Struvite stones are composed of magnesium ammonium phosphate.

GRACE ARVIOLA, MD (TOP 3 - AUG 2014 MED BOARDS; TOPNOTCH MD)

DIAGNOSTIC EXAM - FEB 2015

211

Which are considered vascular phenomena in infective endocarditis? A. Janeway lesions and Osler nodes B. Splinter hemorrhages and Roth spots C. Janeway lesions and Roth spots D. Splinter hemorrhages and Janeway lesions E. Osler nodes and Roth spots

Osler nodes and Roth spots are immunologic phenomena. Osler nodes are subcutaneous nodules in the pulp of the digits. Roth spots are retinal hemorrhages in the eyes. Splinter hemorrhages are micro-thromoboemboli. Janeway lesions are erythematous or hemorrhagic nontender lesions on the palms or soles.

GRACE ARVIOLA, MD (TOP 3 - AUG 2014 MED BOARDS; TOPNOTCH MD)

DIAGNOSTIC EXAM - FEB 2015

212

Which heavy metal plays a role in the treatment of relapsing acute promyelocytic leukemia? A. Arsenic B. Lead C. Copper D. Mercury E. Zinc

AML with the t(15,17) (promyelocytic leukemia) is treated with pharmacologic doses of ATRA (all-trans retinoic acid) combined with conventional chemotherapy, or more recently, with arsenic salts, which appear to cause PML-RARa to be degraded.

GRACE ARVIOLA, MD (TOP 3 - AUG 2014 MED BOARDS; TOPNOTCH MD)

DIAGNOSTIC EXAM - FEB 2015

213

Which characteristic of a tumor will make it less radiosensitive? A. Peripheral location B. Highly oxygenated C. Poorly vascularized D. A and B E. B and C

Poorly vascularized, poorly oxygenated, and a central location makes a tumor less radiosensitive.

GRACE ARVIOLA, MD (TOP 3 - AUG 2014 MED BOARDS; TOPNOTCH MD)

DIAGNOSTIC EXAM - FEB 2015

214

Subacute combined degeneration of the spinal cord is seen in deficiency of which vitamin? A. B1 B. B2 C. B3 D. B5 E. B12

The combined degenration of both ascending and descending tracts of the spinal cord is characteristic of vitamin B12 deficiency.

GRACE ARVIOLA, MD (TOP 3 - AUG 2014 MED BOARDS; TOPNOTCH MD)

DIAGNOSTIC EXAM - FEB 2015

215

A 38 year old male patient presents with a longstanding history of abdominal pain and intermittent diarrhea. Imaging studies showed cobblestone appearance of the colon with ulcers that were sporadically located. Biopsy revealed non-caseating granulomas. What antibodies might this patient potentially have? A. p-ANCA B. c-ANCA C. Anti-Saccharomyces antibodies D. Antibodies to gliadin E. Anti-mitochondrial antibodies

Anti-Saccharomyces antibodies are present in Crohn's disease.

GRACE ARVIOLA, MD (TOP 3 - AUG 2014 MED BOARDS; TOPNOTCH MD)

DIAGNOSTIC EXAM - FEB 2015

210

TOPNOTCH MEDICAL BOARD PREP PATHOLOGY SUPEREXAM Page 28 of 99 For inquiries visit www.topnotchboardprep.com.ph or email us at [email protected]

TOPNOTCH MEDICAL BOARD PREP PATHOLOGY SUPEREXAM For inquiries visit www.topnotchboardprep.com.ph or email us at [email protected] Item # 216

QUESTION

EXPLANATION

AUTHOR

What fusion gene is carried in the Philadelphia chromosome of CML? A. C-myc and N-myc B. BCR-ABL C. BRCA1 and BRCA2 D. NOD2 E. JAK/STAT

CML is distinguished from other myeloproliferative disorders by the presence of a chimeric BCR-ABL gene derived from portions of the BCR gene on chromosome 22 and the ABL gene on chromosome 9.

GRACE ARVIOLA, MD (TOP 3 - AUG 2014 MED BOARDS; TOPNOTCH MD)

A 42 year old female presents with a 5 year history of palpable breast mass. Biopsy showed signet ring cells arranged in an Indian file pattern. The mass is most likely: A. Invasive ductal carcinoma B. Invasive lobular carcinoma C. Paget's disease of the breast D. Carcinoma in situ E. Medullary carcinoma of the breast The chest x-ray of a cyanotic infant revealed an egg-shaped heart. Which statement is correct? A. The pulmonary veins empty into the right atrium. B. The underlying pathology is obstruction of the right ventricular outflow tract. C. This condition is common among infants of diabetic mothers. D. The infant probably has Down's syndrome. E. The aorta is constricted at a site just distal to the ligamentum arteriosum. A patient with retroperitoneal fibrosis is also at risk of having: A. Left-sided varicocele B. Hashimoto's thyroiditis C. Crohn's disease D. Primary sclerosing cholangitis E. Whipple's disease

The histologic hallmark is the presence of dysohesive infiltrating tumor cells, often arranged in a single file pattern or in loose clusters or sheets. Signet ring cellscontaining an intracytoplasmic mucin droplet are common.

GRACE ARVIOLA, MD (TOP 3 - AUG 2014 MED BOARDS; TOPNOTCH MD)

DIAGNOSTIC EXAM - FEB 2015

This is transposition of the great arteries. Choice A is TAPVC, choice B is TOF, choice D is endocardial cushion defect, and choice E is coarctation of the aorta.

GRACE ARVIOLA, MD (TOP 3 - AUG 2014 MED BOARDS; TOPNOTCH MD)

DIAGNOSTIC EXAM - FEB 2015

Retroperitoneal fibrosis is associated with Reidel thyroiditis, PSC, and right-sided varicocele.

GRACE ARVIOLA, MD (TOP 3 - AUG 2014 MED BOARDS; TOPNOTCH MD)

DIAGNOSTIC EXAM - FEB 2015

220

This subtype of Hodgkin's lymphoma is considered non-classical. A. Nodular sclerosis B. Lymphocyte-predominant C. Lymphocyte-rich D. Lymphocyte-depleted E. Mixed cellularity

In LP, the Reed-Sternberg cells have a distinctive Bcell immunophenotype that differs from that of the classical types.

GRACE ARVIOLA, MD (TOP 3 - AUG 2014 MED BOARDS; TOPNOTCH MD)

DIAGNOSTIC EXAM - FEB 2015

221

Which of the following is true regarding dense deposit disease? A. Characterized by immune complex deposits in the glomerular basement membrance and activiation of classical pathway B. Highly responsive to treatment with immunosuppresive drugs such as pulse cyclophosphamide combined with steroids C. Diminished serum levels of factor B and properdin D. Dense materials are primarily deposited on the podocyte-GBM junction E. Less recurrence among allograft recipients compared to Type I MPGN 54 y/o male patient went for an OPD consult secondary to a chief complaint of urinary frequency and nocturia. PMHx: CABG-2 years ago. FM: (+) DM (+) Htn both parents. he is on insulin therapy since he was 45 y/o; other medications include the ff: losartan, metoprolol, ACEI. pertinent physical examination showed the following data: BP 140/80, PR 98 bpm, T 37.1C; BMI: 34 kg/m2, soft nontender abdomen, no suprapubic tenderness, no CVA tenderness, (+) bipedal edema. which of the following is a correct morphologic characteristics behind his most plausible cause of urinary complaint? A. presence of focal thickening of the glomerular capillary basement membrane B. presence of mesangial proliferation secondary to hyperplasia of the mesangial cells C. PAS negative nodular lesions noted on the periphery of the glomerulus. D. presence of hyaline arteriolosclerosis affecting both afferent and efferent arterioles E. All of the above

dense deposit disease (MPGN type II) - is a primary type MPGN associated with activation of the alternative pathway. This is based on the diminished serum levels of factor B and properdin along with normal C1 an C4 levels. Ultrastructurally, type II MPGN is characterized by deposition of dense material along the GBM proper. Natural history of patients with this disease showed refractory to combined pulse cyclophosphamide and steroids. there is also high incidence of recurrence among transplant patient compared to type I MPGN . Robbins 8th ed pp 928929

LEAN ANGELO SILVERIO, MD (TOP 4 - AUG 2014 MED BOARDS; TOPNOTCH MD), MD

MIDTERM EXAM 1 - FEB 2015

A 54 y/o patient complaining of nocturia and frequency along with constellation of clinical findings such as on diabetic and on insulin therapy, hypertensive, (+) Family history, obese, bipedal edema without any overt clinical evidence of infection is most likely suffering from DM nephropathy. renal morphologic changes include the following. Widespread thickening of the capillary basement membrane. diffuse increase of mesangial matrix secondary to GBM thickening with minimal mesangial cell proliferation. Presence of PAS positive nodules ( Kimmelsteil Wilson nodules) along the periphery of the glomerulus. diffuse hyaline arteriolosclerosis affecting both afferent and efferent arterioles. Robbins 8th ed pp 1140-1141

LEAN ANGELO SILVERIO, MD (TOP 4 - AUG 2014 MED BOARDS; TOPNOTCH MD), MD

MIDTERM EXAM 1 - FEB 2015

217

218

219

222

TOPNOTCH MEDICAL BOARD PREP PATHOLOGY SUPEREXAM Page 29 of 99 For inquiries visit www.topnotchboardprep.com.ph or email us at [email protected]

TOPNOTCH EXAM DIAGNOSTIC EXAM - FEB 2015

TOPNOTCH MEDICAL BOARD PREP PATHOLOGY SUPEREXAM For inquiries visit www.topnotchboardprep.com.ph or email us at [email protected] Item # 223

224

225

226

QUESTION

EXPLANATION

AUTHOR

A 45 y/o male patient brought to ER due to massive hematochezia, he is a known case of liver cirrhosis secondary to Chronic Hep B infection. Clinical findings are of the following: lethargic, BP 60 mmHg palpatory, PR 145 bpm, pale palpebral conjunctiva, icteric sclera, cold clammy extremities. he was successfully stabilized after undergoing endoscopic band ligation along with blood transfusion and intravenous hydration. during his hospital stay, there was a noted increase in the serum creatinine from 1.1 -2.1 mg/dl accompanied by oliguria. which of the following is true regarding the cause of the oliguria? A. the most affected nephron segment is proximal segment of the proximal convoluted tubule B. he is at the stage wherein there is increased risk for generalized infection and electrolyte abnormalities like hypokalemia C. cell swelling, vacuolization and tubulorrhexis primarily along the descending limb of henle D. Eosinophilic hyaline cast noted along the ascending limb of henle and Distal tubules E. all of the above A 66 y.o male went for consult secondary to recent onset of weight loss accompanied by anorexia and easy fatigability. PE findings showed normotensive, tachycardic, pale palpebral conjunctiva, enlarged lymph nodes on bilateral cervical, axillary, inguinal regions. Traube space is obliterated and liver edge is 5cm from right subcostal margin. CBC showed normocytic anemia, thrombocytopenia and lymphocytosis. A diagnosis of chronic lymphocytic leukemia is entertain, which of the following is consistent regarding the diagnosis? A. there is diffuse effacement of lymph nodes by predominantly small lymphocytes along with loose aggregrates of larger activated lymphocytes. B. disruption of normal immune function accompanied with hypergammaglobulinemia C. prolymphocytic transformation to diffuse B cell lymphoma is rare D. There is a high incidence of chromosomal translocation E. all of the above. Which of the following factors is associated with worse prognosis in Acute Lymphoblastic Leukemia? A. Presence of philadelphia chromosome B. Peripheral blood blast count of 80,000 C. Presentation of symptoms at 5 years of age D. presence of chromosomal translocation t (12,21) E. hyperploidy

the rise of the serum creatinine and presence of oliguria after an hypotensive episode is suggestive of ischemic type AKI. Morphologic changes include patchy necrosis primarily along the the straight portion of the PCT and ascending limb of henle, eosinophilic cast composed of tamm horsfall protein are noted along the ascending limb and the distal tubules. In contrast, toxin mediated AKI is manifested by diffuse damage along the whole length of the PCT. based on lab findings and having oliguria, patient is still on maintenance phase. It is the recovery phase that is associated with increase urinary volume, hypokalemia and susceptibility to infection. Robbins 8th ed pp 937-938

LEAN ANGELO SILVERIO, MD (TOP 4 - AUG 2014 MED BOARDS; TOPNOTCH MD), MD

CLL is the most common leukemia of adults in the western countries. It is distinguished to SLL only by absolute lymphocytic count of >4000/mm3. Morphologic characteristics include diffuse effacement of the LN along with loose aggregrates of atypical larger lymphocytes which collectively called proliferation centers. unlike most leukemias, it is rare for CLL to undergo chromosomal translocation. furthermore, CLL also has an unknown mechanism that disrupts the normal immune function resulting to a decrease in antibody production. CLL is also prone to undergo Richter transformation wherein there is transformation of the primary cancer to DLBCL.

LEAN ANGELO SILVERIO, MD (TOP 4 - AUG 2014 MED BOARDS; TOPNOTCH MD), MD

MIDTERM EXAM 1 - FEB 2015

ALL has one of the greatest prognosis since it is highly responsive to chemotherapy, however there are some parameters associated with worse prognosis. These are the following: age under 2 at presentation, presentation at adulthood or adolescence, blast count of more than 100,000; presence of phidelphia chromosome. On the otherhand, favorable prognostic factors include the ff: an age 2-10 years old, a low white cell count, hyperploidy, trisomy of chromosome 4,7,and 10, presence of t(12,21) Robbins 8th ed pp 603 expect clinical scenarios in your board exam. An elderly patient presenting with recurrent pneumonia, low back pain and polyuria supported with lab evidence of pancytopenia and radiographic finding of compression fractures of lesions on vertebral column is consistent with multiple myeloma. features of MM stems from the effect of plasmacytic growth on axial skeleton, production of excessive immunoglobulins and alteration of humoral immunity. Because of the factors produced by plasma cells, it causes the activation of osteoclast leading to bone resorption and attendant hypercalcemia. Definitive diagnosis is made only by doing bone marrow examination. Patient usually suffers renal insufficiency secondary to bence jones proteinuria. Infection is the most common cause of death among this patient secondary to abnormal immunoglobulin production and decreased number of white blood cells. MGUS and not MM is the most common plasma cell dyscrasia

LEAN ANGELO SILVERIO, MD (TOP 4 - AUG 2014 MED BOARDS; TOPNOTCH MD), MD

MIDTERM EXAM 1 - FEB 2015

LEAN ANGELO SILVERIO, MD (TOP 4 - AUG 2014 MED BOARDS; TOPNOTCH MD), MD

MIDTERM EXAM 1 - FEB 2015

A 72 y/o male went for consult secondary to 3 days of fever accompanied by productive cough. History revealed that he had multiple episodes of pneumonia within the last 3 years .aside from the primary complaint, he also had polyuria, continuous low back pain even at rest and constipation. PE are normal except for the ff: pale conjunctiva, bibasilar crackles, hypotonic bowel sounds, paravertebral tenderness, smooth, non nodular slightly enlarged prostate, weak bilateral LE strength 2/5. Radiographic examination showed pneumonia of bilateral lower lobes, with incidental finding of thoracic compression fractures of t5-t6 t8-t9 level and multiple lucencies along the thoracic vertebral bodies. CBC showed normocytic normochromic anemia, leukopenia, thrombocytopenia. UA revealed massive protenuria. which of the following is consistent with the most possible primary diagnosis A. patient is suffering from hypocalcemia and hypoglobulinemia B. it is the most common plasma cell dyscrasia C. definitive diagnosis can be established only through clinical and radiographic findings

TOPNOTCH MEDICAL BOARD PREP PATHOLOGY SUPEREXAM Page 30 of 99 For inquiries visit www.topnotchboardprep.com.ph or email us at [email protected]

TOPNOTCH EXAM MIDTERM EXAM 1 - FEB 2015

TOPNOTCH MEDICAL BOARD PREP PATHOLOGY SUPEREXAM For inquiries visit www.topnotchboardprep.com.ph or email us at [email protected] Item #

QUESTION

EXPLANATION

AUTHOR

TOPNOTCH EXAM

D. Infection is the most common cause of death E. all of the above

227

Which of the following is not an immune complex mediated disease? A. Systemic Lupus Erythematosus B. Multiple sclerosis C. Polyarteritis nodosa D. Poststreptococcal glomerulonephritis E. None of the above

Multiple sclerosis is an example of type IV hypersensitivity reaction. Other type III or immune complex mediated reaction aside from the choices are serum sickness, arthus reaction and reactive bacterial arthritis.

LEAN ANGELO SILVERIO, MD (TOP 4 - AUG 2014 MED BOARDS; TOPNOTCH MD), MD

MIDTERM EXAM 1 - FEB 2015

228

What is the most serious complication of Tuberculous meningitis? A. Tuberculoma formation B. Fibrinous basal exudates leading to cranial nerve palsies C. Choroid plexus involvement leading to diffuse meningoencephalitis D. Obliterative endarteritis E. none of the above A 42 y/o male presents with right abdominal mass associated with gross hematuria. He also had episodes of diaphoresis and refractory hypertension. Imaging showed renal mass and adrenal medullary tumor. What is your primary diagnosis? A. Sipple syndrome B. Tuberous sclerosis C. Von hippel lindau syndrome D. Li Fraumeni syndrome E. None of the above

the most serious complication of chronic tuberculous meningitis is arachnoid fibrosis leading to hydrocephalus and obliterative endarteritis leading to brain infarction. This is SIMILAR TO PREVIOUS BOARD EXAM CONCEPT/PRINCIPLE.

LEAN ANGELO SILVERIO, MD (TOP 4 - AUG 2014 MED BOARDS; TOPNOTCH MD), MD

MIDTERM EXAM 1 - FEB 2015

Von hippel lindau disease is a type of familial tumor syndrome characterized by renal cell carcinoma, pheochromocytoma and cerebellar hemangioblastoma. Sipple syndrome aka MEN type IIA is a triad of parathyroid hyperplasia, medullary carcinoma, pheochromocytoma. Tuberous sclerosis is associated with renal angiomyolipoma, retinal hamartoma,cortical tubers, cardiac rhabdomyoma, shagreen patches and ash leaf patch. Gorlin syndrome.Li Fraumeni syndrome is caused by p53 mutation and associated with sarcomas, breast cancer, adrenal cortical tumors, leukemia and gliomas. SIMILAR TO PREVIOUS BOARD EXAM CONCEPT/PRINCIPLE last aug 2014. taken verbatim Robbins 8th ed pp 704

LEAN ANGELO SILVERIO, MD (TOP 4 - AUG 2014 MED BOARDS; TOPNOTCH MD), MD

MIDTERM EXAM 1 - FEB 2015

LEAN ANGELO SILVERIO, MD (TOP 4 - AUG 2014 MED BOARDS; TOPNOTCH MD), MD

MIDTERM EXAM 1 - FEB 2015

Menetrier disease is characterized by diffuse hyperplasia of the foveolar epithelium of the body and fundus of the stomach. It is secondary to proliferation of mucous neck cells and not the gastric connective tissue. The hyperplasia is secondary to the excessive secretion of TGF alpha. although it is a self limiting condition, it has an increased risk for the development of gastric adenocarcinoma.

LEAN ANGELO SILVERIO, MD (TOP 4 - AUG 2014 MED BOARDS; TOPNOTCH MD), MD

MIDTERM EXAM 1 - FEB 2015

Rotavirus is the most common cause of severe childhood diarrhea. It affects only the apical and mid villous enterocytes leading to loss of absorptive capacity of the small intestine. All the other virus shows nonspecific affectation of different population of enterocytes. Robbins 8th ed pp 804-805.

LEAN ANGELO SILVERIO, MD (TOP 4 - AUG 2014 MED BOARDS; TOPNOTCH MD), MD

MIDTERM EXAM 1 - FEB 2015

229

230

231

232

What is the most striking histologic finding in Desquamative Interstitial Pneumonia secondary to smoking? A. Thickened alveolar septa due to sparse inflammatory lymphocytic infiltrate B. Mild interstitial fibrosis C. Necrotic type II pneumocytes D. accumulation of a large number of macrophages with brown pigment cytoplasmic bodies. E. none of the above An 18 yo female patient presents with chronic epigastric pain associated with weight loss, diarrhea, and peripheral edema. Upon endoscopy, menetrier disease was given as a diagnosis. Which of the following is true regarding the diagnosis? A. there is an irregular enlargement of the gastric rugal folds secondary to hyperplasia of gastric connective tissue B. associated with excessive secretion of platelet derived growth factor C. it is a progressive unremmittng condition however no risk for gastric adenocarcinoma D. characterized by hyperplasia of foveolar mucous neck cells E. all of the above Which of the following pathogenic organism can cause diarrhea by selectively destroying apical mature enterocytes of the small intestine? A. Norovirus B. adenovirus C. Rotavirus D. Norwalk virus E. All of the above

TOPNOTCH MEDICAL BOARD PREP PATHOLOGY SUPEREXAM Page 31 of 99 For inquiries visit www.topnotchboardprep.com.ph or email us at [email protected]

TOPNOTCH MEDICAL BOARD PREP PATHOLOGY SUPEREXAM For inquiries visit www.topnotchboardprep.com.ph or email us at [email protected] Item # 233

234

235

236

237

238

QUESTION A 61 y/o male presents with hesitancy, dysuria, and nocturia. BPH is entertained. Which of the following is true regarding the diagnosis? A. This is secondary to hypertrophy of the stromal and epithelial cells B. the microscopic hallmark is nodularity C. presence of true capsule creates a plane between the normal tissue and prostatic nodules D. the cause of the nodularity is secondary to increase mitosis of the epithelial cells E. all of the above what is the most serious consequence of alkaptonuria? A. Renal failure B. Hypertrophic cardiomyopathy C. Arthropathy D. Liver failure E. None of the above

Which of the following describes the resolution stage of lobar pneumonia A. Progressive enzymatic digestion of alveolar exudates producing granular semifluid debris ingested by macrophage B. Vascular dilatation with intraalveolar fluid and presence of numerous bacteria C. massive confluent exudation with neutrophils, rbc, and alveolar fibrin D. progressive disintegration of RBC and presence of fibrinosuppurative exudates E. none of the above A 28 y/o female veterinarian presents with 3 week history dry cough accompanied by low grade fever and night sweats. CXR revealed cavitary lesion noted on bilateral lung apices. AFB showed negative for 3 specimen. PPD is also negative. Which of the following is true regarding the most plausible diagnosis in this case? A. the organism is internalized primarily by NK cells and atypical lymphocytes after opsonization with antibody B. there is absence of caseation necrosis C. produces concentric calcification of the lesion ( tree bark appearance) when drug control is achieved D. rarely become disseminated even in immunocompromised state. E. all are correct. A 59 y/o female presents with rapidly enlarging anterior neck mass accompanied with proggressive of dyspnea, dysphagia and hoarseness of 1 month duration. What would be the most consistent pathologic finding if biopsy is done to the mass? A. large pleomorphic multinucleated giant cells with fusiform cells B. spindle cells with amyloid deposits on adjacent stroma C. finely dispersed chromatin giving a optically clear or empty appearance of the nucleus D. uniform cells forming small follicles containing colloid substance E. None of the above. A 45 y/o male presents with a rapidly growing nodular skin lesion on the forehead with noted telangiectasia on top of the lesion. If biopsy was done, which of the following will be consistent with the most plausible diagnosis? A. Highly anaplastic with necrosis and presence of polygonal cells with numerous areas of keratinization B. basaloid cells with hyperchromatic nuclei with palisading alignment in the periphery of the tumor C. hyperkeratosis with horn cyst D. basaloid cell with with hair like differentiation E. presence of civatte bodies

EXPLANATION

AUTHOR

TOPNOTCH EXAM MIDTERM EXAM 1 - FEB 2015

Benign Prostatic hyperplasia is characterized by hyperplasia of the stromal and epithelial cells along ther prostatic periurethral zone. The hallmark is the presence of nodularity. The compression of the adjacent normal prostate by the nodules creates a pseudocapsular plane. the pathogenesis behind the hyperplasia is not secondary to increase mitosis but rather impaired cell death.

LEAN ANGELO SILVERIO, MD (TOP 4 - AUG 2014 MED BOARDS; TOPNOTCH MD), MD

Alkaptonuria is an autosomal recessive disorder secondary to lack of homogentisic oxidase which converts homogentisic acid to methylacetoacetic acid. It causes black discoloration of the urine if it allows to stand on ambient air. It accumulates throughout the body causing blue to black pigmentation of soft tissue such as ears, nose and face. however, it also causes pigmentation of the articular cartilages causing its loss of resiliency and fibrillation. there is no association b/w alkaptonuria and cardiomyopathy,liver failure and renal failure. B- congestion, C- red hepatization, D- gray hepatization. Robbins 8th ed pp 712-713

LEAN ANGELO SILVERIO, MD (TOP 4 - AUG 2014 MED BOARDS; TOPNOTCH MD), MD

MIDTERM EXAM 1 - FEB 2015

LEAN ANGELO SILVERIO, MD (TOP 4 - AUG 2014 MED BOARDS; TOPNOTCH MD), MD

MIDTERM EXAM 1 - FEB 2015

the most likely impression regarding this case is histoplasmosis. TB is set aside because of the absence of positive sputum smear along with negative PPD. Histoplasmosis clinical and morphological presentation greatly mimicks that of TB. H. capsulatum is internalized by macrophage after opsonization. there is also presence of caseation necrosis. fulminant disemminated histoplasmosis can occur in immunocompromised state. This patient is also veterinarian which most likely expose to bird droppings.

LEAN ANGELO SILVERIO, MD (TOP 4 - AUG 2014 MED BOARDS; TOPNOTCH MD), MD

MIDTERM EXAM 1 - FEB 2015

based on the rapidity of symptoms, the most likely diagnosis is anaplastic carcinoma. B- Medullary carcinoma. C- papillary carcinoma. D- follicular carcinoma

LEAN ANGELO SILVERIO, MD (TOP 4 - AUG 2014 MED BOARDS; TOPNOTCH MD), MD

MIDTERM EXAM 1 - FEB 2015

based on the characteristics, this is most likely a case of basal cell carcinoma. A- squamous cell carcinoma, c- seborrheic keratoses, d- trichoepithelioma, e- lichen planus

LEAN ANGELO SILVERIO, MD (TOP 4 - AUG 2014 MED BOARDS; TOPNOTCH MD), MD

MIDTERM EXAM 1 - FEB 2015

TOPNOTCH MEDICAL BOARD PREP PATHOLOGY SUPEREXAM Page 32 of 99 For inquiries visit www.topnotchboardprep.com.ph or email us at [email protected]

TOPNOTCH MEDICAL BOARD PREP PATHOLOGY SUPEREXAM For inquiries visit www.topnotchboardprep.com.ph or email us at [email protected] Item # 239

QUESTION

EXPLANATION

AUTHOR

A 49 y/o female presented to the ER secondary to right tibial fracture after a fall from a standing height, Xray revealed middle transverse fracture of the right tibia with noyed enlarged, sclerotic irregular thickening of both cortical and cancellous bone. further physical examination noted weakness of bilateral hip flexors and knee flexors, sensory deficit on L3S1 dermatome bilateral. patient presents with prominent zygoma and supraorbital ridge with frontal bossing. she also has lateral rectus palsy on (R), (L) peripheral facial palsy. which of the following is the most likely diagnosis of the patient? A. Pagets disease B. early onset idiopathic osteoporosis C. osteomalacia D. multiple enchondromatosis E. Osteopetrosis What is the most common malignancy of the eyelid? A. Squamous cell carcinoma B. Basal cell carcinoma C. Sebaceous carcinoma D. lymphangioma E. Hemangioma

Pagets disease or Osteitis deformans is a rare skeletal disease characterized by haphazard mosiaic pattern of lamellar bone formation. Clinically it affects the axial skeleton leading to compression of multiple spinal and cranial nerve exits. Bony overgrowth of skull base also causes different cranial nerve palsies. they also prominence of facial bones presenting as leontiasis ossea.

LEAN ANGELO SILVERIO, MD (TOP 4 - AUG 2014 MED BOARDS; TOPNOTCH MD), MD

basal cell carcinoma is the most common malignancy of the eyelid with predilection on the lower lid and medial canthus. This is followed by sebaceous carcinoma and squamous cell carcinoma. Robbins 8th ed p 1348

LEAN ANGELO SILVERIO, MD (TOP 4 - AUG 2014 MED BOARDS; TOPNOTCH MD), MD

MIDTERM EXAM 1 - FEB 2015

In pathologic examination of a brain of a person who died from rabies which of the following areas of the brain where the pathognomonic negri bodies will be found? A. cerebrum B. cerebellum C. hippocampus D. thalamus E. B and C a 65 year old male patient presents with a chronic history of decreasing urinary stream, dribbling of urine and difficulty in voding, What is the pathophysiologic process involved in a person diagnosed to have BPH? A. hypertrophy B. hyperplasia C. neoplasia D. metaplasia E. none of the above

SIMILAR TO PREVIOUS BOARD EXAM CONCEPT/PRINCIPLE, in our exam we had to choose between cerebellum or hippocampus, both answers are correct and should be either cerebellum or hippocampus

KEVIN BRYAN LO, MD (TOP 7 - AUG 2014 MED BOARDS; TOPNOTCH MD)

MIDTERM 2 EXAM - FEB 2015

SIMILAR TO PREVIOUS BOARD EXAM CONCEPT/PRINCIPLE

KEVIN BRYAN LO, MD (TOP 7 - AUG 2014 MED BOARDS; TOPNOTCH MD)

MIDTERM 2 EXAM - FEB 2015

243

What percentage of patients with hepatitis B eventually proceed to have hepatocellular carcinoma? A. 6-15% B. 12-20% C. >30% D. 50% of glomeruli exhibiting crescent formation, fibrinoid necrosis, proliferation of endothelial and mesangial cells B. > deposits between basement membrane and epithelial cells of the glomeruli

MIGUEL RAFAEL RAMOS, MD (TOP 3 - FEB 2012 MED BOARDS; TOPNOTCH MD)

MIDTERM 1 EXAM - FEB 2013

Sinus histiocytosis (also called reticular hyperplasia refers to the distention and prominence of the lymphatic sinusoids. This particular form of hyperplasia is prominent in lymph nodes draining cancers such as carcinoma of the breast. (SIMILAR TO PREVIOUS BOARD EXAM CONCEPT/PRINCIPLE)

MIGUEL RAFAEL RAMOS, MD (TOP 3 - FEB 2012 MED BOARDS; TOPNOTCH MD)

MIDTERM 1 EXAM - FEB 2013

Guillan Barre Syndrome >> type IV reaction

MIGUEL RAFAEL RAMOS, MD (TOP 3 - FEB 2012 MED BOARDS; TOPNOTCH MD)

MIDTERM 1 EXAM - FEB 2013

TOPNOTCH MEDICAL BOARD PREP PATHOLOGY SUPEREXAM Page 73 of 99 For inquiries visit www.topnotchboardprep.com.ph or email us at [email protected]

TOPNOTCH MEDICAL BOARD PREP PATHOLOGY SUPEREXAM For inquiries visit www.topnotchboardprep.com.ph or email us at [email protected] Item # 539

540

541

542

QUESTION

AUTHOR

TOPNOTCH EXAM MIDTERM 1 EXAM - FEB 2013

A presence of necrosis with nuclear changes involving basophilia of the chromatin which later on fades or dissolves is called? A) Karyolysis B) Pyknosis C) Karyorrhexis D) Histiocytosis

Karyolysis>> basophilia of the chromatin fades or dissolves

MIGUEL RAFAEL RAMOS, MD (TOP 3 - FEB 2012 MED BOARDS; TOPNOTCH MD)

A 32 year old male was admitted due to fatigue, unexplained fever and spontaneous mucosal and cutaneous bleeding lasting for about 2 weeks. Laboratory work ups revelaed anemia, neutropenia and thrombocytopenia. Peripheral blood smear examination revealed a red staining peroxidase-positive structures with abnormal azurophilic granules. What is the possible diagnosis to this case? A) Acute Lymphoblastic Leukemia B) Acute Myelogenous Leukemia C) Chronic Lymphoblastic Leukemia D) Chronic Myelogenous Leukemia Which of the following describes a malignant condition? A. A small nodule of well-developed and organized pancreatic substance is found in the submucosa of the small intestine. B. Biopsy of a pulmonary "mass" reveals disorganized but histologically normal cartilage, bronchi and vessels C. A gelatinous mass is seen to fill the peritoneal cavity in a person known to have an "enlarged appendix." D. In chronic gastroesophageal reflux, squamous epithelium in the lower esophagus is replaced by glandular epithelium. E. None of the above A 67/M, a known case of lung cancer, came in due to easy fatigability and generalized weakness. Patient was noticeably pale. Your primary impression is anemia secondary to chronic disease. In this condition, one expects low levels of the following parameters except: A. Fe saturation B. Total iron binding capacity C. Transferrin D. Ferritin E. Serum iron

red-staining peroxidase-positive structure >> auer rods

MIGUEL RAFAEL RAMOS, MD (TOP 3 - FEB 2012 MED BOARDS; TOPNOTCH MD)

MIDTERM 1 EXAM - FEB 2013

A (choristoma), B (hamartoma), and D (metaplasia) describe nonmalignant conditions. C refers to pseudomyxoma peritonei; which occurs due to seeding of metastatic cells into the peritoneum from appendiceal carcinomas.

ABDELSIMAR OMAR II, MD (TOP 2 - AUG 2013 MED BOARDS; TOPNOTCH MD - 200 QUESTIONS) AND MARC DENVER TIONGSON, MD (40 QUESTIONS)

FINAL EXAM - FEB 2014

Ferritin is a soluble iron binding storage protein. It is decreased in IDA; but increased in sideroblastic anemia (iron overload disease) and in anemia of chronic disease / ACD (IL-1 and TNF-a induces its release.) Serum iron levels may differentiate IDA/ACD (decreased iron) from iron overload diseases (increased iron.) Total iron binding capacity correlates with transferrin levels. Whether transferrin increases or decreases may be predicted by what happens to ferritin since decreased ferritin stores increase synthesis of transferrin in the liver. Thus high ferritin -> low transferrin / TIBC; and vice versa. Fe saturation represents the percentage of binding sites on transferrin occupied by Fe. It is increased in Fe overload states; and decreased in IDA and ACD. History of breastfeeding and recent pregnancy suggest that the cystic mass is a galactocele. Without such a history, primary impression would be a macrocyst.

ABDELSIMAR OMAR II, MD (TOP 2 - AUG 2013 MED BOARDS; TOPNOTCH MD - 200 QUESTIONS) AND MARC DENVER TIONGSON, MD (40 QUESTIONS)

FINAL EXAM - FEB 2014

ABDELSIMAR OMAR II, MD (TOP 2 - AUG 2013 MED BOARDS; TOPNOTCH MD - 200 QUESTIONS) AND MARC DENVER TIONGSON, MD (40 QUESTIONS)

FINAL EXAM - FEB 2014

History of postmenopausal bleeding in patient with known risk factors (obesity, nulliparity) raises suspicion for endometrial cancer. Increased estrogen exposure increases risk. Coffee, smoking, OCP use, physical activity, and use of raloxifene are protective.

ABDELSIMAR OMAR II, MD (TOP 2 - AUG 2013 MED BOARDS; TOPNOTCH MD - 200 QUESTIONS) AND MARC DENVER TIONGSON, MD (40 QUESTIONS)

FINAL EXAM - FEB 2014

Onset of bloody stools, circulatory collapse and abdominal distension is typical of necrotizing enterocolitis. Prematurity is an important risk factor. Microscopically, mucosal or transmural coagulative necrosis is seen.

ABDELSIMAR OMAR II, MD (TOP 2 - AUG 2013 MED BOARDS; TOPNOTCH MD - 200 QUESTIONS) AND MARC DENVER TIONGSON, MD (40 QUESTIONS)

FINAL EXAM - FEB 2014

543

On physical examination, a cystic 3 x 3 cm breast mass was palpated on a 27 year old woman who just gave birth 6 months ago, and who had exclusively breastfed her child. What is your primary impression? A. Macrocyst B. Galactocele C. Mastitis D. Fibrocystic changes E. Breast abscess

544

A 66-year old diabetic, nulliparous woman complains of post-menopausal vaginal bleeding. Prior to menopause, which occurred at age 55, she had irregular menses. She denies the use of estrogen replacement therapy. Her examination is significant for obestity and hypertension. The following are risk factors for her condition except: A. Early age at menarche B. Late menopause C. Diabetes D. Smoking E. Obesity A pre-term infant born to an 18-year old primigravid developed abdominal distension and hypotension on the fourth day of life. You noted that the patient passed bloody stools. Abdominal radiographs demonstrate gas within the intestinal wall. Microscopic examination of the diseased bowel segment in this condition would reveal: A. Fat necrosis B. Coagulative necrosis C. Liquefactive necrosis D. Caseous necrosis

545

EXPLANATION

TOPNOTCH MEDICAL BOARD PREP PATHOLOGY SUPEREXAM Page 74 of 99 For inquiries visit www.topnotchboardprep.com.ph or email us at [email protected]

TOPNOTCH MEDICAL BOARD PREP PATHOLOGY SUPEREXAM For inquiries visit www.topnotchboardprep.com.ph or email us at [email protected] Item #

QUESTION

EXPLANATION

AUTHOR

TOPNOTCH EXAM

E. Apoptosis

546

547

548

549

550

A 28/M presents with a month-long history of low-grade fever. On PE, an apical systolic murmur is appreciated on auscultation; the Traube's space was obliterated; and hemorrhagic nontender lesions on the palms and soles were noted. Patient has an unremarkable past medical history. Blood culture woud most likely grow which infectious agent: A. Staphylococcus aureus B. Cardiobacterium sp. C. Streptococcus pyogenes D. Haemophilus influenzae E. Streptococcus viridans A 50/M came in with a chief complaint of dysphagia, heartburn and regurgitation of sour tasting fluid. He has weekly episodes of retching and vomiting after drinking sprees. On endoscopy the distal 3rd of his esophagus is seen to have tongues of red, velvety mucosa extending upward from the gastroesophageal junction alternating with residual smooth squamous mucosa and interfaces with light brown columnar mucosa distally. 6cm of the esophagus is affected. Upon biopsy of the area, histologic findings include abundant metaplastic goblet cells, atypical mitoses, nuclear hyperchromasia and failure of epithelial cells to mature as they reach the esophageal surface. What is his most likely diagnosis? A. Mallory-Weiss tear B. Reflux esophagitis C. Barrett esophagus D. Adenocarcinoma of the esophagus E. Squamous cell carcinoma of the esophagus A 7-year old boy presents to the ER with a 3day history of vomiting. A week prior to consult, the patient complained of fever and malaise; for which he was given Aspirin by his mother. On PE, the patient is tachycardic and tachypneic. Serum chemistries reveal elevated transaminases. The key pathologic finding in the liver of patients with this condition is: A. Microvesicular steatosis B. Concentric bile duct fibrosis C. Massive hepatocellular necrosis D. Piecemeal hepatocellular necrosis E. Portal bridging fibrosis A 45.F presents with a 4-day history of an acutely painful right sided goiter. She has no previous history of thyroid disease; and clinically appears euthyroid. Thyroid gland biopsy would reveal disruption of thyroid follicles with extravasation of colloid leading to a polymorphonuclear infiltrate. A granulomatous reaction may be seen. What is the most likely diagnosis? A. Subacute lymphocytic thyroiditis B. Hashimoto's thyroiditis C. DeQuervain's thyroiditis D. Grave's disease E. Colloid adenomatous goiter A 52-year old woman presents with a 4-month history of profuse watery diarrhea, weakness and dehydration. Testing reveals a serum calcium level of 11.4 mg/dL and a potassium level of 2.1 mEq/L. Abdominal CT done revealed a pancreatic mass. What is the most likely diagnosis? A. Glucagonoma B. Somatostatinoma C. Insulinoma D. Carcinoid tumor E. VIPoma

A classic case of infective endocarditis. The progression of symptoms over one month suggests subacute IE; most commonly due to less virulent organisms, most commonly, Streptococcus viridans.

ABDELSIMAR OMAR II, MD (TOP 2 - AUG 2013 MED BOARDS; TOPNOTCH MD - 200 QUESTIONS) AND MARC DENVER TIONGSON, MD (40 QUESTIONS)

FINAL EXAM - FEB 2014



ABDELSIMAR OMAR II, MD (TOP 2 - AUG 2013 MED BOARDS; TOPNOTCH MD - 200 QUESTIONS) AND MARC DENVER TIONGSON, MD (40 QUESTIONS)

FINAL EXAM - FEB 2014

Patient has Reye syndrome.

ABDELSIMAR OMAR II, MD (TOP 2 - AUG 2013 MED BOARDS; TOPNOTCH MD - 200 QUESTIONS) AND MARC DENVER TIONGSON, MD (40 QUESTIONS)

FINAL EXAM - FEB 2014

Remember the classic cases! Middle-aged woman with PAINLESS goiter and hypothyroid sx = Hashimoto's. Postpartal woman with PAINLESS goiter and signs of thyrotoxicosis = Subacute lymphocytic thyroiditis.

ABDELSIMAR OMAR II, MD (TOP 2 - AUG 2013 MED BOARDS; TOPNOTCH MD - 200 QUESTIONS) AND MARC DENVER TIONGSON, MD (40 QUESTIONS)

FINAL EXAM - FEB 2014

VIPoma (aka Verner Morrison syndrome) is a rare endocrine tumor, usually originating from the non islet cells of the pancreas that produces VIP. It is also known as the WDHA syndrome because massive amounts of VIP cause Watery Diarrhea with resultant Dehydrataion, Hypokalemia and Achlorydia. Patients also present with hypercalcemia hyperglycemia and metabolic acidosis.

ABDELSIMAR OMAR II, MD (TOP 2 - AUG 2013 MED BOARDS; TOPNOTCH MD - 200 QUESTIONS) AND MARC DENVER TIONGSON, MD (40 QUESTIONS)

FINAL EXAM - FEB 2014

TOPNOTCH MEDICAL BOARD PREP PATHOLOGY SUPEREXAM Page 75 of 99 For inquiries visit www.topnotchboardprep.com.ph or email us at [email protected]

TOPNOTCH MEDICAL BOARD PREP PATHOLOGY SUPEREXAM For inquiries visit www.topnotchboardprep.com.ph or email us at [email protected] Item # 551

552

553

554

555

QUESTION

EXPLANATION

AUTHOR

A 70/M, retired teacher, complains of multiple raised pigmented lesions over his back. Some of the lesions are pruritic. On PE, you note multiple roughened brown waxy lesions that appear "stuck on" over the patient's back. Biopsy of the lesion reveals sheets of small cells that resemble basal cells of normal epidermis; associated with the presence of small keratinfilled cysts. What is your primary impression? A. Actinic keratosis B. Seborrheic keratosis C. Malignant melanoma D. Basal cell carcinoma E. Squamous cell carcinoma A 35/M with a 10-pack year history of smoking comes in due to a cold painful fingertip with a beginning ulcer. You elicit a history of Raynaud phenomenon in the patient. On PE, you note decreased brachial, ulnar and radial pulses. What is your primary impression? A. Microscopic polyangitis B. Polyarteritis nodosa C. Churgg-Strauss syndrome D. Takayasu arteritis E. Thromboangiitis obliterans Urinalysis reveals increased urobilinogen, but absent urine bilirubin. This is consistent with which of the following conditions: A. Hereditary spherocytosis B. Hepatitis C. Pancreatic head malignancy D. Common bile duct obstruction E. Rotor syndrome

The histologic features of the lesion does NOT suggest malignancy, ruling out C, D and E. Seborrheic keratosis is a common, multiple, benign skin tumor; and appear as well-circumscribed brown plaques with a stuck on appearance. Actinic keratosis lesions on the other hand are skin colored, yellowish or erythematous illdefined irregular shaped scaly macules or plaques localized in sun-exposed areas of the body. Usually, a case of actinic keratoses would feature a 'farmer' or any other character with a history of chronic sun exposure.

ABDELSIMAR OMAR II, MD (TOP 2 - AUG 2013 MED BOARDS; TOPNOTCH MD - 200 QUESTIONS) AND MARC DENVER TIONGSON, MD (40 QUESTIONS)

Thromboangiitis obliterans or Buerger's disease is a non-atherosclerotic vascultiis resulting in segmental occlusion of medium-sized vessels. Patients are usuall young, male, with a strong history of smoking. It can affect the radial artery, presenting as recurrent Raynaud's phenomenon; or the tibial artery, presenting as instep claudication that persists even after exercise.

ABDELSIMAR OMAR II, MD (TOP 2 - AUG 2013 MED BOARDS; TOPNOTCH MD - 200 QUESTIONS) AND MARC DENVER TIONGSON, MD (40 QUESTIONS)

FINAL EXAM - FEB 2014

The point of the question is to differentiate between urobilinogen and urine bilirubin. Urobilinogen is formed by intestinal bacteria from conjugated bilirubin after it is secreted into the lumen; it is then reabsorbed by the enterohepatic circulation. Urine bilirubin, on the other hand, comes from conjugated bilirubin in the serum that is filtered into the urine. Recall that unconjugated bilirubin is NOT water- soluble and bound to albumin; and is not filtered. In obstructive jaundice (C and D), bile is not secreted into the intestinal lumen where bacteria converts conjugated bilirubin into urobilinogen. Thus, no urobinogen is found in the urine. In hepatitis, mixed bilirubinemia occurs (both unconjugated and conjugated bilirubin increase). Thus, we expect an increase in urine bilirubin due to increased amounts of conjugated bilirubin filtered by the kidney. In hemolytic anemia, unconjugated bilirubinemia predominates. Some of this excess bilirubin is secreted into the lumen where it is converted into urobilinogen. Urine bilirubin on the other hand is decreased, since most of the serum bilirubin in hemolytic anemia is UNCONJUGATED and not filtered by the kidneys.

ABDELSIMAR OMAR II, MD (TOP 2 - AUG 2013 MED BOARDS; TOPNOTCH MD - 200 QUESTIONS) AND MARC DENVER TIONGSON, MD (40 QUESTIONS)

FINAL EXAM - FEB 2014

ABDELSIMAR OMAR II, MD (TOP 2 - AUG 2013 MED BOARDS; TOPNOTCH MD - 200 QUESTIONS) AND MARC DENVER TIONGSON, MD (40 QUESTIONS)

FINAL EXAM - FEB 2014

Medulloblastoma occurs predominantly in children and exclusively in the cerebellum. Histology reveals extremely cellular tumor with sheets of anaplastic small blue cells. Another commonly occuring cerebellar tumor in childen in plemorphic astrocytoma; but this benign condition would not usually present with necrosis and mitoses on histology.

ABDELSIMAR OMAR II, MD (TOP 2 - AUG 2013 MED BOARDS; TOPNOTCH MD - 200 QUESTIONS) AND MARC DENVER TIONGSON, MD (40 QUESTIONS)

FINAL EXAM - FEB 2014

A 60/M presents due to gross hematuria. You elicit a 3-month history of right-sided flank discomfort and abdominal fullness. Vital signs are as follows: BP: 170/100, HR: 122, RR: 22, T: afeb. On PE, you palpate a right-sided lower abdominal mass. You ordered an abdominal CT; the findings of which are consistent with renal cell carcinoma. Renal cell carcinoma is MOST COMMONLY derived from cells of the: A. Glomerulus B. Proximal tubule C. Loop of Henle D. Distal tubule E. Collecting tubule A 7-year old boy presents with a 5-day history of gradually worsening headaches and intermittent vomiting without fever. Patient was also noted to be walking with poor balance and coordination. MRI done revealed a cerebellar mass. Patient subsequently underwent excision of the tumor. Biopsy reveals that the tumor is extremely cellular, with sheets of anaplastic ("small blue") cells. Individual tumor cell are small, with little cytoplasm and hyperchromatic nuclei. Mitotic figures are abundant. What is the diagnosis? A. Pilocytic astrocytoma B. Pleomorphic xanthoastrocytoma C. Medulloblastoma D. Craniopharyngioma E. Glioblastoma multiforme

TOPNOTCH MEDICAL BOARD PREP PATHOLOGY SUPEREXAM Page 76 of 99 For inquiries visit www.topnotchboardprep.com.ph or email us at [email protected]

TOPNOTCH EXAM FINAL EXAM - FEB 2014

TOPNOTCH MEDICAL BOARD PREP PATHOLOGY SUPEREXAM For inquiries visit www.topnotchboardprep.com.ph or email us at [email protected] Item # 556

QUESTION

EXPLANATION

AUTHOR

Your 80-year old lola has become increasingly forgetful. She is no longer able to help in household tasks. She later become more irritable. Systemic physical examination and routine work-up is unremarkable. Her minimental status examination (MMSE) score is 20/30; there are no other significant neurologic examination findings. Her condition is characterized by which of the following histologic features: A. Neurofibrillary tangles B. Pick bodies C. Lewy neurites D. Fibrillary gliosis E. Rosenthal fibers A 70/M is evaluated for a heart murmur. On PE, a 3/6 systolic ejection murmur radiating to the neck is heard on auscultation. Echocardiogram shows aortic stenosis and normal systolic function. Which of the following cellular changes is BEST demonstrated in this condition? A. Atrophy B. Coagulative necrosis C. Hypertrophy D. Dystrophic calcification E. Metastatic calcification On the 28th day of the menstrual cycle, a woman undergoes menstrual bleeding lasting 2-4 days consuming 3 pads per day. Which of the following cellular changes is BEST demonstrated in this condition? A. Atrophy B. Apoptosis C. Coagulative necrosis D. Liquefactive necrosis E. Hypertrophy

At the microscopic level, Alzheimer's Disease is diagnoed by the prsence of plaques and neurofibrillary tangles. Lewy bodies are seen in Parkinson's disease; Pick bodies are seen in Pick disease, a subtype of Frontotemporal lobar degeneration. (In frontotemporal lobar degeneration, behavior changes precede memory disturbances which assist in their separation from AD on clinical grounds.) Rosenthal fibers are features of pilocytic astrocytoma.

ABDELSIMAR OMAR II, MD (TOP 2 - AUG 2013 MED BOARDS; TOPNOTCH MD - 200 QUESTIONS) AND MARC DENVER TIONGSON, MD (40 QUESTIONS)

"Dystrophic calcification of the aortic valces is an important cause of aortic stenosis in elderly persons." Basic Patho 9e p.25

ABDELSIMAR OMAR II, MD (TOP 2 - AUG 2013 MED BOARDS; TOPNOTCH MD - 200 QUESTIONS) AND MARC DENVER TIONGSON, MD (40 QUESTIONS)

FINAL EXAM - FEB 2014

Menses is an example of apoptosis or programmed cell death.

ABDELSIMAR OMAR II, MD (TOP 2 - AUG 2013 MED BOARDS; TOPNOTCH MD - 200 QUESTIONS) AND MARC DENVER TIONGSON, MD (40 QUESTIONS)

FINAL EXAM - FEB 2014

559

A 45/M has been drinking alcoholic beverages heavily for the past 10 years. The following are laboratory findings expected in alcohol abuse except: A. Hyperuricemia B. AST>ALT C. Increased GGT D. Hypertriglyceridemia E. Fasting hyperglycemia

ABDELSIMAR OMAR II, MD (TOP 2 - AUG 2013 MED BOARDS; TOPNOTCH MD - 200 QUESTIONS) AND MARC DENVER TIONGSON, MD (40 QUESTIONS)

FINAL EXAM - FEB 2014

560

A 30/M, call-center agent, known case of HIVAIDS, presents with progressive generalized swelling. On PE, patient has non-pitting edema over the lower extremities extending up to the mid-abdomen. Edema was refractory to steroid therapy. Renal biopsy done revealed increased mesangial matrix, obliterated capillary lumina, and deposition of hyaline masses and lipid droplets in affected glomeruli. Immunofluorescence microscopy revealed trapping of immunoglobulins and complement in areas of hyalinosis. On electron microscopy, the podocytes exhibit effacement of foot processes. What is the most likely diagnosis? A. Membranous nephropathy B. Minimal change disease C. Focal segmental glomerulosclerosis D. Membranoproliferative glomerulonephritis E. IgA nephropathy Drugs can cause alteration in both acute and chronic alterations in respiratory structure and function. Among these drugs that cause pulmonary disease is nitrofurantoin which is associated with: a. interstitial fibrosis b. hypersensitivity pneumonitis c. bronchiolitis obliterans d. eosinophilic pneumonia

Hyperuricemia occurs in chronic alcohol use because lactic acid and beta-hydroxybutyrate compete with uric acid for excretion in the proximal tubules. Alcohol is a mitochondrial toxin that causes release of ALT, located in the mitochondria. It also induces hyperplasia of SER causing increased synthesis of GGT. Increased levels of NADH also lead to the conversion of DHAP into G3P which is used as a substrate for triglyceride synthesis in the liver. Fasting HYPOglycemia occurs because excess NADH causes pyruvate (the substrate for gluconeogenesis) to be converted to lactate. Remember that FSGS is the predominant glomerular lesion in patients with HIV-associated nephropathy.

ABDELSIMAR OMAR II, MD (TOP 2 - AUG 2013 MED BOARDS; TOPNOTCH MD - 200 QUESTIONS) AND MARC DENVER TIONGSON, MD (40 QUESTIONS)

FINAL EXAM - FEB 2014

Hypersensitivity pneumonitis. Table 15-7 of Robbins

BLAKE WARREN ANG, MD (TOP 1 - AUG 2013 MED BOARDS; TOPNOTCH MD)

MIDTERM 2 EXAM - FEB 2014

557

558

561

TOPNOTCH MEDICAL BOARD PREP PATHOLOGY SUPEREXAM Page 77 of 99 For inquiries visit www.topnotchboardprep.com.ph or email us at [email protected]

TOPNOTCH EXAM FINAL EXAM - FEB 2014

TOPNOTCH MEDICAL BOARD PREP PATHOLOGY SUPEREXAM For inquiries visit www.topnotchboardprep.com.ph or email us at [email protected] Item # 562

QUESTION

EXPLANATION

AUTHOR

Certain pulmonary entities exhibit a histologic picture of eosinophilic infiltration. Which of the following disease is characterized by a benign clinical course? a. Acute eosinophilic pneumonia b. Loeffler syndrome c. Tropical eosinophilia d. Chronic eosinophilic pneumonia

Loeffler syndrome or Simple pulmonary eosinophilia is characterized by transient pulmonary lesions, blood eosinophilia and a benign clinical course.

BLAKE WARREN ANG, MD (TOP 1 - AUG 2013 MED BOARDS; TOPNOTCH MD)

563

Patient presented with fever, headache, muscle aches, leg pains and cough. PE did not show findings of consolidation. Histologic pattern of the disease show a predominant interstitial nature of inflammatory reaction virtually localized within the walls of the alveoli. What is the most probable causative agent? a.) Klebsiella pneumonia b.) Streptococcus pyogenes c.) Mycoplasma pneumoniae d.) Staphylococcus aureus

Atypical pneumonia include viral pneumonias and mycoplasma pneumonia. P714 Robbins.

BLAKE WARREN ANG, MD (TOP 1 - AUG 2013 MED BOARDS; TOPNOTCH MD)

MIDTERM 2 EXAM - FEB 2014

564

Fleur de lis pattern is characteristic of which infectious agent? a. Pseudomonas aeruginosa b. Mycoplasma pneumonia c. Streptococcus pyogenes d. Staphylococcus aureus



BLAKE WARREN ANG, MD (TOP 1 - AUG 2013 MED BOARDS; TOPNOTCH MD)

MIDTERM 2 EXAM - FEB 2014

565

Which of the following antibodies can cross the placental circulation? a. IgA b. IgG2 c. IgE d. IgG4

IgG4. IgG2 is the only subclass of IgG that may be unable cross the placenta. Source: Henrys Diagnostics

BLAKE WARREN ANG, MD (TOP 1 - AUG 2013 MED BOARDS; TOPNOTCH MD)

MIDTERM 2 EXAM - FEB 2014

566

10 month old male presented with paralytic poliomyelitis after given an oral polio vaccine. What is the most probable underlying etiology? a. Severe combined immune deficiency syndrome b. Bruton’s Aggamaglobulinemia c. DiGeorge Syndrome d. Isolated IgA deficiency

Bruton’s /x-linked aggamaglobulinemia can predispose to paralytic poliomyelitis after live poliovirus immunization. Robbins p232

BLAKE WARREN ANG, MD (TOP 1 - AUG 2013 MED BOARDS; TOPNOTCH MD)

MIDTERM 2 EXAM - FEB 2014

567

10 month old male patient had recurrent bouts of infection with Haemophilus and Staphylococcus aureus. He also had 3 episodes of Giardiasis since 6 months of age. What is the most probable defect? a. Complement b. B cells c. T cell d. NK cells



BLAKE WARREN ANG, MD (TOP 1 - AUG 2013 MED BOARDS; TOPNOTCH MD)

MIDTERM 2 EXAM - FEB 2014

568

15 year old patient presented with recurrent sinopulmonary infections of Haemophilus or Streptococcal origin. He has had recurrent bouts diarrhea despite treatment. What is the expected histologic picture of the patient? a. Decrease number of B cells in lymphoid tissues and blood. b. Decrease number of B cells in the lymphoid follicles of spleen, liver and gut. c. Normal or near-normal numbers of B cells in the blood and lymphoid tissues. d. Increase numbers of B cells in the peripheral circulation. Helicobacter pylori is associated with blood type O individuals. The reason for this lies in the organisms capacity to bind to certain antigens expressed on the surface of cells like H antigen of the ABO blood group systems and the antigens of which other blood group system? a. Lutheran b. Lewis c. Kell d. Rh blood group

Answer is C as per Robbins p233

BLAKE WARREN ANG, MD (TOP 1 - AUG 2013 MED BOARDS; TOPNOTCH MD)

MIDTERM 2 EXAM - FEB 2014

Lewis as per Henry’s diagnostics

BLAKE WARREN ANG, MD (TOP 1 - AUG 2013 MED BOARDS; TOPNOTCH MD)

MIDTERM 2 EXAM - FEB 2014

569

TOPNOTCH MEDICAL BOARD PREP PATHOLOGY SUPEREXAM Page 78 of 99 For inquiries visit www.topnotchboardprep.com.ph or email us at [email protected]

TOPNOTCH EXAM MIDTERM 2 EXAM - FEB 2014

TOPNOTCH MEDICAL BOARD PREP PATHOLOGY SUPEREXAM For inquiries visit www.topnotchboardprep.com.ph or email us at [email protected] Item # 570

QUESTION

EXPLANATION

AUTHOR

TOPNOTCH EXAM MIDTERM 2 EXAM - FEB 2014

What is the average weight of the bone marrow of an adult individual? a. 1000 grams b. 1500 grams c.2000 grams d. 1750 grams

1200-1500 grams as per Henry’s diagnostics

BLAKE WARREN ANG, MD (TOP 1 - AUG 2013 MED BOARDS; TOPNOTCH MD)

571

Caspases are noted for their cysteine content and the ability to cleave aspartic acid residues. They are hence involve in programmed cell death. Which caspase acts as the initiator caspase in humans under the extrinsic pathway? a. Caspase 8 b. Caspase 9 c. Caspase 10 d. Smac/DIABLO

caspase 8 in the nematode C. elegans but Caspase 10 in humans p30 robbins

BLAKE WARREN ANG, MD (TOP 1 - AUG 2013 MED BOARDS; TOPNOTCH MD)

MIDTERM 2 EXAM - FEB 2014

572

40 year old male has had recurrent episodes of epigastric pain for the past 3 weeks, relieved by eating. He had an endoscopy with biopsy done. Which of the following is true regarding the expected histologic picture of the gastric biopsy specimen? a. Intraepithelial neutrophils and subepithelial plasma cells are characteristic of H. pylori gastritis b. Although there is a good concordance between colonization of the antrum and cardia, infection of the cardia occurs at somewhat higher rates. c. H. pylori shows tropism to gastric epithelia and is generally generally found in association with gastric intestinal metaplasia or duodenal epithelium. d. Atrophic gastric mucosa with lymphoid aggregates can be seen. Which of the following is not found the mesangium of a Post-Infectious Glomerulonephritis? a. IgG b.IgM c. IgA d. Complement

A as per robbins p778 in the setting of an acute H pylori gastritis. D occurs in the setting of chronic H. pylori gastritis. Infection of the cardia occur at lower rates and H. pylori generally does not colonize duodenal epithelium.

BLAKE WARREN ANG, MD (TOP 1 - AUG 2013 MED BOARDS; TOPNOTCH MD)

MIDTERM 2 EXAM - FEB 2014

IgA is not found in PSGN but is more prominent in Buerger’s disease.

BLAKE WARREN ANG, MD (TOP 1 - AUG 2013 MED BOARDS; TOPNOTCH MD)

MIDTERM 2 EXAM - FEB 2014

574

Which of the Gatrointestinal Polyposis syndromes does not have a hereditary component? a. Peutz-Jegher’s syndrome b. Juvenile Polyposis c. Cowden Syndrome d. Cronkhite-Canada syndrome

A is associated with an LKB1/STK11 mutation. Juvenile polyposis is associated with a mutation in the SMAD4 gene. Cowden with PTEN. Cronkhite-Canada syndrome makes its distinction by being nonhereditary.

BLAKE WARREN ANG, MD (TOP 1 - AUG 2013 MED BOARDS; TOPNOTCH MD)

MIDTERM 2 EXAM - FEB 2014

575

Most common site of AV malformation in the brain? a. ACA b. MCA c. PCA d. Basilar artery

MCA as per robbins p1299

BLAKE WARREN ANG, MD (TOP 1 - AUG 2013 MED BOARDS; TOPNOTCH MD)

MIDTERM 2 EXAM - FEB 2014

576

Flexner-wintersteiner rosettes is associated with a. Neuroblastoma b. Retinoblastoma c. Medulloblastoma d. Glioblastoma



BLAKE WARREN ANG, MD (TOP 1 - AUG 2013 MED BOARDS; TOPNOTCH MD)

MIDTERM 2 EXAM - FEB 2014

577

70 year old female presented with a slow growing breast mass with a rubbery consistency. Histologic picture show islands of cells in pale gray-blue gelatin. What is the most probable tumor? a. Medullary breast cancer b. Ductal carcinoma c. Colloid Carcinoma d. Papillary breast CA

Colloid CA as per robbins

BLAKE WARREN ANG, MD (TOP 1 - AUG 2013 MED BOARDS; TOPNOTCH MD)

MIDTERM 2 EXAM - FEB 2014

578

Which of the following is not associated with advancing age in the human heart? a. sigmoid septum b. mitral valve prolapse c. Lipofuscin deposistion d. Lambl excresences

all are associated with aging except MVP robbins p532

BLAKE WARREN ANG, MD (TOP 1 - AUG 2013 MED BOARDS; TOPNOTCH MD)

MIDTERM 2 EXAM - FEB 2014

573

TOPNOTCH MEDICAL BOARD PREP PATHOLOGY SUPEREXAM Page 79 of 99 For inquiries visit www.topnotchboardprep.com.ph or email us at [email protected]

TOPNOTCH MEDICAL BOARD PREP PATHOLOGY SUPEREXAM For inquiries visit www.topnotchboardprep.com.ph or email us at [email protected] Item # 579

QUESTION

EXPLANATION

AUTHOR

TOPNOTCH EXAM MIDTERM 2 EXAM - FEB 2014

Cardiac hypertrophy is noted with ventricular wall thickness of about: a. 0.5cm b. 1cm c. 1.5cmcm d. 2cm

2cm; normal thickness is 1-1.5cm

BLAKE WARREN ANG, MD (TOP 1 - AUG 2013 MED BOARDS; TOPNOTCH MD)

580

Intestinal metaplasia of the stomach raises a predisposition towards adenocarcinoma development. Which cell most likely found confirms intestinal metaplasia? a. Mucous cells b. Goblet cells c. Isolated islands of Brunner glands d. Simple columnar epithelia

Goblet cells and intestinal columnar absorptive cells when present points to intestinal metaplasia

BLAKE WARREN ANG, MD (TOP 1 - AUG 2013 MED BOARDS; TOPNOTCH MD)

MIDTERM 2 EXAM - FEB 2014

581

A 7 year old male was admitted due to easy fatigability, fever, and easy bruisability for 2 months. CBC done showed the following: Hgb=94 mg/L, Hct=0.29, Plt=54, WBC=45.50, Segmenters=0.10, Lymphocytes=0.66, Metamyelocyte=0.04, Blast=0.16, Myelocyte=0.04. Bone marrow biopsy showed hypercellular marrow with cells which have scant cytoplasm with nuclei containing coarse and clumped chromatin. Immunostaining showed (-) myeloperoxidase and (+) periodic acid-Schiff cytoplasmic material. What is the most likely diagnosis? A. Acute lymphoblastic leukemia B. Acute myeloblastic leukemia C. Chronic lymphocytic leukemia D. Chronic myelogenous leukemia E. Multiple myeloma A 30-year-old woman presents with malaise and increasing fatigue. On physical examination, she has anicteric sclerae and jaundice, and laboratory evaluation finds elevated AST and ALT. Liver biopsy showed shrunken hepatocytes with highly eosinophilc cytoplasm and fragmented nuclei. There were no noted inflammatory infiltrates. Which of the following terms best describes this process occurring in the hepatocytes? A. Apoptosis B. Autophagy C. Heterophagy D. Necrosis E. Pinocytosis A 60-year-old man died secondary to coronary artery disease. During autopsy, the lumen of the coronary arteries was covered with fibrous plaques and an area with ruptured plaque and superimposed thrombosis. Sections from these abnormal areas revealed a fibrous cap and a central core (largely lipid) core with an area showing disruption of the fibrous cap with thrombosis. Which one of the following substances promotes atherosclerosis by stimulating smooth-muscle cells to migration and proliferation in sites of endothelial injury? A. α-interferon (α-INF) B. β-transforming growth factor (βTGF) C. Interleukin-1 (IL1) D. Platelet derived growth factor (PDGF) E. Vascular endothelial growth factor (VEGF) A 56-year old post-gastrectomy patient consults due to pallor and easy fatigability. CBC done showed anemia. Peripheral smear showed large, egg-shaped macro-ovalocytes and hypersegmented neutrophils. Patient presents with? A. Sideroblastic anemia B. Iron deficiency anemia C. Anemia of chronic disease D. Aplastic anemia E. Megaloblastic anemia

By definition, in ALL, blasts compose more than 25% of the marrow cellularity. The nuclei of lymphoblasts in Wright-Giemsa-stained preparations have somewhat coarse and clumped chromatin and one or two nucleoli; myeloblasts tend to have finer chromatin and more cytoplasm, which may contain granules. The cytoplasm of lymphoblasts often contains large aggregates of periodic acid-Schiff-positive material, whereas myeloblasts are often peroxidase positive.

TIMOTHY TANG LEE SAY, MD (TOP 4 - AUG 2013 MED BOARDS; TOPNOTCH MD)

MIDTERM 1 EXAM - FEB 2014

Apoptosis involves single cells, not large groups of cells, and with apoptosis the cells shrink and there is increased eosinophilia of cytoplasm. The shrunken apoptotic cells form apoptotic bodies, which may be engulfed by adjacent cells or macrophages. With apoptosis there is no inflammatory response, the cell membranes do not rupture, and there is no release of macromolecules

TIMOTHY TANG LEE SAY, MD (TOP 4 - AUG 2013 MED BOARDS; TOPNOTCH MD)

MIDTERM 1 EXAM - FEB 2014

The pathogenesis of atherosclerosis depends in part on the inflammatory function of macrophages, which involves the release of numerous cytokines. Plateletderived growth factor (PDGF) is mitogenic and chemotactic for smooth-muscle cells. This may explain the recruitment and proliferation of smooth-muscle cells in atherosclerosis.

TIMOTHY TANG LEE SAY, MD (TOP 4 - AUG 2013 MED BOARDS; TOPNOTCH MD)

MIDTERM 1 EXAM - FEB 2014

In the peripheral blood the earliest change is usually the appearance of hypersegmented neutrophils, which appear even before the onset of anemia. Normally, neutrophils have three or four nuclear lobes, but in megaloblastic anemias neutrophils often have five or more. The red cells typically include large, egg-shaped macro-ovalocytes

TIMOTHY TANG LEE SAY, MD (TOP 4 - AUG 2013 MED BOARDS; TOPNOTCH MD)

MIDTERM 1 EXAM - FEB 2014

582

583

584

TOPNOTCH MEDICAL BOARD PREP PATHOLOGY SUPEREXAM Page 80 of 99 For inquiries visit www.topnotchboardprep.com.ph or email us at [email protected]

TOPNOTCH MEDICAL BOARD PREP PATHOLOGY SUPEREXAM For inquiries visit www.topnotchboardprep.com.ph or email us at [email protected] Item # 585

586

587

588

589

QUESTION A 40-year old consulted due to a slowly enlarging, soft, painless, pre-auricular mass. Surgical excision of the mass was done. Microscopically, the mass was composed of small, dark epithelial cells intermingled with a loose, myxoid connective tissue stroma with islands and strands of myoepithelial cells. These findings are suggestive of? A. Warthin tumor B. Pleomorphic adenoma C. Mucoepidermoid carcinoma D. Myoepithelioma E. Adenoid cystic carcinoma A 46-year old female came in due to prolonged heavy menses. Dilatation and curettage was done. Endometrial scrapings revealed an increase in the number and size of endometrial glands, marked gland crowding and branching (back-to-back appearance) with little intervening stroma. The epithelial cells remain cytologically normal. This is suggestive of? A. Endometrial polyp B. Leiomyoma C. Endometriosis D. Endometrial hyperplasia E. Endometrial carcinoma A 10-year old male complained of a painful, enlarging mass on the right femur. X-ray of the affected area showed layers of bone deposited in an onion-skin fashion. Surgical excision was done. Microscopic examination of the mass showed sheets of uniform, round cells with small amounts of clear cytoplasm with few mitoses and little intervening cytoplasm. What is the diagnosis? A. Osteosarcoma B. Chondroblastoma C. Ewing sarcoma D. Chondrosarcoma E. Rhabdomyosarcoma

A 50-year old male consulted due to recurrent left ankle pain and swelling relieved by intake of NSAIDS. Arthrocentesis was done and microscopic examination of the synovial fluid showed long, slender, needle-shaped crystals which are negatively bifringent. This is indicative of? A. Gouty arthritis B. Calcium pyrophosphate deposition disease C. Viral arthritis D. Psoriatic arthritis E. Pseudogouty arthritis An 80-year old female was noted to have insidious onset of memory loss accompanied by alterations in mood and behavior, and impairment in higher level of intellectual function. There were no noted symptoms of ataxia, language disturbances or alterations in personality. This disease entity is differentiated from other causes of dementia due to the presence of? A. Cytoplasmic round to oval filamentous inclusion bodies that stain strongly with silver (Pick Bodies) B. Tau-containing neurofibrillary tangles C. Ballooned neurons positive for phosphorylated neurofilaments D. Patchy perivacuolar or plaque type patterns of misfolded protein deposition E. Spherical collections of dilated, tortuous, silver-staining neuritic processes (dystrophic neurites), often around a central amyloid core (neuritic/senile plaques)

EXPLANATION

AUTHOR

TOPNOTCH EXAM MIDTERM 1 EXAM - FEB 2014

The most common neoplasm of the parotid gland is the pleomorphic adenoma (mixed tumor), which histologically reveals epithelial structures embedded within a mesenchyme-like stroma consisting of mucoid, myxoid, or chondroid tissue.

TIMOTHY TANG LEE SAY, MD (TOP 4 - AUG 2013 MED BOARDS; TOPNOTCH MD)

Most commonly, the prolonged unremitting estrogen stimulation results in endometrial hyperplasia. World Health Organization (WHO) classification takes into account both the architectural and cytologic features, in the sense of dividing the hyperplasias into simple and complex on the basis of the architecture, and subdividing each into typical and atypical on the basis of their cytology. Endometrial hyperplasia is most commonly seen during the perimenopausal period.

TIMOTHY TANG LEE SAY, MD (TOP 4 - AUG 2013 MED BOARDS; TOPNOTCH MD)

MIDTERM 1 EXAM - FEB 2014

Ewing sarcoma and PNETs arise in the medullary cavity and invade the cortex and periosteum to produce a soft tissue mass. The tumor is tan-white, frequently with hemorrhage and necrosis. It is composed of sheets of uniform small, round cells that are slightly larger than lymphocytes with few mitoses and little intervening stroma. Ewing sarcoma and PNETs typically present as painful enlarging masses in the diaphyses of long tubular bones (especially the femur) and the pelvic flat bones. Some patients have systemic signs and symptoms, including fever, elevated erythrocyte sedimentation rate, anemia, and leukocytosis that can mimic infection. X-rays show a destructive lytic tumor with infiltrative margins and extension into surrounding soft tissues. There is a characteristic periosteal reaction depositing bone in an onionskin fashion. The major morphologic manifestations of gout are acute arthritis, chronic tophaceous arthritis, tophi in various sites, and gouty nephropathy. Acute arthritis is characterized by a dense neutrophilic infiltrate permeating the synovium and synovial fluid. Long, slender, needle-shaped monosodium urate crystals are frequently found in the cytoplasm of the neutrophils as well as in small clusters in the synovium. The synovium is edematous and congested, and contains scattered mononuclear inflammatory cells. When the episode of crystallization abates and the crystals resolubilize, the attack remits At the microscopic level, Alzheimer disease is diagnosed by the presence of plaques (a type of extracellular lesion); and neurofibrillary tangles (a type of intracellular lesion). Neuritic plaques are focal, spherical collections of dilated, tortuous, silverstaining neuritic processes (dystrophic neurites), often around a central amyloid core.

TIMOTHY TANG LEE SAY, MD (TOP 4 - AUG 2013 MED BOARDS; TOPNOTCH MD)

MIDTERM 1 EXAM - FEB 2014

TIMOTHY TANG LEE SAY, MD (TOP 4 - AUG 2013 MED BOARDS; TOPNOTCH MD)

MIDTERM 1 EXAM - FEB 2014

TIMOTHY TANG LEE SAY, MD (TOP 4 - AUG 2013 MED BOARDS; TOPNOTCH MD)

MIDTERM 1 EXAM - FEB 2014

TOPNOTCH MEDICAL BOARD PREP PATHOLOGY SUPEREXAM Page 81 of 99 For inquiries visit www.topnotchboardprep.com.ph or email us at [email protected]

TOPNOTCH MEDICAL BOARD PREP PATHOLOGY SUPEREXAM For inquiries visit www.topnotchboardprep.com.ph or email us at [email protected] Item # 590

591

592

593

594

595

QUESTION

EXPLANATION

AUTHOR

A 20-year old female complained of multiple bullae and vesicles on the face, axilla, and trunk. The lesions were noted to rupture easily that leaves shallow erosions. This disorder is caused by autoantibodies that result in dissolution of intercellular attachments within the epidermis and mucosal epithelium. The autoantibodies are directed against? A. Desmoglein B. Hemidesmosomes C. Reticulin D. Laminin E. Keratin A 6-year old female was noted to have bipedal edema. Urinalysis done showed no hematuria and massive proteinuria. Blood pressure was normal. There was a rapid response in the disease after administration of steroids. Electron microscopy will show? A. Thickened GBM, and double contour or tram-track appearance of glomerular capillary wall B. Irregular thickening of the GBM, lamination of lamina densa, and foci rarefraction C. Diffuse effacement of foot processes of podocytes, normal GBM, and absence of deposits D. Diffuse effacement of foot processes of podocytes and denudation of underlying GBM E. Thickening of the GBM with effacement of foot processes and subendothelial deposits A 16-year old female complains of a midline anterior neck mass. The mass was 2 cm in diameter, soft, painless and moves when tongue is protruded. Excision of the mass was done and histology showed ducts and cysts lined by stratified squamous epithelium with adjacent lymphocytic infiltrate. This suggest? A. Brachial pouch cyst B. Thyroglossal duct cyst C. Epidermal inclusion cyst D. Thyroid cyst E. Sebaceous cyst A 50-year old female, menopause for 4 years will have a cytohormonal maturity index of? A. Parabasal=0, Intermediate=60, Superficial Cells=40 B. Parabasal=20, Intermediate=80, Superficial Cells=0 C. Parabasal=20, Intermediate=40, Superficial Cells=40 D. Parabasal=80, Intermediate=10, Superficial Cells=10 E. Parabasal=20, Intermediate=0, Superficial Cells=80 A 23-year old medical student who had overnight binge drinking suddenly developed boring abdominal pain relieved by doubling up. Serum amylase and lipase were elevated. What type of necrosis occurs when pancreatic lipases were released in the substance of the pancreas and peritoneal cavity? A. Liquefactive necrosis B. Coagulative necrosis C. Caseous necrosis D. Fat necrosis E. Gangrenous necrosis A 30-year old male had recurrent diarrhea, crampy abdominal pain, and fever for 3 months. Colonoscopy done showed areas of long serpentine linear ulcers oriented along the axis of the bowel with surrounding mucosal sparing. Biopsy of the lesion will show? A. Mucosal ulceration and mononuclear inflammatory infiltrates in lamina propia B. Transmural involvement of the bowel by inflammatory process with mucosal damage, non-casseating granuloma and fissuring with formation of fistula C. Submucosal fibrin and mucosal architectural disarray D. Inflammatory infiltrates predominantly lymphocytic without granuloma formation E. Submucosal polyps with underlying plasmacytic infiltrates

In pemphigus vulgaris, acantholysis selectively involves the layer of cells immediately above the basal cell layer, giving rise to a suprabasal acantholytic blister. It is caused by a type II hypersensitivity reaction. Patient sera contain pathogenic IgG antibodies to intercellular desmosomal proteins (desmoglein types 1 and 3) of skin and mucous membranes. The distribution of these proteins within the epidermis determines the location of the lesions.

TIMOTHY TANG LEE SAY, MD (TOP 4 - AUG 2013 MED BOARDS; TOPNOTCH MD)

This relatively benign disorder is the most frequent cause of the nephrotic syndrome in children. It is characterized by glomeruli that have a normal appearance by light microscopy but show diffuse effacement of podocyte foot processes when viewed with the electron microscope. Even with the electron microscope, the GBM appears normal. The only obvious glomerular abnormality is the uniform and diffuse effacement of the foot processes of the podocytes. The cytoplasm of the podocytes thus appears flattened over the external aspect of the GBM, obliterating the network of arcades between the podocytes and the GBM. There are also epithelial cell vacuolization, microvillus formation, and occasional focal detachments. When the changes in the podocytes reverse (e.g., in response to corticosteroids), the proteinuria remits.

TIMOTHY TANG LEE SAY, MD (TOP 4 - AUG 2013 MED BOARDS; TOPNOTCH MD)

MIDTERM 1 EXAM - FEB 2014

This is a congenital anomaly of the thyroid. A persistent sinus tract may remain as a vestigial remnant of the tubular development of the thyroid gland. Parts of this tube may be obliterated, leaving segments to form cysts. Segments of the duct and cysts that occur in the neck are covered by stratified squamous epithelium identical to the tongue in the region of foramen cecum. Anomalies in the lower neck area more proximal to the thyroid gland are lined by epithelium resembling the thyroid acinar epithelium. Adjacent to the lining epithelium are lymphocytic infiltrates. Intermediate cells are responsive to progesterone and superficial cells are responsive to estrogen. At the onset of menopause, the low levels of estrogen and progesterone will favor proliferation of parabasal cells making it the most predominant cell type on Pap smear.

TIMOTHY TANG LEE SAY, MD (TOP 4 - AUG 2013 MED BOARDS; TOPNOTCH MD)

MIDTERM 1 EXAM - FEB 2014

TIMOTHY TANG LEE SAY, MD (TOP 4 - AUG 2013 MED BOARDS; TOPNOTCH MD)

MIDTERM 1 EXAM - FEB 2014

Fat necrosis refers to focal areas of fat destruction, typically resultingfrom release of activated pancreatic lipases into the substance of the pancreas and peritoneal cavity. On histologic examination, the necrosis takes the form of foci of shadowy outlines of necrotic fat cells, with basophilic calcium deposits, surrounded by an inflammatory reaction.

TIMOTHY TANG LEE SAY, MD (TOP 4 - AUG 2013 MED BOARDS; TOPNOTCH MD)

MIDTERM 1 EXAM - FEB 2014

Crohn disease is characterized by (1) sharply delimited and typically transmural involvement of the bowel by an inflammatory process with mucosal damage, (2) the presence of noncaseating granulomas in 40% to 60% of cases, and (3) fissuring with formation of fistulae.

TIMOTHY TANG LEE SAY, MD (TOP 4 - AUG 2013 MED BOARDS; TOPNOTCH MD)

MIDTERM 1 EXAM - FEB 2014

TOPNOTCH MEDICAL BOARD PREP PATHOLOGY SUPEREXAM Page 82 of 99 For inquiries visit www.topnotchboardprep.com.ph or email us at [email protected]

TOPNOTCH EXAM MIDTERM 1 EXAM - FEB 2014

TOPNOTCH MEDICAL BOARD PREP PATHOLOGY SUPEREXAM For inquiries visit www.topnotchboardprep.com.ph or email us at [email protected] Item # 596

597

598

599

600

601

QUESTION

EXPLANATION

AUTHOR

A 76-year old male consulted due to urinary frequency, dribbling, and sensation of incomplete voiding. Ultrasound of kidney, ureter, bladder and prostate showed a nodularly enlarged prostate with microcalcifications. PSA was 20 ng/mL. Prostatectomy was done and histolopath showed prostatic adenocarcinoma. Prostatic cancers occurs mostly in what area/region of the prostate? A. Inner/central glands B. Median lobe C. Outer/peropheral glands D. Transitional zone E. Peri-urethral zone A 37-year-old woman presents with the acute onset of a productive cough, fever, chills, and pleuritic chest pain. CBC showed leukocytosis with predominance of neutrophils. A chest xray reveals consolidation of the entire lower lobe of her right lung. Which of the following is the most likely diagnosis? A. Bronchiectasis B. Bronchopneumonia C. Interstitial pneumonitis D. Lobar pneumonia E. Adult respiratory distress syndrome A 54-year-old female consults due to epigastric pain. History revealed she was taking mefenamic acid for joint pains. Gastroscopy revealed multiple, hemorrhagic ulcers Capillary basement membrane thickening MEDINA, MD (TOP 4 - > Diffuse mesanglial sclerosis FEB 2012 MED > Nodular Glomerulosclerosis BOARDS; TOPNOTCH >Presence of laminated PAS-positive nodules ( MD) Kimmelsteil-Wilson Nodules)

For the past year, a 48 year old woman has noted that her menstrual periods have been exceptionally heavy and last 7 - 9 days. She has noted occasional minor intermenstrual bleeding. For the past 3 months, she has been taking supplemental dietary iron for IDA. On pelvic examination, the uterine cervix appears normal, but the uterus is enlarged to twice its normal size. Transvaginal ultrasound reveals the presence of a 9 cm solid mass in the uterus. A hysterectomy is performed, and on gross inspection with sectioning the uterus, a reddish-tan mass is found with fleshy cut surface. Microscopically, the mass is highly cellular, with spindle cells having hyperchromatic nuclei and 10-20 mitoses per high power field. Which of the following is the most likely diagnosis? A. Endometrial polyp B. Adenomyosis C. Atypical hyperplasia D. Leiomyosarcoma E. Endometriosis

TOPNOTCH EXAM DIAGNOSTIC EXAM - AUG 2012

This patient is in shock due to cardiac tamponade VON ANDRE DIAGNOSTIC secondary to rupture of the ventricular wall, which MEDINA, MD (TOP 4 - EXAM - AUG usually occurs 4-10 days after the initial M.i FEB 2012 MED 2012 fibrinous pericarditis is common in 3-5 days, this BOARDS; TOPNOTCH condition usually presents with chest pain, often a MD) pericardial friction rub can be auscultated.

PSGN- deposits between basement membrane and epithelial cells of the glomeruli

VON ANDRE MEDINA, MD (TOP 4 - FEB 2012 MED BOARDS; TOPNOTCH MD)

DIAGNOSTIC EXAM - AUG 2012



VON ANDRE MEDINA, MD (TOP 4 - FEB 2012 MED BOARDS; TOPNOTCH MD)

DIAGNOSTIC EXAM - AUG 2012

Leiomyosarcoma

VON ANDRE MEDINA, MD (TOP 4 - FEB 2012 MED BOARDS; TOPNOTCH MD)

DIAGNOSTIC EXAM - AUG 2012

TOPNOTCH MEDICAL BOARD PREP PATHOLOGY SUPEREXAM Page 87 of 99 For inquiries visit www.topnotchboardprep.com.ph or email us at [email protected]

TOPNOTCH MEDICAL BOARD PREP PATHOLOGY SUPEREXAM For inquiries visit www.topnotchboardprep.com.ph or email us at [email protected] Item # 630

631

632

633

634

635

636

QUESTION

EXPLANATION

AUTHOR

TOPNOTCH EXAM DIAGNOSTIC EXAM - AUG 2012

A 40 year old male suddenly experienced severe headache. Upon consultation, his BP is noted to be at 200/120. Past history shows normal BP on regular checkups. Anti hypertensive are immediately given. However, BP remains in the range of 200/120-180/100. Nicardipine drip is instituted. Two days later, the patient died becasue of intracerebral hemorrhage. Biopsy of the kidney revealed onion skin, concentric, laminated thickening of the walls of arterioles with progressive narrowing of the lumina consistent with what kind of arteriolosclerosis? A. Hyaline arteriolosclerosis B. Hyperplastic arteriolosclerosis C. Focal segmental nephrosclerosis D. RPGN E. Benign nephrosclerosis A patient with hemoptysis and renal failure has a renal biopsy which reveals crescentic glomerulonephritis, with linear deposits of IgG and C3 on immunofluorescence. What is the probable diagnosis? A. Acute post-infectious glomerulonephritis B. membranous nephropathy C. Goodpasture’s syndrome D. Minimal change disease E. Wegener's granulomatosis

Hyperplastic arteriolosclerosis

VON ANDRE MEDINA, MD (TOP 4 - FEB 2012 MED BOARDS; TOPNOTCH MD)

Goodpasture's syndrome

VON ANDRE MEDINA, MD (TOP 4 - FEB 2012 MED BOARDS; TOPNOTCH MD)

DIAGNOSTIC EXAM - AUG 2012

A 1 year old patient came in due to fever. On physical examination , you noticed that the patient has epicanthal folds, flat facial profile, macroglossia and presence of simian crease. Further history revealed that the patient was born from a 48 year old mother. What is the most common cause of this chromosal disorder? A. Nondisjunction B. Robertsonian translocation C. Mosaicism D. Chromosomal deletion E. None of the above A 5 year old male developed a demyelinating neuropathy associated with Campylobacter jejuni. He presents clinically with ascending neuromuscular paralysis and areflexia. He was then treated with plasmapharesis, IV immunoglobulins and corticosteroids. The diagnosis to this case belongs to what type of hypersensitivity? A. Type I Hypersensitivity B. Type II Cytotoxic Hypersensitivity C. Type III Immune Complex Hypersensitivity D. Type IV Cell mediated Hypersensitivity E. Any of the above choices What is the most common and the most severe form of LUPUS NEPHRITIS ? A. Minimal mesanglial B. Mesanglial proliferative C. Focal proliferative D. Diffuse proliferative E. Membranous

This is a case of trisomy 21, MC is nondisjunction in 95 % of cases.

VON ANDRE MEDINA, MD (TOP 4 - FEB 2012 MED BOARDS; TOPNOTCH MD)

DIAGNOSTIC EXAM - AUG 2012

This is a case of Guillan Barre Syndrome, a type IV Hypersensitivity reaction

VON ANDRE MEDINA, MD (TOP 4 - FEB 2012 MED BOARDS; TOPNOTCH MD)

DIAGNOSTIC EXAM - AUG 2012

Type IV- Diffuse proliferative

VON ANDRE MEDINA, MD (TOP 4 - FEB 2012 MED BOARDS; TOPNOTCH MD)

DIAGNOSTIC EXAM - AUG 2012

A 23 year old female seek consult due to recurrent pyogenic infections caused mainly by staphylococci and streptoccoci. Further work up revealed that she is suffering from an autosomal recessive disease characterized by a failure of phagolysosomal fusion. Moreover, faulty microtubules was found to impair neutrophil chemotaxis. What is the most likely diagnosis? A. Leukocyte Adhesion deficiency B. Chronic Granulomatous Disease C. Chediak-Higashi Syndrome D. Wiskott-Aldrich Syndrome E. Ataxia-Telangiectasia Which of the following disease-malignancy association is INCORRECT? A. Down syndrome: ALL, AML B. Actinic keratosis: squamous cell cancer of the skin C. Acanthosis nigricans: visceral malignancy D. Dysplastic nevus: malignant melanoma E. None of the above

Phagocyte disorder= Chediak Higashi Syndrome

VON ANDRE MEDINA, MD (TOP 4 - FEB 2012 MED BOARDS; TOPNOTCH MD)

DIAGNOSTIC EXAM - AUG 2012

All of the disease-malignancy association are correct

VON ANDRE MEDINA, MD (TOP 4 - FEB 2012 MED BOARDS; TOPNOTCH MD)

DIAGNOSTIC EXAM - AUG 2012

TOPNOTCH MEDICAL BOARD PREP PATHOLOGY SUPEREXAM Page 88 of 99 For inquiries visit www.topnotchboardprep.com.ph or email us at [email protected]

TOPNOTCH MEDICAL BOARD PREP PATHOLOGY SUPEREXAM For inquiries visit www.topnotchboardprep.com.ph or email us at [email protected] Item # 637

638

639

640

641

642

643

644

QUESTION

EXPLANATION

AUTHOR

Cachexia is an irreversible catabolic reaction (wasting syndrome) common to those individual suffering from malignancies and tuberculosis. Which of the following substances is correlated to the pathophysiology of cachexia? A. elevated TNF alpha B. elevated VEGF C. elevated IL6 D. elevated Il 3 E. none of the above Beta HCG is a tumor marker for? A. Hydatidiform mole B. Choriocarcinoma C. Gestational trophoblastic tumor D. All of the above E. None of the above

increased TNF alpha is associated with cachexia which suppresses the appetite center and promotes beta oxidation of fatty acids for fuel

VON ANDRE MEDINA, MD (TOP 4 - FEB 2012 MED BOARDS; TOPNOTCH MD)



VON ANDRE MEDINA, MD (TOP 4 - FEB 2012 MED BOARDS; TOPNOTCH MD)

DIAGNOSTIC EXAM - AUG 2012

A 65 year old male immigrant from Africa presents to the emergency department after an episode of gross hematuria. He states that he has seen small amounts of blood in his urine from time to time over the past several months. His physical examination is remarkable only for mild hepatosplenomegaly. A urology consult is called, and the urologist performs a bedside cytoscopy. A large fungating mass is seen adherent to the superior part of the bladder. What is the most likely environmental exposure associated with this disease in the patient? A. Cigarette smoking B. Exposure to aniline dyes C. Schistosoma infection D. Infection with papova virus E. Long term indwelling catheter Glanzmann thrombasthenia is a deficiency of? A. GpIIb-IIIa complex B. GpIb C. Factor IX D. Fibrinogen E. vWF

diagnosis: squamous cell carcinoma of the bladder, MC cause is chronic irritation of Schistosoma haematobium.

VON ANDRE MEDINA, MD (TOP 4 - FEB 2012 MED BOARDS; TOPNOTCH MD)

DIAGNOSTIC EXAM - AUG 2012

GpIIb-IIIa complex- Glanzmann GpIb- bernard soulier

VON ANDRE MEDINA, MD (TOP 4 - FEB 2012 MED BOARDS; TOPNOTCH MD)

DIAGNOSTIC EXAM - AUG 2012

What do you call the systemic reaction including rashes, brochoconstriction and periorbital edema after injecting an unknown antigen to different parts of your body? A. Arthus Reaction B. Serum Sickness C. Delayed hypersensitivity reaction D. Anaphylactic Shock E. None of the above Which among the following components of the immune system is NOT included in the pathophysiology of HIV infection? A. CD4 B. CD8 C. Dendritic cells D. Macrophages E. No excemption, all of the above are included in the pathophysiology of HIV infection Which of the following imunoglobulins is/are being secreted by the immune system of the GIT? A. Dimeric IgA B. IgG C. IgM D. B and C only E. All of the above

Arthus reaction is almost the same as serum sickness but occurs as a local inflammatory reaction. Anaphylactic shock is re-exposure to a known allergen, leading to severe vasodilatation and hemodynamic instability.

LITO JAY MACARAIG, MD (TOP 8 - FEB 2013 MED BOARDS; TOPNOTCH MD)

DIAGNOSTIC EXAM - AUG 2013

CD8 or the cytotoxic cells are not included in the pathophysiology of HIV infection. It is the CD4 that is affected by HIV.

LITO JAY MACARAIG, MD (TOP 8 - FEB 2013 MED BOARDS; TOPNOTCH MD)

DIAGNOSTIC EXAM - AUG 2013

As stated in Robbins, the immune system of the GIT secretes dimeric IgA, IgG and IgM.

LITO JAY MACARAIG, MD (TOP 8 - FEB 2013 MED BOARDS; TOPNOTCH MD)

DIAGNOSTIC EXAM - AUG 2013

While on duty as IM resident, a 28 year old female came to ER who presented with th triad of microangiopathic anemia, thrombocytopenia and renal failure. Further history revealed eating half-cooked burgers 2 days PTC. Which among the choices is/are true regarding the etiologic agent? A. Shiga-like toxin B. EHEC C. E. coli 0157:H7 D. B and C only E. All of the above

this is Hemolytic-Uremic Syndrome. It is caused by shiga-like toxin from E. coli 0157:H7 from poorly cooked burger patties.

LITO JAY MACARAIG, MD (TOP 8 - FEB 2013 MED BOARDS; TOPNOTCH MD)

DIAGNOSTIC EXAM - AUG 2013

TOPNOTCH MEDICAL BOARD PREP PATHOLOGY SUPEREXAM Page 89 of 99 For inquiries visit www.topnotchboardprep.com.ph or email us at [email protected]

TOPNOTCH EXAM DIAGNOSTIC EXAM - AUG 2012

TOPNOTCH MEDICAL BOARD PREP PATHOLOGY SUPEREXAM For inquiries visit www.topnotchboardprep.com.ph or email us at [email protected] Item # 645

646

647

648

649

650

651

652

QUESTION

EXPLANATION

AUTHOR

TOPNOTCH EXAM DIAGNOSTIC EXAM - AUG 2013

A 47 year old male consulted your clinic due to unexplained fever accompanied by weakness. CBC revealed leukocytosis of 187,000/mm3. Suspecting leukemia, you requested bone marrow biopsy which revealed scattered macrophages with abundant wrinkled, green blue cytoplasm. What is the diagnosis? A. AML B. CML C. ALL D. CLL E. Acute Promyelocytic Leukemia During your patho rotation, the 3rd year resident showed you a slide containing large cells with multiple nuclei, some have single nucleus with multiple nuclear lobes (each half is a mirror image of the other). With this very limited information, it is safe to say that the patient whom this specimen belongs to is suffering from? A. Chronic Lymphocytic Leukemia B. Hairy Cell Leukemia C. Non-hodgkin's lymphoma D. Hodgkin's lymphoma E. Leukemoid reaction Which among the following statements is/are true regarding thymoma? A. Associated with Pure Red Cell aplasia B. Associated with Myasthenia Gravis C. Histopath findings may show swirling pattern of sphindle-shaped cells D. B and C only E. All of the above This is the most common complication of Chronic Otitis Media, characterized by cystic lesions filled with amorphous debris. A. Chloroma B. Branchial pouch cyst C. Cholesteatoma D. A and C only E. None of the above

the scattered macrophages with abundant wrinkled, green--blue cytoplasm is the "sea-blue histiocytes" which is the pathognomonic finding for Chronic Myelogenous Leukemia

LITO JAY MACARAIG, MD (TOP 8 - FEB 2013 MED BOARDS; TOPNOTCH MD)

the description given points out to the REEDSTERNBERG cells which are the pathognomonic for Hodgkin's lymphoma

LITO JAY MACARAIG, MD (TOP 8 - FEB 2013 MED BOARDS; TOPNOTCH MD)

DIAGNOSTIC EXAM - AUG 2013

topnotch notes

LITO JAY MACARAIG, MD (TOP 8 - FEB 2013 MED BOARDS; TOPNOTCH MD)

DIAGNOSTIC EXAM - AUG 2013

Cholesteatomas are mainly deriveds from desquamated epithelium resulted from chronic otitis media.

LITO JAY MACARAIG, MD (TOP 8 - FEB 2013 MED BOARDS; TOPNOTCH MD)

DIAGNOSTIC EXAM - AUG 2013

A 54 year old female presented with painless palpable mass on upper-outer quadrant of her left breast. History revealed previous surgical removal of a mass (2 months PTC) of which histopath showed fibroadenoma. If you are about to request a biopsy of the new mass, what findings do you expect? A. localized area of inflammation which may progress to abscess formation B. ill-defined, firm, gray-white nodules C. chalky-white foci with or without hemorrhagic debris D. A and B only E. B and C only This is the most common type of breast cancer with characteristic grating sound (like cutting a water chestnut) when cut. A. DCIS B. LCIS C. Invasive ductal CA D. Invasive lobular CA E. Medullary CA

this is a case of Traumatic Fat necrosis. Usual clinical picture is a patient with history of trauma or prior surgery who presented with painless palpable mass. Choices B and C pertains to histopath findings of this condition.

LITO JAY MACARAIG, MD (TOP 8 - FEB 2013 MED BOARDS; TOPNOTCH MD)

DIAGNOSTIC EXAM - AUG 2013

About 70% of diagnosed breast cancers are invasive ductal CA's. The grating sound is due to small, central pinpoint focior streaks of chalky-white elastotic stroma

LITO JAY MACARAIG, MD (TOP 8 - FEB 2013 MED BOARDS; TOPNOTCH MD)

DIAGNOSTIC EXAM - AUG 2013

Psoriasis is a skin disease with predilection to elbows, knees, scalp, and lumbosacral areas. Which among the following lesions is/are related to the said disease? A. Test tubes in rack appearance B. Suprapapillary plates wuth dilated, tortuos blood vessels C. spongiform pustules of Kajog D. A and C only E. All of the above A 12 year old male was brought to your clinic due to pruritic, pink to skin-colored papules with central umbilication. What is your primary differential diagnosis? A. Acne vulgaris B. Verrucae C. Molluscum Contagiosum D. Impetigo E. Phemphigus vulgaris

All of the choices given pertainsto the series of histopath findings can be found on psoriasis.

LITO JAY MACARAIG, MD (TOP 8 - FEB 2013 MED BOARDS; TOPNOTCH MD)

DIAGNOSTIC EXAM - AUG 2013

this is a typical description of Molluscum contagiosum. Pathognomic finding is the central umbilication

LITO JAY MACARAIG, MD (TOP 8 - FEB 2013 MED BOARDS; TOPNOTCH MD)

DIAGNOSTIC EXAM - AUG 2013

TOPNOTCH MEDICAL BOARD PREP PATHOLOGY SUPEREXAM Page 90 of 99 For inquiries visit www.topnotchboardprep.com.ph or email us at [email protected]

TOPNOTCH MEDICAL BOARD PREP PATHOLOGY SUPEREXAM For inquiries visit www.topnotchboardprep.com.ph or email us at [email protected] Item # 653

654

655

656

657

658

659

660

QUESTION

EXPLANATION

AUTHOR

TOPNOTCH EXAM DIAGNOSTIC EXAM - AUG 2013

Retinoblastoma is the most common primary intra-ocular malignancy of children. What do you call its pathognominic morphology which is described as arrangements of a single layer of tumor cells around an apparent lumen? A. Homer Wright Rosettes B. Homer Wright Pseudorosettes C. Flexner-Wintersteiner rosettes D. Mutton-Fat precipitates E. Hollenhorst plaques

Homer Wright Pseudo rosettes - seen in neuroblastoma, medulloblastoma. True rosettes are the flexner wintersteiner rosettes.

LITO JAY MACARAIG, MD (TOP 8 - FEB 2013 MED BOARDS; TOPNOTCH MD)

This is a sequelae of chronic Escherichia coli infection, characterized by histopathologic findings of large, foamy macrophages with laminatedmineralized concretions. A. Hemorrhagic cystitis B. Interstitial cystitis C. Malacoplakia D. Papillary Urothelial CA E. Papillary urothelial neoplasms of Low malignant potential A 16 month old boy was brought to your clinic due to history of 2 days fever. Being so busy, you just requested urinalysis which showed PMN of 15-19 cells per HPF. You were told that the patient had 4 episodes of UTI for the past year. That is the only time you decided to examine his genitalia. You noticed that the orifice of his penile prepuce is too small. What is your diagnosis? A. Hypospadia B. Epispadia C. Phimosis D. Balanitis E. Balanoposthitis A 49 year old female consulted to your clinic due to progressively enlarging abdomen. Upon PE, you noted a non-painful palpable mass at the left lower quadrant. Further studies showed elevated serum AFP, normal beta hCG and inhibin levels. You referred the patient to an obstetrician for operation. You are expecting to find Schiller-Duval bodies on histopath because you know this is a case of? A. Leydig cell tumor B. Sertoli cell tumor C. Yolk sac tumor D. Choriocarcinoma E. Teratoma Which of the following statements is/are true regarding Guillain-Barre Syndrome? A. Immune-mediated demyelinating neuropathy B. Associated with Campylobacter jejuni infection C. Ascending motor paralysis D. A and C only E. All of the above This is a bone disease characterized by mosaic pattern or jigsaw puzzle appearance on microscopy. A. Osteitis Fibrosa Cystica B. Von Recklinghausen disease of the bone C. Paget's disease D. Osteogenesis imperfecta E. Osteopetrosis

the histopath findings described is called MichaelisGuttman bodies which is the pathognomonic finding for Malacoplakia

LITO JAY MACARAIG, MD (TOP 8 - FEB 2013 MED BOARDS; TOPNOTCH MD)

DIAGNOSTIC EXAM - AUG 2013

Hypospadia is the condition where the meatus is located ventrally, epispadia if dorsally. Balanitis is the inflamation of glans penis.

LITO JAY MACARAIG, MD (TOP 8 - FEB 2013 MED BOARDS; TOPNOTCH MD)

DIAGNOSTIC EXAM - AUG 2013

On histopath, Yolk sac tumor will show lacelike reticular cell network of structures resembling endodermal sinuses called Schiller-Duval bodies.

LITO JAY MACARAIG, MD (TOP 8 - FEB 2013 MED BOARDS; TOPNOTCH MD)

DIAGNOSTIC EXAM - AUG 2013



LITO JAY MACARAIG, MD (TOP 8 - FEB 2013 MED BOARDS; TOPNOTCH MD)

DIAGNOSTIC EXAM - AUG 2013

choices A and B were the same. OI is a collagen type I problem. OP is the marble bone disease.

LITO JAY MACARAIG, MD (TOP 8 - FEB 2013 MED BOARDS; TOPNOTCH MD)

DIAGNOSTIC EXAM - AUG 2013

A 28 year-old construction worker fell from a 3storey building and hit his head on the pavement. Patient came in conscious but after 20 minutes, became stupurous and eventually GCS 3. You are entertaining Epidural hematoma. Which among the following is/are associated with Epidural hematoma? A. fractured Pterion B. Lucid interval C. Lenticular lesion on CT scan D. B and C only E. All of the above The triad of Graves disease includes? A. hyperthyroidism B. ophthalmopathy C. Pretibial myxedema D. A and B only E. All of the above



LITO JAY MACARAIG, MD (TOP 8 - FEB 2013 MED BOARDS; TOPNOTCH MD)

DIAGNOSTIC EXAM - AUG 2013



LITO JAY MACARAIG, MD (TOP 8 - FEB 2013 MED BOARDS; TOPNOTCH MD)

DIAGNOSTIC EXAM - AUG 2013

TOPNOTCH MEDICAL BOARD PREP PATHOLOGY SUPEREXAM Page 91 of 99 For inquiries visit www.topnotchboardprep.com.ph or email us at [email protected]

TOPNOTCH MEDICAL BOARD PREP PATHOLOGY SUPEREXAM For inquiries visit www.topnotchboardprep.com.ph or email us at [email protected] Item # 661

QUESTION

EXPLANATION

AUTHOR

TOPNOTCH EXAM MIDTERM 2 - AUG 2013

Type of cellular adaptation exhibited by genital warts? A. hyperplasia B. Hypertrophy C. metaplasia D. dysplasia E. None of the above

Number of cells in organ increase, the process is reversible when stimulus stops

HAZEL KAREN RAZ, MD (TOP 6 - FEB 2013 MED BOARDS; TOPNOTCH MD)

662

Identify the mechanism of injury from cellular hypoxia? A. ATP depletion B. Free radical injury C. A & B D. None E. Hydrolysis

Hypoxia causes injury by depleting ATP sources (no ETC and Krebs cycle) and free radical injury

HAZEL KAREN RAZ, MD (TOP 6 - FEB 2013 MED BOARDS; TOPNOTCH MD)

MIDTERM 2 - AUG 2013

663

During the process of inflammation, laukocytes adhers to integrins found on endothelial surfaces thru this athesion molecules? A. E-selection (rolling) B. PECAM (diapedesis) C. Sialyl lewis (rolling) D. ICAM - I E. None of the above

E-selection (rolling) PECAM (diapedesis) Sialyl lewis (rolling)

HAZEL KAREN RAZ, MD (TOP 6 - FEB 2013 MED BOARDS; TOPNOTCH MD)

MIDTERM 2 - AUG 2013

664

True of the complement system? A. Consists of plasma proteins which participates in immune lysis of cells but does not play a role in inflammation B. C3A is chemotactic (C5A) C. C5A is an anaphyiotoxin (C5A & C3A) D. C3B is part of the MAC (C5B-A) E. None of the above

Consists of plasma proteins which participates in immune lysis of cells AND PLAYS a role in inflammation C3A is chemotactic - (C5A) C5A is an anaphyiotoxin (C5A & C3A) C3B is part of the MAC (C5B-A)

HAZEL KAREN RAZ, MD (TOP 6 - FEB 2013 MED BOARDS; TOPNOTCH MD)

MIDTERM 2 - AUG 2013

665

Observed in deep-sea divers who return to the surface too rapidly, resulting in musculoskeleteal pain and infarcts in the CNS? A. Embolism Syndrome B. Decompression Sickness C. Fluid Emboli D. Paradoxical Emboli E. None of the above

Decompression sickness (DCS; also known as divers' disease, the bends or caisson disease) describes a condition arising from dissolved gases coming out of solution into bubbles inside the body on depressurisation.

HAZEL KAREN RAZ, MD (TOP 6 - FEB 2013 MED BOARDS; TOPNOTCH MD)

MIDTERM 2 - AUG 2013

666

Newborn presents with a prominent acciput, micrognathia, low set ears and rodenbottom feet genetic d/o caused by nondisjunction is called? A. Down Syndrome B. DiGeorge Syndrome C. Edward's Syndrom D. Patau Syndrome E. None of the above Type of hypersensitivity reaction manifested by a patient with weight loss, taxhycardia, exophthalmos and elevated free thyroid hormones? A. Type I Anaphylaxis B. Type II Cytotoxic C. Type III Immune complex D. Type IV Delayed/Cell Mediated E. None of the above A patient whose mother was exposed to DES during pregnancy is prone to developing? A. VSD B. Limb Anomalies C. Acute Leukemia D. Clear Cell CA of Vagina E. None of the above

Edwards syndrome (also known as Trisomy 18 (T18)) is a genetic disorder caused by the presence of all or part of an extra 18th chromosome. This genetic condition almost always results from nondisjunction during meiosis.

HAZEL KAREN RAZ, MD (TOP 6 - FEB 2013 MED BOARDS; TOPNOTCH MD)

MIDTERM 2 - AUG 2013

Type I Anaphylaxis Type II Cytotoxic - Graves Disease Type III Immune complex Type IV Delayed/Cell Mediated

HAZEL KAREN RAZ, MD (TOP 6 - FEB 2013 MED BOARDS; TOPNOTCH MD)

MIDTERM 2 - AUG 2013

In 1971, DES was shown to cause a rare vaginal tumor in girls and women who had been exposed to this drug in utero.

HAZEL KAREN RAZ, MD (TOP 6 - FEB 2013 MED BOARDS; TOPNOTCH MD)

MIDTERM 2 - AUG 2013

Which of the following is the most toxic vitamin? A. Vitamin A B. Vitamin D C. Vitamin E D. Vitamin K E. None of the above

Vitamin D is a group of fatsoluble secosteroids responsible for intestinal absorption of calcium and phosphate.The threshold for vitamin D toxicity has not been established, however, the UL is 4000 IU/day for ages 9–71.

HAZEL KAREN RAZ, MD (TOP 6 - FEB 2013 MED BOARDS; TOPNOTCH MD)

MIDTERM 2 - AUG 2013

667

668

669

TOPNOTCH MEDICAL BOARD PREP PATHOLOGY SUPEREXAM Page 92 of 99 For inquiries visit www.topnotchboardprep.com.ph or email us at [email protected]

TOPNOTCH MEDICAL BOARD PREP PATHOLOGY SUPEREXAM For inquiries visit www.topnotchboardprep.com.ph or email us at [email protected] Item # 670

671

672

673

674

675

676

QUESTION

EXPLANATION

AUTHOR

A 5-Year old boy came to you with complaints of fever, headache and joint pains on physical examination you note erythematons papules and plaques appearing in crops lower abdomen and buttocks. History revealed that he had cough and colds two weeks prior to consult. What is the diagnosis? A. Polyartentis Nodosa B. Buerqer Disease C. Serum Sickness D. Henoch-Schonlein Purpura E. None of the above A 40 year old male presented with crushing chest pain radiationg to his left arm, associated with dizziness and dyspnea. ECG was done at the ER showing chair-pattern tracing at chest leods V2, V3 and aVf. What is the most common complication 7 days after the incident? A. Arythmia B. Myocardial Rupture C. Ventricular Aneurysm D. Dressler Syndrome E. None of the above A 21-year old male who has a history of recurrent throat infection develops fever and joint pains. On physical examination friction rub is heard at the latoral border of the sternum, with rapid pulse. What is the pathognomynic sign in this patient? A. Verrucae B. Anitschow Myocytes C. Aschoff Body D. Heberden's Node E. None of the above A 30 year old female with a history of oral contraceptive use presented with fatigue, headache, palpitations, sore tongue and generalized pallor, she was diagnosed to have filate dificiency. The following is seen in the PBS of patients with FA deficiency except? A. Hyposegmentad neutrophils B. Pancytopenia C. Macrocytosis D. a and b E. none Characteristic histopathologic findings in multiple myeloma? A. Fiery red cytoplasm B. Multiple nuclei with prominent nucleoli and cytoplasmic droplets containing immunoglobulin C. Pink gobluar cytoplasmic inclusions D. AOTA E. None of the above A 35-year old male came in due to fever, weight loss, night sweats and easy bruising. On physical examination, splenomegaly and swollen gums are present. CBC shows leukocytosis with predominance of blasts. PBS shows distinctive needle-like azurophilic granules in myeloblasts. What is the diagnosis? A. ALL B. AML C. CLL D. CML E. None of the above In Von Willebrand disease, deficiency of VWF leads to dysfunctional platelet adhesion. Laboratory findings diagnostic of VWF includes? A. Inc. Bleeding time, Dec. Platelet count, Inc. PT, Inc. PTT B. Inc. Bleeding time, Dec. Platelet Count, Normal PT, PTT C. Inc. Bleeding Time, Normal Platelet Count, Normal PT, Inc. PTT D. Normal Bleeding Time, Normal Platelet Count, Normal PT, Inc. PTT E. None of the above

HSP is a disease of the skin and other organs that most commonly affects children. In the skin, the disease causes palpable purpura (small hemorrhages); often with joint and abdominal pain.

HAZEL KAREN RAZ, MD (TOP 6 - FEB 2013 MED BOARDS; TOPNOTCH MD)

Arythmia - 24 hours Myocardial Rupture - 4 - 7 days Ventricular Aneurysm - > 7 weeks Dressler Syndrome - pericarditis post - MI

HAZEL KAREN RAZ, MD (TOP 6 - FEB 2013 MED BOARDS; TOPNOTCH MD)

MIDTERM 2 - AUG 2013

Aschoff bodies are nodules found in the hearts of individuals with rheumatic fever. They result from inflammation in the heart muscle and are characteristic of rheumatic heart disease.

HAZEL KAREN RAZ, MD (TOP 6 - FEB 2013 MED BOARDS; TOPNOTCH MD)

MIDTERM 2 - AUG 2013

Folate deficiency is a lack of folic acid in the diet and the signs are often subtle. Folate deficiency anemia is the medical name given for the condition. PBS show hypersegmented nuclei, pancytopenia and macrocytosis.

HAZEL KAREN RAZ, MD (TOP 6 - FEB 2013 MED BOARDS; TOPNOTCH MD)

MIDTERM 2 - AUG 2013

In multiple myeloma, collections of abnormal plasma cells accumulate in the bone marrow, where they interfere with the production of normal blood cells. Most cases of myeloma also feature the production of a paraprotein—an abnormal antibody which can cause kidney problems. Bone lesions and hypercalcemia (high calcium levels) are also often encountered

HAZEL KAREN RAZ, MD (TOP 6 - FEB 2013 MED BOARDS; TOPNOTCH MD)

MIDTERM 2 - AUG 2013

AML is a cancer of the myeloid line of blood cells, characterized by the rapid growth of abnormal white blood cells that accumulate in the bone marrow and interfere with the production of normal blood cells. AML is the most common acute leukemia affecting adults, and its incidence increases with age.

HAZEL KAREN RAZ, MD (TOP 6 - FEB 2013 MED BOARDS; TOPNOTCH MD)

MIDTERM 2 - AUG 2013

Inc. Bleeding time, Dec. Platelet count, Inc. PT, Inc. PTT (DIC) Inc. Bleeding time, Dec. Platelet Count, Normal PT, PTT (Thrombocytopenia) Normal Bleeding Time, Normal Platelet Count, Normal PT, Inc. PTT (Hemophilia)

HAZEL KAREN RAZ, MD (TOP 6 - FEB 2013 MED BOARDS; TOPNOTCH MD)

MIDTERM 2 - AUG 2013

TOPNOTCH MEDICAL BOARD PREP PATHOLOGY SUPEREXAM Page 93 of 99 For inquiries visit www.topnotchboardprep.com.ph or email us at [email protected]

TOPNOTCH EXAM MIDTERM 2 - AUG 2013

TOPNOTCH MEDICAL BOARD PREP PATHOLOGY SUPEREXAM For inquiries visit www.topnotchboardprep.com.ph or email us at [email protected] Item # 677

QUESTION

EXPLANATION

AUTHOR

TOPNOTCH EXAM MIDTERM 2 - AUG 2013

Deficiency of alpha-1 antitrypsin leads to this type of emphysema? A. Centrilobular B. Panacinar C. Paraseptal D. Irregular E. None of the above

Severe A1AT deficiency causes panacinar emphysema or COPD in adult life in many people with the condition (especially if they are exposed to cigarette smoke), as well as various liver diseases in a minority of children and adults, and occasionally more unusual problems

HAZEL KAREN RAZ, MD (TOP 6 - FEB 2013 MED BOARDS; TOPNOTCH MD)

678

Most common type of cancer arising from the distal 1/3 of esophagus? A. Squamous cell CA B. Barrett's esophagus C. Adenocareinoma D. Netastasis E. None of the above

Adenocarcinoma arises from glandular cells that are present at the junction of the esophagus and stomach.

HAZEL KAREN RAZ, MD (TOP 6 - FEB 2013 MED BOARDS; TOPNOTCH MD)

MIDTERM 2 - AUG 2013

679

Most common pituitary tumor? A. Prolactinoma B. Somatotropic Adenoma C. Acidophilic adenoma D. Corticotrophic Adenoma E. None of the above

A prolactinoma is a benign tumor (adenoma) of the pituitary gland that produces a hormone called prolactin. It is the most common type of pituitary tumor. Symptoms of prolactinoma are caused by too much prolactin in the blood (hyperprolactinemia) or by pressure of the tumor on surrounding tissues.

HAZEL KAREN RAZ, MD (TOP 6 - FEB 2013 MED BOARDS; TOPNOTCH MD)

MIDTERM 2 - AUG 2013

680

A 20-year old female presented with malar rash, oral ulcers, protosensitivity and proteinuria with granules or casts, kidney biopsy was done showing a "wire loop abnormality" on light microscope with marked subendothelial immune complex deposition on electron microscope. What is the diagnosis? A. IqA nephropathy B. Goodpasture syndrome C. Focal segmental glomerulosclerosis D. Lupus nephropathy E. None of the above What endothelial-leukocyte adhesion molecule has a major role in adhesion, arrest and transmigration of neutrophils, monocytes and lymphocytes? A. P-selectin B. E-selectin C. VCAM-1 D. ICAM-1 Mrs. Kaka Awa, 32 yo presented to the ED because she passed blood instead of urine morning PTA after aerobics the night before. She also had 3-day history of recurrent attacks of tolerable abdominal pain and vomiting. Physical examination revealed mild jaundice, abdominal distention, and bilateral lower extremity edema. Laboratory evaluation revealed hemoglobin level of 7.0 g/dL, pancytopenia, total serum bilirubun level 4.4 mg/dL, ALT = 51 U/L, AST = 17 U/L, serum creatinine normal. Initial screening with sugar water hemolysis test was positive. Findings at abdominal radiography showed a small amount of bowel gas but disclosed no other abnormalities. US and CT findings included inferior vena caval, hepatic venous, and portal venous thrombosis, with an associated BuddChiari Syndrome. Further studies revealed acquired mutations in the phosphatidylinositol glycan complementation group A gene (PIGA). In the absence of liver disease, this raises the suspicion of what condition? A. Paroxysmal Cold Hemoglobinuria B. March Hemoglobinuria C. Hereditary Spherocytosis D. Marchiafava-Micheli Syndrome Which of the following is most likely associated with the Philadelphia chromosome? A. Found in AML B. t (9;23) C. BCR-ABL gene D. Good prognosis

A wire-loop lesion may be present in stage III and IV. This is a glomerular capillary loop with subendothelial immune complex deposition that is circumferential around the loop.

HAZEL KAREN RAZ, MD (TOP 6 - FEB 2013 MED BOARDS; TOPNOTCH MD)

MIDTERM 2 - AUG 2013

Answer: D. ICAM-1 (Table 2-1, Robbins and Cotran Pathologic Basis of Disease, 8th ed.) *SIMILAR TO PREVIOUS BOARD EXAM CONCEPT/PRINCIPLE

MICHELLE JAY FRANCISCO, MD (TOP 9 - FEB 2013 MED BOARDS; TOPNOTCH MD)

MIDTERM 1 - AUG 2013

Answer: D. Marchiafava-Micheli Syndrome Notes: Paroxysmal NOCTURNAL Hemoglobinuria (PNH), sometimes referred to as Marchiafava-Micheli Syndrome, is a disease that results from acquired mutations in the PIGA gene, an enzyme that is essential for the synthesis of certain cell surface proteins. PNH triad: hemolysis, pancytopenia and distinct tendency to venous thrombosis. Screening test: Sugar water hemolysis test, confirmatory test: Ham’s test. The only form of treatment that currently can provide a definitive cure for PNH is allogeneic bone marrow transplantation (BMT).

MICHELLE JAY FRANCISCO, MD (TOP 9 - FEB 2013 MED BOARDS; TOPNOTCH MD)

MIDTERM 1 - AUG 2013

Answer: C. BCR-ABL gene

MICHELLE JAY FRANCISCO, MD (TOP 9 - FEB 2013 MED BOARDS; TOPNOTCH MD)

MIDTERM 1 - AUG 2013

681

682

683

TOPNOTCH MEDICAL BOARD PREP PATHOLOGY SUPEREXAM Page 94 of 99 For inquiries visit www.topnotchboardprep.com.ph or email us at [email protected]

TOPNOTCH MEDICAL BOARD PREP PATHOLOGY SUPEREXAM For inquiries visit www.topnotchboardprep.com.ph or email us at [email protected] Item # 684

685

686

687

688

689

QUESTION

EXPLANATION

AUTHOR

After a week in the hospital for treatment of an upper respiratory infection complicated by pneumonia, a 43 yo female develops skin lesions that are 2 to 4 mm in diameter. These lesions are red, papulovesicular, oozing, and crusted and are located on her trunk and extremities. The lesions begin to disappear after she is discharged from the hospital a week later. What is the most likely pathogenesis for her skin lesions? A. Type I hypersensitivity B. Drug reaction C. Bacterial septicemia D. Photosensitivity A 45 yo female complained of headaches for about a month. She then suffered a generalized seizure and became obtunded. Her serum calcium concentration was found to be markedly elevated at 15.4 mg/dL, with a serum phosphorus level of only 1.9 mg/dL. The serum albumin level was 4.2 g/dL. A chest radiograph showed multiple lung masses, and there appeared to be lytic lesions of the vertebral column. Which of the following conditions best accounts for these findings? A. Parathyroid carcinoma B. Metastatic breast cancer C. Tuberculosis D. Vitamin D toxicity In the third trimester of pregnancy, a 28 yo woman discovers a lump in her right breast. Her physician palpates a 2-cm, discrete, freely movable mass beneath the nipple. After delivery of a term infant, the mass appears to decrease slightly in size. The infant breast-feeds without difficulty. This breast lesion is most likely to be a (an) A. Intraductal papilloma B. Phyllodes tumor C. Lobular carcinoma in situ D. Fibroadenoma An otherwise healthy 72 yo male has increasing difficulty with urination. He has to get up several times each night because of a feeling of urgency, but each time, the urine volume is not great. He has difficulty starting and stopping urination. This problem has gotten worse over the last few years. His serum PSA level is slightly increased but stable over this time. A biopsy of the prostate is most likely to reveal which of the following? A. Hyperplastic nodules of stroma and glands lined by two layers of epithelium B. Poorly differentiated glands lined by a single layer of epithelium and packed back to back C. Foci of chronic inflammatory cells in the stroma and in normal-appearing glands D. Areas of liquefactive necrosis filled with neutrophils This term refers to an increase in white blood cells in cerebrospinal fluid A. Cylindruria B. Pleocytosis C. Leukocytosis D. Xanthochromia

Answer: B. Drug reaction Notes: The time course fits best with a drug reaction producing an acute erythematous dermatitis. Urticaria from type I hypersensitivity is not as severe or as long lasting. Sepsis rarely involves the skin with an erythematous dermatitis. Photosensitivity may be enhanced by drugs, but UV light is the key component in light that produces photodermatitis.

MICHELLE JAY FRANCISCO, MD (TOP 9 - FEB 2013 MED BOARDS; TOPNOTCH MD)

Answer: B. Metastatic breast cancer Notes: The most common cause for clinically significant hypercalcemia in adults is a malignancy. Metastatic disease from common primaries such as breast, lung, and kidney tumors is much more frequent than parathyroid carcinoma, which tends to be local but aggressive.

MICHELLE JAY FRANCISCO, MD (TOP 9 - FEB 2013 MED BOARDS; TOPNOTCH MD)

MIDTERM 1 - AUG 2013

Answer: D. Fibroadenoma Notes: Fibroadenomas are common, and they may enlarge in pregnancy or late in menstrual cycle. Most intraductal papillomas are smaller than 1 cm, and they are not influenced by hormonal changes. Phyllodes tumors are uncommon, and they tend to be larger than 4 cm. Lobular carcinoma in situ (LCIS) is typically an ill-defined lesion without a mass effect.

MICHELLE JAY FRANCISCO, MD (TOP 9 - FEB 2013 MED BOARDS; TOPNOTCH MD)

MIDTERM 1 - AUG 2013

Answer: A. Hyperplastic nodules of stroma and glands lined by two layers of epithelium Notes: The clinical features are typical of nodular hyperplasia of prostate. Mild elevation of the PSA level can occur with nodular hyperplasia. The area of the prostate that is most often involved with nodular hyperplasia to produce significant obstruction is in the inner (transitional and periurethral) zone.

MICHELLE JAY FRANCISCO, MD (TOP 9 - FEB 2013 MED BOARDS; TOPNOTCH MD)

MIDTERM 1 - AUG 2013

Answer: B. Pleocytosis

MICHELLE JAY FRANCISCO, MD (TOP 9 - FEB 2013 MED BOARDS; TOPNOTCH MD)

MIDTERM 1 - AUG 2013

A 60 yo male who has terminal carcinoma of the colon develops widespread ecchymoses over his skin surface. The PT is 3 seconds, PTT is 55 seconds, platelet count is 15,200/uL, fibrinogen level is 75mg/dL, and fibrin split products levels are very elevated. Which of the following morphologic findings would you most expect to find on examination of the PBS? A. Howell-Jolly bodies B. Tear-drop cells C. Macro-ovalocytes D. Schistocytes E. Target cells

Answer: D. Schistocytes Notes: This is an example of a DIC with associated microangiopathic haemolytic anemia. The DIC developed the setting of a mucin-secreting adenocarcinoma. Schistocytes are fragmented RBCs.

MICHELLE JAY FRANCISCO, MD (TOP 9 - FEB 2013 MED BOARDS; TOPNOTCH MD)

MIDTERM 1 - AUG 2013

TOPNOTCH MEDICAL BOARD PREP PATHOLOGY SUPEREXAM Page 95 of 99 For inquiries visit www.topnotchboardprep.com.ph or email us at [email protected]

TOPNOTCH EXAM MIDTERM 1 - AUG 2013

TOPNOTCH MEDICAL BOARD PREP PATHOLOGY SUPEREXAM For inquiries visit www.topnotchboardprep.com.ph or email us at [email protected] Item # 690

691

692

693

694

695

696

QUESTION

EXPLANATION

AUTHOR

Mutya, a 3 yo girl presents with dark precipitates along gingival margins, radiopaque deposits in the epiphyses of her bones, and urinary excretion of delta-aminolevulinic acid (delta-ALA). Her father states that they live in an old house that has chipped paint. The child’s blood would most likely have which of the following? A. Schistocytes and helmet cells B. Basophilic stippling of erythrocytes C. Increased osmotic fragility of erythrocytes D. Clumping of erythrocytes at temperatures below 30OC A 35 yr-old woman who has been taking oral contraceptives for many years presents with acute abdominal pain and fullness. Paracentesis harvests 200 ml of bloody fluid. Imaging studies show a 6-cm mass in the liver that is subsequently resected. Histologic exam of this specimen would most likely reveal this to be which of the following? A. Angiosarcoma B. Cholangiosarcoma C. Focal nodular hyperplasia D. Hepatocellular carcinoma E. Liver cell adenoma The alterations in the hemodynamic, metabolic and immune responses evident in stressed patients are orchestrated by endogenous polypeptides known as cytokines. They are produced by immune cells in direct response to injury, with levels correlating with the degree of tissue damage. Despite considerable overlap in bioactivity among cytokines, they are commonly classified by their predominant effect as proinflammatory or antiinflammatory. Which is not considered as the former? A. IL-1 B. IL-4 C. IL-6 D. IFN-y Malignancy is second only to trauma as the leading cause of death in children. In infants, it is the most frequent cause of death after prematurity and congenital anomalies. Approximately 40% of childhood anomalies are A. Lymphoma B. Leukemia C. Neuroblastoma D. Rhabdomyosarcoma Upon investigation, a farmer was diagnosed to be infected with actinomycetes. His lung was noted to have local, subacute hypersensitivity reaction characterized by edema and necrosis along with complement activation. Your impression is : A. Serum Sickness B. Arthus reaction C. Graft-versus-host disease D. Goodpasture syndrome A 70 y.o man with sepsis has a pH of 7.18. which of the following statements is true regarding his metabolic acidosis? A. Tissue hypoxia leads to increased oxidative metabolism. B. Acute compensation for metabolic acidosis is primarily renal. C. Metabolic acidosis results from loss of bicarbonate or gain of fixed acids. D. Restoration of blood pressure with vsopressors corrects the acidosis associated with circulatory failure. In differentiating obstructive from restrictive lung diseases, this spirometry criteria characterizes the former: A. Decreased TLC B. Decreased FEV1/FVC ratio C. Decreased FRC/FEV ratio D. Decreased RV

Answer: B. Basophilic stippling of erythrocytes Notes: Classic features of lead poisoning forming a gingival lead line (composed of precipitated lead sulfide), radiopaque deposits in epiphyses, basophilic stippling of erythrocytes, increased delta-ALA, and peripheral neuropathy and other CNS changes.

MICHELLE JAY FRANCISCO, MD (TOP 9 - FEB 2013 MED BOARDS; TOPNOTCH MD)

Answer: E Liver cell adenomas may occur after several years of taking oral contraceptives but the actual mechanism of tumor formation is unknown.

MICHELLE JAY FRANCISCO, MD (TOP 9 - FEB 2013 MED BOARDS; TOPNOTCH MD)

MIDTERM 1 - AUG 2013

Answer: B IL-4, IL-10, IL-3 and TGF-B are all anti-inflammatory.

MICHELLE JAY FRANCISCO, MD (TOP 9 - FEB 2013 MED BOARDS; TOPNOTCH MD)

MIDTERM 1 - AUG 2013

Answer: B Solid tumor for < 2 y.o = neuroblastoma, > 2 y.o= Wilms tumor

MICHELLE JAY FRANCISCO, MD (TOP 9 - FEB 2013 MED BOARDS; TOPNOTCH MD)

MIDTERM 1 - AUG 2013

Answer: B Serum sickness- type III systemic; Goodpasture- type II; GVHD – type IV

MICHELLE JAY FRANCISCO, MD (TOP 9 - FEB 2013 MED BOARDS; TOPNOTCH MD)

MIDTERM 1 - AUG 2013

Answer: C Metabolic acidosis- initial compensation is respiratory. Hypoxia leads to anaerobic metabolism; volume replacement & not vasopressors will correct underperfusion.

MICHELLE JAY FRANCISCO, MD (TOP 9 - FEB 2013 MED BOARDS; TOPNOTCH MD)

MIDTERM 1 - AUG 2013

Answer: B Obstructive: Inc.TLC, FRC,RV but FEV1 is more dramatically reduced = dec.FEV1/FVC ratio

MICHELLE JAY FRANCISCO, MD (TOP 9 - FEB 2013 MED BOARDS; TOPNOTCH MD)

MIDTERM 1 - AUG 2013

TOPNOTCH MEDICAL BOARD PREP PATHOLOGY SUPEREXAM Page 96 of 99 For inquiries visit www.topnotchboardprep.com.ph or email us at [email protected]

TOPNOTCH EXAM MIDTERM 1 - AUG 2013

TOPNOTCH MEDICAL BOARD PREP PATHOLOGY SUPEREXAM For inquiries visit www.topnotchboardprep.com.ph or email us at [email protected] Item # 697

698

699

700

QUESTION

EXPLANATION

AUTHOR

TOPNOTCH EXAM MIDTERM 1 - AUG 2013

An 85 y.o male patient was recently found to have an apical lung mass. A few months after, patient was noted to have episodes of drooping of eyelids, pupillary constriction and anhidrosis. This condition is due to A. Neoplasm of neuroendocrine Kulchitsky cells B. Lambert-Eaton syndrome manifested as muscle weakness C. Ectopic secretion of serotonin known as carcinoid syndrome D. Compression of cervical sympathetic plexus The least important independent risk factor associated with increased risk for DVT: A. Obesity B. Central venous catheter C. Hospitalization with recent surgery D. Previous DVT

Answer: D Pancoast tumor- cause Horner’s syndrome (triad of ptosis, miosis, anhidrosis)

MICHELLE JAY FRANCISCO, MD (TOP 9 - FEB 2013 MED BOARDS; TOPNOTCH MD)

Answer: A Venostasis of lower extremities is asso.with prolonged bed rest, standing or sitting, my immobilization and ,muscular paralysis asso.with trauma and gen.& spinal anesthesia.

MICHELLE JAY FRANCISCO, MD (TOP 9 - FEB 2013 MED BOARDS; TOPNOTCH MD)

MIDTERM 1 - AUG 2013

Most tumors tend to metastasize via the lymphatics or blood vessels. Which among these deviate from the more common pattern of spread and metastasize over the surface of viscera or body cavities? A. Colon CA B. Gastric CA C. Mesothelioma D. Hepatoblastoma A 30 yr-old woman presents to her physician complaining of a recent nodular growth on her left neck. Further history reveals a 16-lbs weight loss and intermittent fevers over the past3-6 months. Physical examination reveals the presence of a firm, mobile, non-tender, 2.5 cm lower cervical lymph node, left neck. A biopsy is performed and pathology report indicates the presence of large cells with multilobate nuclei, abundant pale cytoplasm and significant band of sclerosis. Immunohistochemistry is positive for the presence of CD15 and CD30 cell markers. These findings are most consistent with which of the following neoplasms? A. Acute lymphoblastic leukemia B. HodgkinLymphoma C. Lymphoplasmacytic lymphoma D. Mantle zone lymphoma E. Mycosis fungoides

Answer: C Pattern of spread for mesothelioma is distinctive, it can even be diagnosed radiologically by a thick rind of tumor tissue it characteristically produces over the surface of involved lung.

MICHELLE JAY FRANCISCO, MD (TOP 9 - FEB 2013 MED BOARDS; TOPNOTCH MD)

MIDTERM 1 - AUG 2013

Answer: B Findings are consistent with nodular sclerosing HL, most common type (65-75%); cells described pertain to Reed-Sternberg cells.

MICHELLE JAY FRANCISCO, MD (TOP 9 - FEB 2013 MED BOARDS; TOPNOTCH MD)

MIDTERM 1 - AUG 2013



TOPNOTCH MEDICAL BOARD PREP PATHOLOGY SUPEREXAM Page 97 of 99 For inquiries visit www.topnotchboardprep.com.ph or email us at [email protected]

TOPNOTCH MEDICAL BOARD PREP PATHOLOGY SUPEREXAM For inquiries visit www.topnotchboardprep.com.ph or email us at [email protected] Item # 1 2 3 4 5 6 7 8 9 10 11 12 13 14 15 16 17 18 19 20 21 22 23 24 25 26 27 28 29 30 31 32 33 34 35 36 37 38 39 40 41 42 43 44 45 46 47 48 49 50 51 52 53 54 55 56 57 58 59 60 61 62 63 64 65 66 67 68 69 70 71 72 73 74 75 76 77 78 79 80 81 82 83 84 85 86 87

ANSWER D E C A C C B C E B D A B C D B B B B A C C A D C E B C C C C D B C E D C C D B B D D E A C D A E B A A C C B C B A B A D A C E D A D C B A D D A D C B C A D E B C C B C B D

88 89 90 91 92 93 94 95 96 97 98 99 100 101 102 103 104 105 106 107 108 109 110 111 112 113 114 115 116 117 118 119 120 121 122 123 124 125 126 127 128 129 130 131 132 133 134 135 136 137 138 139 140 141 142 143 144 145 146 147 148 149 150 151 152 153 154 155 156 157 158 159 160 161 162 163 164 165 166 167 168 169 170 171 172 173 174 175

B C A A B A C A C A B D C C C D A A B E D B C A E B E D A B C C E A or B A E B E B C C B B B B D A E A B C A D B C B D C D B C B B D C C C D D E B D B B E A C D A D C E A B C D E D

176 177 178 179 180 181 182 183 184 185 186 187 188 189 190 191 192 193 194 195 196 197 198 199 200 201 202 203 204 205 206 207 208 209 210 211 212 213 214 215 216 217 218 219 220 221 222 223 224 225 226 227 228 229 230 231 232 233 234 235 236 237 238 239 240 241 242 243 244 245 246 247 248 249 250 251 252 253 254 255 256 257 258 259 260 261 262 263

A E A D E D D A D A C B D A A C C C D B A A E C C C D B B C A D C B C D A C E C B B C D B C D D A A D B D C D D C B C A C A B A B E B A B D B D C D E C D B B C D C D B B C D E

264 265 266 267 268 269 270 271 272 273 274 275 276 277 278 279 280 281 282 283 284 285 286 287 288 289 290 291 292 293 294 295 296 297 298 299 300 301 302 303 304 305 306 307 308 309 310 311 312 313 314 315 316 317 318 319 320 321 322 323 324 325 326 327 328 329 330 331 332 333 334 335 336 337 338 339 340 341 342 343 344 345 346 347 348 349 350 351

A E A B D D B D A E D B C B C B A D C C B A A E D B C A B E C A E A C E B B C A D C B A D C B D E D D B A C C B C C D D A B C B D C A D D A B C D C D B D A C D D B D A B A B B

TOPNOTCH MEDICAL BOARD PREP PATHOLOGY SUPEREXAM Page 98 of 99 For inquiries visit www.topnotchboardprep.com.ph or email us at [email protected]

352 353 354 355 356 357 358 359 360 361 362 363 364 365 366 367 368 369 370 371 372 373 374 375 376 377 378 379 380 381 382 383 384 385 386 387 388 389 390 391 392 393 394 395 396 397 398 399 400 401 402 403 404 405 406 407 408 409 410 411 412 413 414 415 416 417 418 419 420 421 422 423 424 425 426 427 428 429 430 431 432 433 434 435 436 437 438 439

D A C B C B B A C E B D D B C A E B A D A E B A C A C E D C B B A B D A B A C A B B C E B D A C D B D C D D D C B E A A D A D C B D B A E B B D C A C D B C D A C C C B C D A B

TOPNOTCH MEDICAL BOARD PREP PATHOLOGY SUPEREXAM For inquiries visit www.topnotchboardprep.com.ph or email us at [email protected] 440 441 442 443 444 445 446 447 448 449 450 451 452 453 454 455 456 457 458 459 460 461 462 463 464 465 466 467 468 469 470 471 472 473 474 475 476 477 478 479 480 481 482 483 484 485 486 487 488 489 490 491 492 493 494 495 496 497 498 499 500 501 502 503 504 505 506 507 508 509 510 511 512 513 514 515 516 517 518 519 520 521 522 523 524 525 526 527 528

A D C A B D C C B C A D E C C D B B D C A B B D A C D B D E A C A D C A D B D D A B C A A C D A D B B B B C C C A D A B B B B E A C A B D C E B C D B A D C B B A A C A C C C D B

529 530 531 532 533 534 535 536 537 538 539 540 541 542 543 544 545 546 547 548 549 550 551 552 553 554 555 556 557 558 559 560 561 562 563 564 565 566 567 568 569 570 571 572 573 574 575 576 577 578 579 580 581 582 583 584 585 586 587 588 589 590 591 592 593 594 595 596 597 598 599 600 601 602 603 604 605 606 607 608 609 610 611 612 613 614 615 616 617

C C D D A D D B C D A B C D B D B E C A C E B E A B C A D B E C B B C A D B B C B B C D C D B B C B D B A A D E B D C A E A C B D D B C D B A B B A E E D A E A C A B A C D B C D

618 619 620 621 622 623 624 625 626 627 628 629 630 631 632 633 634 635 636 637 638 639 640 641 642 643 644 645 646 647 648 649 650 651 652 653 654 655 656 657 658 659 660 661 662 663 664 665 666 667 668 669 670 671 672 673 674 675 676 677 678 679 680 681 682 683 684 685 686 687 688 689 690 691 692 693 694 695 696 697 698 699 700

A C C C B A B E E B A D B C A D D C E A D C A B B E E B D E C E C E C C C C C E C E E A C D C B C B D D D B C A D B C B C A D D D C B B D A B D B E B B B C B D A C B

TOPNOTCH MEDICAL BOARD PREP PATHOLOGY SUPEREXAM Page 99 of 99 For inquiries visit www.topnotchboardprep.com.ph or email us at [email protected]

View more...

Comments

Copyright ©2017 KUPDF Inc.
SUPPORT KUPDF